2ちゃんねる ■掲示板に戻る■ 全部 1- 最新50    

■ このスレッドは過去ログ倉庫に格納されています

面白い問題おしえて〜な 二十問目

1 :132人目の素数さん:2012/12/22(土) 13:17:38.28 .net
過去ログ
http://www3.tokai.or.jp/meta/gokudo-/omoshi-log/
まとめwiki
http://www6.atwiki.jp/omoshiro2ch/

1 http://cheese.2ch.net/test/read.cgi/math/970737952/
2 http://natto.2ch.net/test/read.cgi/math/1004839697/
3 http://science.2ch.net/test/read.cgi/math/1026218280/
4 http://science.2ch.net/test/read.cgi/math/1044116042/
5 http://science.2ch.net/test/read.cgi/math/1049561373/
6 http://science.2ch.net/test/read.cgi/math/1057551605/
7 http://science2.2ch.net/test/read.cgi/math/1064941085/
8 http://science3.2ch.net/test/read.cgi/math/1074751156/
9 http://science3.2ch.net/test/read.cgi/math/1093676103/
10 http://science4.2ch.net/test/read.cgi/math/1117474512/
11 http://science4.2ch.net/test/read.cgi/math/1134352879/
12 http://science6.2ch.net/test/read.cgi/math/1157580000/
13 http://science6.2ch.net/test/read.cgi/math/1183680000/
14 http://science6.2ch.net/test/read.cgi/math/1209732803/
15 http://science6.2ch.net/test/read.cgi/math/1231110000/
16 http://science6.2ch.net/test/read.cgi/math/1254690000/
17 http://kamome.2ch.net/test/read.cgi/math/1284253640/
18 http://kamome.2ch.net/test/read.cgi/math/1307923546/
19 http://uni.2ch.net/test/read.cgi/math/1320246777/

2 :132人目の素数さん:2012/12/22(土) 13:33:14.21 .net
>>1

3 :あぼーん:あぼーん.net
あぼーん

4 :あぼーん:あぼーん.net
あぼーん

5 :132人目の素数さん:2012/12/28(金) 19:25:29.09 .net
ほんとだ

6 :132人目の素数さん:2012/12/31(月) 00:43:18.26 .net
てきとーに自作を投げっぱなしにしてみる
(√(15)-√(247))/(√(57)+√(65))

7 :132人目の素数さん:2012/12/31(月) 02:12:41.43 .net
>>6
それをどうしろと言うのかね?

     r;;;;;ノヾ
     ヒ =r=;'
     _ヽ二/_
  / ̄  /~〉  ̄ヽ
  / ヽ o ^~ o ノ l
 (  ヽ o   o ノ l
  ヽ  ノ    ヽノ
   ヽノ     ヽ
   ノ、____i
    l   l  l
    l   l  l
    l   l  l
    l   l  l
    .(__⌒)_⌒)

8 :謹賀新年:2013/01/04(金) 22:38:15.34 .net
集合Aは自然数の有限部分集合であって、n個の要素を持つ(n≧1)。
Aの部分集合うち、それに含まれる要素の総和が奇数となるものはいくつあるか?

9 :132人目の素数さん:2013/01/04(金) 23:43:48.08 .net
奇数の要素のうちから奇数個選んで偶数の要素は好きに選べばいい。

Aの要素のうち奇数であるものの個数をmとする。
このとき、Aの要素のうち偶数であるものの個数は(n-m)。

m≧1のとき、m個の奇数から奇数個選ぶ場合の数は
mC1 + mC3 + mC5 + mC7 + ……
= (1/2){(mC0 + mC1 + mC2 + mC3 + ……) - (mC0 - mC1 + mC2 - mC3 + ……)}
= (1/2){(1+1)^m - (1-1)^m}
= 2^(m-1)
で、偶数(n-m)個の方の選び方が2^(n-m)通りあるから、掛けて 2^(n-1)。
ただしAが奇数をひとつも含まない場合は0。

10 :132人目の素数さん:2013/01/05(土) 00:20:00.33 .net
B−{a}<−>B∪{a}。

11 :132人目の素数さん:2013/01/05(土) 01:13:17.33 .net
>>9
なるほど!

>>10
> <−>
この記号は何?

12 :132人目の素数さん:2013/01/05(土) 01:28:24.09 .net
>>11
1対1対応だろう

13 :132人目の素数さん:2013/01/05(土) 20:30:40.46 .net
(1/2)(2^m-0^m)(2^(n-m))=2^(n-1)-2^(n-m-1)0^m

m=0
2^(n-1)-2^(n-m-1)0^m=2^(n-1)-2^(n-1)0^0=0

14 :あぼーん:あぼーん.net
あぼーん

15 :132人目の素数さん:2013/01/05(土) 21:19:48.35 .net
Aが奇数aを含むとき、Aの部分集合全体を
「aを含まないもの全部」=X
「aを含むもの全部」=Y
と分類すると、XもYも2^(n-1)個の元からなる。

このとき、Xの元Bと、Yの元B∪{a}との対応は1対1で、
これら2^(n-1)個のどのペアも、どちらか片方のみが
要素の総和が奇数となっている。
ってことか。>>10

16 :132人目の素数さん:2013/01/05(土) 21:40:09.93 .net
だろうね>>15

17 :あぼーん:あぼーん.net
あぼーん

18 :132人目の素数さん:2013/01/05(土) 23:27:11.06 .net
なるほど、なかなか面白いな

19 :132人目の素数さん:2013/01/05(土) 23:38:30.56 .net
むむむ…

20 :132人目の素数さん:2013/01/06(日) 01:54:04.24 .net
x を複素数とするとき、√(x - 1/x) + √(1 - 1/x) = x を解くと
複素数の問題だから、根号内条件は使えないので、分母の条件 x≠0 だけで解くと、
x = (1±√5)/2 が出るんだけど、実際に代入すると、一方は見たさないんよな。
さて、計算過程で何か見落としがあるのかな?

21 :あぼーん:あぼーん.net
あぼーん

22 :132人目の素数さん:2013/01/06(日) 12:34:36.47 .net
>>20
そりゃあるんだろね。

23 :132人目の素数さん:2013/01/06(日) 14:28:57.20 .net
√(x-1/x)=±1.
√(1-1/x)=±(x-1).

24 :132人目の素数さん:2013/01/06(日) 15:42:45.44 .net
>>23
落ち着け

25 :132人目の素数さん:2013/01/06(日) 18:03:34.04 .net
>>20
x = (1-√5)/2
√(x-1/x) = 1
√(1-1/x) = -(1+√5)/2
で合うけど

26 :132人目の素数さん:2013/01/06(日) 18:16:39.22 .net
>>25
>> √(1-1/x) = -(1+√5)/2
おちつけ

27 :132人目の素数さん:2013/01/06(日) 18:44:42.75 .net
>>20>>24>>26

√が表すのをどちらか一方に決めるのなら
そのことも解くときに使わなくちゃいけないってだけだろ

28 :132人目の素数さん:2013/01/06(日) 19:00:42.52 .net
JCにも分かるように教えろ!

29 :132人目の素数さん:2013/01/06(日) 19:24:55.05 .net
わざわざ複素数って書いてるんだから、√は2価じゃないのか

30 :132人目の素数さん:2013/01/06(日) 20:25:38.46 .net
複素数を扱う式の中で
√4 = ±2は許されたっけ?

31 :132人目の素数さん:2013/01/07(月) 11:43:22.40 .net
複素数でも√aは1価じゃないの?

32 :132人目の素数さん:2013/01/07(月) 20:24:17.04 .net
>>31
だと思うけどなあ

x = (1-√5)/2 とする
x は x^2 - x - 1 = 0の解のひとつなので
x^2 - x = 1

√(1 - 1 / x)
= √{(x^2 - x) / x^2}
= √(1 / x^2)
= 1 / |x| > 0 > -(1 + √5) / 2

じゃないの?
>>25

33 :132人目の素数さん:2013/01/07(月) 22:53:51.88 .net
勝手に条件付け加えるならその条件も使わなきゃ関係ないものが入ってくることもあるさ

34 :132人目の素数さん:2013/01/10(木) 00:59:53.92 .net
じゃあ、例えばxが実数とか何も書かれていなくて、次を解けって問題があったらどう解くのでしょうか?
√(x^2-1) + √(x-1) = x√(x)

35 :132人目の素数さん:2013/01/10(木) 01:08:09.89 .net
何も書かれていないのは、書かなくても文脈/慣習から推測できるから省略しているだけだから
(方程式の「解」の概念を拡張しようと試みる場面は除く)

36 :132人目の素数さん:2013/01/10(木) 01:15:33.97 .net
>>34
C.1147
http://www.komal.hu/verseny/feladat.cgi?a=honap&h=201212&t=mat&l=en

37 :132人目の素数さん:2013/01/18(金) 18:42:56.06 .net
ひとりUNOが無限に終わらない順番パターンは存在するか?

38 :132人目の素数さん:2013/01/31(木) 00:41:25.62 .net
数学の問題じゃないんだけど、この図形のトリック分かる?

http://kie.nu/KZx

39 :132人目の素数さん:2013/01/31(木) 00:55:01.62 .net
斜辺が折れ線になっている(上では微妙にへこんでて下では微妙にふくらんでる)

40 :あぼーん:あぼーん.net
あぼーん

41 :132人目の素数さん:2013/02/03(日) 01:02:31.29 .net
緑と赤の直角三角形は斜線の傾きがそれぞれ
2/5と3/8。
目で見ると分からないけど傾きが違う

42 :132人目の素数さん:2013/02/03(日) 09:01:18.94 .net
まさかこのスレにそれを出す奴がいるとは思わなかった。

43 :132人目の素数さん:2013/02/05(火) 05:30:12.69 .net
△ABCにおいてBCの中点をM、BからACに降ろした垂線の足をHとする。
また、AMとBHの交点をPとする。AM=8、BM=4、BP・HP=12のとき
△ABCの面積として考えられるものをすべて求めよ。


知恵袋で見つけた

44 :あぼーん:あぼーん.net
あぼーん

45 :132人目の素数さん:2013/02/06(水) 02:25:55.58 .net
>>43
おもしろかった。

Mを中心とする半径4の円は、B,C,Hを通る。
この円が直線AMの延長から切り取る弦と、弦BHとに対し、
方羃の定理を適用すると、(4+PM)(4-PM)=12から
PM=2、AP=6となる。

次に線分AMとBHに対し、AP・PM = 12 = BP・PHが
成り立っていることから、方羃の定理の逆を適用すれば、
4点ABMHは同一円周上にあり、従って∠PMB=90°がわかる。

以上よりAM⊥BCなので、△ABCの面積は8*8/2=32。
他の可能性なんかあるのかな。

46 :132人目の素数さん:2013/02/07(木) 00:26:27.66 .net
△ABCが鋭角三角形の場合はそれで正解だね
∠Cが鈍角三角形の場合も考えてみて

47 :132人目の素数さん:2013/02/07(木) 00:30:53.58 .net
∠Cが鈍角三角形っておかしいなw
∠Cが鈍角のときだね

48 :132人目の素数さん:2013/02/09(土) 22:32:01.42 .net
この問題とかどう?
http://www.imslow.kr/ghost/

49 :132人目の素数さん:2013/02/10(日) 04:26:26.92 .net
>>48
開いてないが、URLでググったら、ブラクラらしい。

50 :132人目の素数さん:2013/02/10(日) 12:57:33.91 .net
.krの時点で見る気起きねえ

51 :132人目の素数さん:2013/02/10(日) 15:36:11.41 .net
定番ブラクラはNG済み

52 :132人目の素数さん:2013/03/20(水) 19:39:31.53 .net
J国とC国の領土が海を隔てて存在している。
海上に両国の国境を引き、国境上のどの地点から見ても
両国の領土までの最短距離が等しくなるようにしたい。
このように国境を定めることは可能だろうか?

53 :あぼーん:あぼーん.net
あぼーん

54 :132人目の素数さん:2013/03/20(水) 21:26:12.36 .net
ある条件のもとで可能

55 :あぼーん:あぼーん.net
あぼーん

56 :132人目の素数さん:2013/03/21(木) 07:23:17.83 .net
境界線が連続ならば可能じゃないの

57 :132人目の素数さん:2013/03/21(木) 10:33:25.07 .net
任意の実数x,yに対して f((x+y)/2)=(f(x)+f(y))/2 を満たす関数f(x)がある。
(1)f(x)のうち、不連続関数となるものがあるかを示せ
(2)f(x)を連続関数と仮定した場合におけるf(x)を求めよ

58 :132人目の素数さん:2013/03/21(木) 11:09:46.45 .net
定番の問題じゃねーか

59 :132人目の素数さん:2013/03/21(木) 11:19:44.72 .net
コーシーとはちゃうで

60 :132人目の素数さん:2013/03/21(木) 21:10:29.61 .net
>>34-36

√(x+1) = y とおいて

 x = y^2 -1 = (y-1)(y+1),

0 = (右辺)^2 - (左辺)^2
 = x^3 - (x-1)(y+1)^2
 = {x(y-1)^2 - (x-1)}(y+1)^2
 = {(x-y)(y+1)}^2
 = {(y^2 -y-1)(y+1)}^2,

y = -1, φ.
 でもいいが

61 :132人目の素数さん:2013/03/21(木) 21:17:07.74 .net
>>60 訂正、スマソ

 y = -1, φ, -1/φ
しかし定義から y≧0 なので
 y = φ = (1+√5)/2, 黄金比

62 :132人目の素数さん:2013/03/21(木) 21:54:32.15 .net
C.4508
a,b,c>0 のとき a^(3/4) + b^(3/4) + c^(3/4) > (a+b+c)^(3/4) を示せ。(h=201301)

C.1157
aは実数とする。2次方程式
 xx + ax + (1 - 1/aa) = 0 (a≠0)
が重根をもつ条件は? (h=201302)

B.4524
自然数上の関数gがすべての自然数nについて
 g(1) + g(2) + ・・・・・ + g(n) = n・g(n)
を満たすとき、g(k) = g(1) を示せ。(h=201303、改作)

63 :ななし:2013/03/21(木) 21:59:48.08 .net
>>62

C.4508
 a^(3/4) = a/a^(1/4) > a/(a+b+c)^(1/4),
循環的にたす。

C.1157
 判別式 = 0 から。

B.4524
 nについての数学的帰納法で。

64 :132人目の素数さん:2013/03/27(水) 00:46:11.72 .net
twitter で見た問題。

長方形 ABCD と、辺 CD 上の点 P がある。但し、AB=20、PD=6 とする。
半直線 BP が、辺 AD を延長した直線と交わる点を Q とするとき、△PCQ の面積を求めよ。

65 :132人目の素数さん:2013/03/27(水) 02:00:00.26 .net
CP・DQ=AD・DP。

66 :132人目の素数さん:2013/03/27(水) 22:26:07.33 .net
最近知った面白い数学の問題。

xy平面上に原点を中心とした半径1の円周がある。
この円周上のあらゆる点を2つのグループA、Bのいずれかにグループ分けするとする。(A、Bはそれぞれ連続でなくても良い。)
さて、グループAの点全体を原点を中心に一定の角度θ回転させたものをA'とするとき、「A'がBと重ならない」かつ「A'とBを合わせた全体が元の円周と一致しない」を満たすようなグループ分け方法及びθの一例を具体的に示せ

67 :132人目の素数さん:2013/03/27(水) 22:31:18.46 .net
むむ…
一見するとそんな方法はなさそうに思えてしまうな

68 :66:2013/03/27(水) 22:34:12.14 .net
ダメな例1:円周上の点をx軸からの角度で指定するとして、0°以上90°未満をA、90°以上180°未満をB、180°以上270°未満をA、270°以上360°未満をBとグループ分けして、
θを180°ととると、確かに「A'はBと重なっていない」が、「A'とBを合わせた全体が元の円周と一致してしまう」ので、題意満たさず。

ダメな例2:(1,0)、(0,1)、(-1,0)、(0,-1)をA、それ以外をBとし、θを90°ととった場合も例1同様題意満たさず。

69 :132人目の素数さん:2013/03/27(水) 23:12:44.99 .net
B以外の円周上の点からB以外の円周上の点への写像(回転)であって、
単射でもなく、全射でもないような写像の例を示せってことか?
そんなのあるのかな?

70 :132人目の素数さん:2013/03/27(水) 23:16:12.93 .net
αを無理数, P_n:=(cos(nαπ),sin(nαπ)) (n=1,2,...) として
A:={P_1,P_2,...}, B:=円周-A, θ:=απ とすれば A'={P_2,P_3,..}
A'⊆A, A'≠A で条件をみたす

71 :132人目の素数さん:2013/03/27(水) 23:20:21.79 .net
>>70
なるほどこれならいけるな
俺は立体射影で有理数と無理数に対応する点を使って考えてたけどうまくいかなかった

72 :132人目の素数さん:2013/03/28(木) 00:06:04.14 .net
>>70
A:={P_1,P_2,...}の最後の要素が、回転後Bと重なると思うんだけど

73 :132人目の素数さん:2013/03/28(木) 00:07:19.98 .net
閉集合を2つの開集合に分けることを分けたと言って良いなら
任意の開集合に分けた時点で題意は満たしたことになる気がする

例えばA∈(0,π)、B∈(π,2π)、θ=0
0とπが露骨に未定義なのが気に入らないなら
適当に境界に収束する関数を取ってもいい

74 :132人目の素数さん:2013/03/28(木) 00:09:16.46 .net
>>72
最後の要素とは
Aって円周上稠密になるんじゃないの?

75 :132人目の素数さん:2013/03/28(木) 22:57:51.99 .net
>>73
未定義の点があっちゃ駄目だろ。

具体的に。

76 :132人目の素数さん:2013/03/29(金) 01:02:45.90 .net
【問題】
p,q,rを実数, aを正の実数とするとき, 次の積分を工夫して計算せよ。

∫∫∫(px^2+qy^2+rz^2)dxdydz
但し, 積分領域はx^2+y^2+z^2≦a^2とする。

77 :132人目の素数さん:2013/03/29(金) 04:17:24.53 .net
∫∫∫(px^2+qy^2+rz^2)dxdydz=p∫x^2∫∫dydzdx+q∫y^2∫∫dxdzdy+r∫z^2∫∫dxdydz
∫x^2∫∫dydzdx=π∫x^2(a^2-x^2)dx

78 :◆yEy4lYsULH68 :2013/03/29(金) 05:15:18.84 .net
>>913
ソイツの馬鹿息子は痴漢行為で逮捕され、ほんで大学を懲戒解雇になった自慢
の息子なんやろ。親子揃って馬鹿丸出しや。世間の笑い者として有名やろが。

ケケケ狢

>913 :名無しさん:2013/03/20(水) 15:56:28 ID:???
> http://ja.wikipedia.org/wiki/%E5%A2%97%E7%94%B0%E8%8A%B3%E9%9B%84
>
> 芳雄のwiki
>

79 :132人目の素数さん:2013/03/29(金) 07:48:03.91 .net
アキレスと亀のような、
何一つ間違ってない過程から正しくない結論を導く話が好きだな

80 :132人目の素数さん:2013/03/29(金) 21:12:08.79 .net
2ch流の誤変換はこの場合はアウトーッ、だな。
仮定

81 :132人目の素数さん:2013/03/30(土) 10:55:52.68 .net
11^(13^(15^(17^(19^(...^(97^99)))...)の下二桁を求めよ

82 :132人目の素数さん:2013/03/30(土) 11:16:17.70 .net
与えられた円の中心をコンパスのみで図示せよ

83 :◆yEy4lYsULH68 :2013/03/30(土) 12:04:15.23 .net
>>913
ソイツの馬鹿息子は痴漢行為で逮捕され、ほんで大学を懲戒解雇になった自慢
の息子なんやろ。親子揃って馬鹿丸出しや。世間の笑い者として有名やろが。

ケケケ狢

>913 :名無しさん:2013/03/20(水) 15:56:28 ID:???
> http://ja.wikipedia.org/wiki/%E5%A2%97%E7%94%B0%E8%8A%B3%E9%9B%84
>
> 芳雄のwiki
>

84 :132人目の素数さん:2013/03/30(土) 13:39:13.18 .net
>>81
a=17^19^21^…^99とおく.
aは17の冪乗数で明らかに奇数だから,
15^a≡3 mod4
(∵(15^n)_{n=1,2,3,…}≡(3,1,3,1,3,1,…) mod4)

b=15^aとおくと,b≡3 mod4より,
13^b≡7 mod10
(∵(13^n)_{n=1,2,3,…}≡(3,9,7,1,3,9,7,1,…) mod10)

c=13^bとおくとc≡7 mod10より,c=10q+7とおける(q,r∈N)
d≡11^cとすると,
d=(10+1)^c
=Σ[i=0,c] C[c,i]10^c (C[c,i]は二項係数)
≡C[c,0]10^0+C[c,1]10^1 mod100
≡1+10c
≡1+10(10q+7)
≡71

以上のことから
11^(13^(15^(17^(19^(...^(97^99)))...)
=d
≡71 mod100
となるため,下二桁は71である.

85 :◆yEy4lYsULH68 :2013/03/30(土) 14:02:49.68 .net
>>913
ソイツの馬鹿息子は痴漢行為で逮捕され、ほんで大学を懲戒解雇になった自慢
の息子なんやろ。親子揃って馬鹿丸出しや。世間の笑い者として有名やろが。

ケケケ狢

>913 :名無しさん:2013/03/20(水) 15:56:28 ID:???
> http://ja.wikipedia.org/wiki/%E5%A2%97%E7%94%B0%E8%8A%B3%E9%9B%84
>
> 芳雄のwiki
>

86 :132人目の素数さん:2013/03/30(土) 15:05:48.46 .net
>>84
正解です

87 :◆yEy4lYsULH68 :2013/03/30(土) 15:39:33.56 .net
>>913
ソイツの馬鹿息子は痴漢行為で逮捕され、ほんで大学を懲戒解雇になった自慢
の息子なんやろ。親子揃って馬鹿丸出しや。世間の笑い者として有名やろが。

ケケケ狢

>913 :名無しさん:2013/03/20(水) 15:56:28 ID:???
> http://ja.wikipedia.org/wiki/%E5%A2%97%E7%94%B0%E8%8A%B3%E9%9B%84
>
> 芳雄のwiki
>

88 :132人目の素数さん:2013/04/02(火) 07:00:48.45 .net
2000から2999までの整数のうち、
3乗したものを一の位から3桁ずつ区切って和をとったものが
元の数に等しいものを全て求めよ

89 :132人目の素数さん:2013/04/02(火) 07:35:27.30 .net
…無いんじゃね?
それとも問題を読み違えたか…

90 :132人目の素数さん:2013/04/04(木) 21:56:27.91 .net
x≠yのとき次の2つの等式が同値であることを証明せよ
(x-1)x^(n+1)=(y-1)y^(n+1)
xy(x+y-1)^n=(x-1)(y-1)(x+y)^n

91 :132人目の素数さん:2013/04/04(木) 22:29:00.35 .net
無理。

92 :132人目の素数さん:2013/04/05(金) 22:45:44.45 .net
n個ある箱にm個のボールが入っているとき、最初にボールを見つける回数の期待値を求めよ

93 :132人目の素数さん:2013/04/05(金) 23:05:20.20 .net
問題が意味をなす時、箱を同時に開ければ良いので答えは1

94 :132人目の素数さん:2013/04/05(金) 23:44:52.49 .net
1つづつしか開けることはできないとした場合

95 :132人目の素数さん:2013/04/06(土) 00:03:20.57 .net
>>92が面白い問題になるような後出し条件を考えよ

96 :132人目の素数さん:2013/04/06(土) 00:31:25.02 .net
それは超難問だな

97 :132人目の素数さん:2013/04/06(土) 00:45:17.48 .net
ただし、箱には1つしかボールが入らないものとする

98 :132人目の素数さん:2013/04/11(木) 23:48:57.93 .net
今からコインを1秒に1回投げるゲームをする。
表が1000回連続、もしくは裏が1000回連続で出た時にこのゲームを終了する。
Nを1000以上の自然数とし、ゲーム開始からN秒後までにゲームが終了する、終了している確率をXとする。

(1)Xが99%を超える事は有り得るか?
(2)有り得るとしたらそれはNがいくつの時か?
(3)NとXの関係式を導いて下さい。

99 :132人目の素数さん:2013/04/12(金) 00:50:35.73 .net
表の確率=1 ならN=1000

100 :132人目の素数さん:2013/04/12(金) 01:30:23.74 .net
固有値の練習問題か

101 :132人目の素数さん:2013/04/13(土) 10:18:47.82 .net
【問題】

円に内接する四角形ABCDがある.

△ABC, △BCD, △CDA, △DABの内心をそれぞれI, J, K, Lとする.

四角形IJKLは長方形であることを証明せよ.

102 :132人目の素数さん:2013/04/14(日) 21:49:51.94 .net
ある私鉄会社の駅であるA駅は上りの一番ホーム、下りの2番ホーム、
そして支線に向かう3番ホームの3つのホームがある。だが支線用の
3番ホームは2番ホームとの共用で、支線から来た乗客が昇り路線
を利用するには反対側の1番ホームに行かねばならない。しかし3番ホーム
には階段しかないのだ。階段を上るのが嫌だという乗客がそのまま
2番ホームに来た列車に乗り込んで隣のB駅まで行ってそこでエスカレーター
に乗って上りの特急を使うという事は時間的な遅れ無しに可能だろうか?
因みにA駅では急行列車は停まるが特急は通過する。隣のB駅には特急が
停まる。

103 :132人目の素数さん:2013/04/14(日) 22:40:59.16 .net
鉄オタはキチガイ、まで読んだ

104 :132人目の素数さん:2013/04/14(日) 23:20:08.72 .net
西村京太郎に聞けば

105 :132人目の素数さん:2013/04/14(日) 23:47:46.74 .net
うむ。実のところこの問題(102)には正解は無い。だが多少の推論を
行う事は可能だ。実際想定されたような乗換が可能だったとしよう。
そうであれば、上り特急に乗りたい乗客は全員そうするだろうという
事だ。だがそれは鉄道会社にとって望ましい事だろうか。

106 :あぼーん:あぼーん.net
あぼーん

107 :132人目の素数さん:2013/04/15(月) 00:11:42.80 .net
経由した駅を正しく申告しなけりゃ無賃乗車だ

108 :132人目の素数さん:2013/04/15(月) 06:35:00.16 .net
数学というより算数って感じのパズルっぽい問題だけど解説が意味不明なので知恵を貸してください

【正方形3個からなる図形(図1)を組み合わせて長方形を作る。
このとき作られた長方形を図1の図形を二個組み合わせた長方形(図2)で分割しうる場合と分割し得ない場合がある。
たとえば図1の図形を組みあわせて図3のような長方形を作れば、この長方形は図2の図形で分割する事ができる。
それでは図1の図形を組み合わせて作ることのできる長方形で、かつ図2の図形では分割し得ないのはどれか。】

図1は↓のようなL字 図2は↓のLを2個組み合わせた縦3*横2の長方形 図3は図2を縦に2個横に2個並べた6*4の長方形

□□

選択肢
1 縦3*横4の長方形
2 縦5*横6の長方形
3 縦4*横9の長方形
4 縦5*横9の長方形
5 縦5*横10の長方形

分かりますか?

109 :132人目の素数さん:2013/04/15(月) 07:35:47.49 .net
5は3の倍数で無いから問題外
1,2,3は図2で分割できることがすぐ分かる
答えがあるなら4しかない

実際4は図1で作れる

110 :132人目の素数さん:2013/04/15(月) 08:56:29.24 .net
解説も同じことをいっていて
3の倍数であり6の倍数でないのは4っていってます
でもその考えだと例えば3*3の9マスの正方形も3の倍数であり6の倍数でないですよね
この正方形L字の図形3つで組み合わせることなんてできなくないですか?

111 :132人目の素数さん:2013/04/15(月) 09:04:31.53 .net
>>110
解説は少し端折ってるんだと思うよ。
>>109さんの言うように、1、2、3、5は除外される。
4は、6の倍数ではないので「図2の図形では分割し得ない」を満たし、
かつ、3の倍数なので図1の図形を組み合わせて作ることのできる」の“候補”だってだけ。
実際作れるかどうかは確認が必要で>>109さんはちゃんと言及している。

112 :132人目の素数さん:2013/04/15(月) 09:22:39.70 .net
できるものを探すというよりできないものを省くという考え方をしてるんですね
ありがとうございました

113 :132人目の素数さん:2013/04/15(月) 15:46:47.67 .net
出来るものを全て探してるんだろ

114 :132人目の素数さん:2013/04/15(月) 23:12:16.84 .net
0〜9までの10個の数字を4つ選ぶとき(重複あり)特定の数字4桁に一致する確率は
1/10*1/10*1/10*1/10で1/10000ですよね?
数字の順番は気にせず特定の4桁の4つの数字と一致する確率というのはどう考えればいいんでしょうか
コンビネーション使おうと思ったんですが重複ありだと10C4とはいかず
ひとつ数字選んで箱に戻してまたひとつ選んでって考えだと
(10C1)^4で特定の4桁と順番まで一致する確率と同じになってしまいます

115 :132人目の素数さん:2013/04/15(月) 23:27:20.63 .net
>>114
例えば1111に一致する確率と1234に一致する確率は等しくない

116 :132人目の素数さん:2013/04/16(火) 00:20:22.35 .net
なるほど
では順番は関係なく数字だけ一緒な確率はx/10000
特定の数字が4つともバラバラならx=4!
2つ同じで2つバラバラならx=4!/2!
3つ同じで1つだけ違うならx=4!/3!
4つとも一緒ならx=1

であってますか?

117 :132人目の素数さん:2013/04/16(火) 18:31:29.86 .net
このスレが最適かな?
自分で作ったわけでは無いけど、法則性あるらしい・・・。
おれには無理だorz


Q. XとYを求めよ。
■27
5490 25090 19600 39200 44690
7410 33810 26400 52800 60210

■26
6200 28800 22600 45200 51400
6300 28300 22000 44000 50300

■25
6000 27800 21800 43600 49600
6200 28600 22400 44800 51000

■26
6500 X ? ? ?
6000 Y ? ? ?

118 :132人目の素数さん:2013/04/17(水) 15:34:13.77 .net
オカルト板に「エスパー検定」ってスレは無いの?
ここは、数学板だよ?

119 :132人目の素数さん:2013/04/19(金) 17:41:02.93 .net
規則性なし、かな?

120 :132人目の素数さん:2013/04/21(日) 22:52:23.63 .net
http://i.imgur.com/JTRWCgk.jpg

121 :132人目の素数さん:2013/04/22(月) 00:05:21.69 .net
16/5

122 :132人目の素数さん:2013/04/22(月) 00:17:22.49 .net
8/3

123 :132人目の素数さん:2013/04/28(日) 00:19:16.43 .net
3の倍数と3が付く数字に☆マークを付けていく。
無限に☆マークを付けていった時☆マーク率は何%に近づくか?

124 :132人目の素数さん:2013/04/28(日) 11:29:10.27 .net
100

125 :132人目の素数さん:2013/04/28(日) 12:31:22.03 .net
Π(パイ)を無限に表記したとき、同じ数字が1まんこ並ぶ事はあるか?ないか?
そしてそれを証明せよ。

126 :132人目の素数さん:2013/04/28(日) 13:54:47.80 .net
無理

127 :132人目の素数さん:2013/04/28(日) 14:13:46.85 .net
証明不可能であることを証明せよ

128 :132人目の素数さん:2013/04/28(日) 14:31:34.05 .net
問題文くらいまともに書け

129 :132人目の素数さん:2013/04/28(日) 15:44:47.26 .net
証明不可能であることを証明できないことなら証明できる

130 :132人目の素数さん:2013/04/28(日) 15:56:10.38 .net
>>124
桁が増えるほど3を含む割合が高くなっていくだろうことは分かるけど、
それを示す方法が分からん

131 :132人目の素数さん:2013/04/28(日) 16:22:39.97 .net
p>1-(9/10)^n

132 :132人目の素数さん:2013/04/29(月) 14:03:37.64 .net
上の右辺は先頭を固定したn+1桁の数で先頭以外に3を含まない数の割合
n+1桁の数で3を含まない数の割合pはそれより大きい

133 :132人目の素数さん:2013/04/30(火) 23:55:13.46 .net
AB=6、BC=5、CA=4の△ABCがある。
∠Aの二等分線とBCの交点をD、
∠Bの二等分線とCAの交点をEとし、
CからADに下ろした垂線の足をF、
CからBEに下ろした垂線の足をGとおくとき、FGの長さを求めよ。

134 :132人目の素数さん:2013/05/01(水) 00:11:09.91 .net
再録かよ

135 :132人目の素数さん:2013/05/01(水) 02:45:41.79 .net
>>133

第二余弦定理より、
 cos(∠A) = 9/16,
 cos(∠B) = 3/4,
 cos(∠C) = 1/8,
∴ ∠C = 2∠B,


AE : EC = AB : BC = 6 : 5,
AC=4 より、 AE=24/11, EC=20/11.

BD : CD = AB : AC = 6 : 4,
BC=5 より、BD=3, CD=2.

AF : FD = 5 : 1,
より AF=(5/6)AD, FD=(1/6)AD.
AD=3√2.

BG : GE = 11 : 1,
より BG=(11/12)BE, GE=(1/12)*BE.
BE=(12/11)*5√(7/8).

一方、CF=√(7/2), CG=5/√8,

よって、FG=3/2.

136 :132人目の素数さん:2013/05/01(水) 03:47:42.49 .net
>>135
中学生の問題です

137 :132人目の素数さん:2013/05/01(水) 07:57:06.36 .net
それが何か?

138 :132人目の素数さん:2013/05/01(水) 09:37:06.56 .net
6点(0,0),(1,0),(2,0),(0,1),(1,1),(2,1)を考え
点(0,1)から点(2,0)まで移動する最短経路を考える。
n=1以下のように表記する。
□□
n=2の場合は以下であり、点(0,2)から点(3,0)までを考慮する。
□□
□□□
n=3を同様に以下とするとき、最短経路の数Pnを求めよ。
□□
□□□
□□□□

139 :132人目の素数さん:2013/05/01(水) 09:50:50.62 .net
┌┬┐   こういうことだろうか
└┴┘
┌┬┐
├┼┼┐
└┴┴┘
┌┬┐
├┼┼┐
├┼┼┼┐
└┴┴┴┘

140 :132人目の素数さん:2013/05/01(水) 10:24:18.57 .net
そう

141 :132人目の素数さん:2013/05/01(水) 11:25:12.70 .net
カタラン

142 :132人目の素数さん:2013/05/01(水) 12:05:22.81 .net
C[n+2]-C[n+1] かな

143 :132人目の素数さん:2013/05/01(水) 13:10:52.65 .net
nのときの図に対して、n+1個の横マスを下段に追加した図がn+1のときの図、ということ?

144 :132人目の素数さん:2013/05/01(水) 13:17:25.24 .net
nのときの図に対して、n+2個の横マスを下段に追加した図がn+1のときの図

145 :132人目の素数さん:2013/05/01(水) 13:20:54.09 .net
n+2か。最初が2個だった

146 :132人目の素数さん:2013/05/01(水) 13:33:47.06 .net
>>142でいいんじゃないの

147 : ◆6LZ.cs02lU :2013/05/01(水) 19:41:45.52 .net
1,2,3,4,5に続く数
(漢数字酉)

148 :132人目の素数さん:2013/05/02(木) 14:58:58.89 .net
>>136
中学の知識で解いてほしいのか?
いやだね、教えてやらねーYO!

149 :132人目の素数さん:2013/05/02(木) 18:19:50.60 .net
じゃん

150 :132人目の素数さん:2013/05/02(木) 18:27:35.30 .net
三角形の秘密はね♪

151 :132人目の素数さん:2013/05/02(木) 18:49:34.86 .net
ガイシンナイシンスイシンジュウシンボウシンシンシン、ヘイヘイヘイッ♪

152 :132人目の素数さん:2013/05/02(木) 21:13:11.72 .net
>>120>>133

DQNにも分かる解法を教えてください

153 :132人目の素数さん:2013/05/02(木) 21:15:53.41 .net
DQNに教えたって無駄

154 :132人目の素数さん:2013/05/02(木) 21:35:49.49 .net
120は相似比と1:√2くらいで解けるけど133を中学生チックに解くのは俺にはできん

155 :132人目の素数さん:2013/05/02(木) 23:08:18.86 .net
AB//FG

156 :132人目の素数さん:2013/05/03(金) 18:46:38.41 .net
任意の四角形において
対辺の積の和は必ず対角線の積以上になるってやつ
AB×CD + AD×BC ≧ AC×BD
中学のときに習ったけど何で未だにこうなるかわからん

157 :あのこうちやんは始皇帝だった:2013/05/03(金) 19:39:55.15 .net
テメ〜ら、いいかげんにしねえと、ブッ殺すぞ!

 無職の、知的障害の、女性恐怖症の、頭デッカチの虚弱児・ひ弱の、ゴミ・クズ・カス・無能・虫けらのクソガキども!

 死ね!!!!!!!!!!!!!!!!!!!!!!

158 :132人目の素数さん:2013/05/03(金) 20:01:38.83 .net
>>156
http://www004.upp.so-net.ne.jp/s_honma/ptolemaios/ptolemaios.htm

159 :132人目の素数さん:2013/05/03(金) 21:46:01.60 .net
>>155
やっと分かった
CF、CGを延長してABと交わる点をH、Iとすると、
△BCGと△BHGが合同だから△BCHはBC=BHの二等辺三角形
同様に△ACIもAC=AIの二等辺三角形
BH=BC=5だからAH=1、
AI=AC=4、AH=1だからHI=3
F、Gはそれぞれ二等辺三角形の底辺の中点で、△CHIでHIとFGが平行、中点連結定理でFG=HI/2=3/2

160 :132人目の素数さん:2013/05/04(土) 01:51:20.20 .net
>>159
神と呼ばせてください

161 :あぼーん:あぼーん.net
あぼーん

162 :132人目の素数さん:2013/05/09(木) 20:30:46.97 .net
水槽の中に魚が200匹います。そのうち99%がグッピーです。ここからグッピーのみを取り出して、グッピーの、魚全体に対する割合を98%にしたいと思います。何匹取り出せば良いでしょうか。
ちなみにマイクロソフトの入社問題です。

163 :132人目の素数さん:2013/05/09(木) 20:37:42.85 .net
100?

164 :132人目の素数さん:2013/05/09(木) 22:00:50.82 .net
やっぴー グッピー うれピーな

165 :132人目の素数さん:2013/05/09(木) 22:01:22.56 .net
面白いのか?

166 :あぼーん:あぼーん.net
あぼーん

167 :132人目の素数さん:2013/05/09(木) 22:13:24.05 .net
マイクロソフトのこと時々マイケルソフトって読んじゃう

168 :132人目の素数さん:2013/05/10(金) 01:01:51.53 .net
>>162
普通に計算すれば答えは簡単だけど、即答出来る人材かどうかを知りたいんだろうね、マイクロソフト側としては
マイクロソフトの入社試験では金貨100枚を分けあう問題が秀逸だった

169 :あぼーん:あぼーん.net
あぼーん

170 :132人目の素数さん:2013/05/10(金) 02:09:09.71 .net
>>168
並みいる秀才たちを困惑させた「マイクロソフトの入社試験問題」集 − 全20問
http://ameblo.jp/zero-123-456-789/entry-11118132469.html

50組の夫婦のいる村の男全員が不貞をしています、という問題、解答見たけどさっぱり理解出来ん
誰か解説してくれ

長方形のケーキがあります、という問題には別解を思いついた

171 :132人目の素数さん:2013/05/10(金) 02:45:26.83 .net
数学じゃなくてなぞなぞみたいなものだな
つか面白くない

172 :132人目の素数さん:2013/05/10(金) 07:04:13.29 .net
マイクロソフトだったっけ


An=n^(n-1)^(n-2)^(n-3)^....^2^1

Anを、nから1までの数を下から累乗で積み重ねた数とする。
A1=1 A2=2^1 A3=3^2^1
A97は、97^96^95^94^93^......^2^1で、巨大数となる。

このとき、(A97-A1)(A97-A2)(A97-A3)....(A97-A98)(A97-A99)
はいくつになるか。

173 :132人目の素数さん:2013/05/10(金) 07:11:02.62 .net
>>172
何故 (A97-A97) は書かないのか
ソフトウェア業界の体質を象徴しているのか?

174 :132人目の素数さん:2013/05/10(金) 08:46:23.52 .net
>>170
不貞の夫が一人だけなら、その妻は必ず気付く。
なぜなら、自分の夫以外の49人の夫は不貞をしていないことが
分かっているはずなので、候補は自分の夫のみになるから。
だから、1日目に誰も殺されないということは、
不貞の夫は一人でないことを示す。

N日目には、不貞の夫がN人ちょうどであれば、その妻にはわかる。
他の不貞の夫はN-1人しかいないが、N-1日目までの推論で
不貞の夫がN-1人以下であることはない(N人以上いる)から。

N=50日目に、不貞の夫が全部で50人いて、
自分の夫も不貞をしていることが分かる。

175 :132人目の素数さん:2013/05/10(金) 09:32:31.44 .net
>>174
でも実際には、少なくとも自分の夫以外の49人が不貞してて、
他の妻から見ても48人以上不貞してる事は分かる事は
1日目の段階で明らかだよな?

そんな、不貞の夫が1人しかいないという
明らかに偽の仮定をしないといけないのか?

176 :132人目の素数さん:2013/05/10(金) 09:50:12.16 .net
仮定から夫全員不貞してるじゃん

177 :132人目の素数さん:2013/05/10(金) 10:03:14.66 .net
推論の順番が逆だよな。

前提条件として全ての妻は49人の確定した不貞男と
1人の不確定な不貞男(自分の夫)がいることを知っている。
従って、妻達の興味の対象は他の妻が確定した不貞男を
48人と思っているか、49人と思っているかである。
これについて、女王の発言は何の情報も与えない。
故に、何もおきない。

不貞男が1人しかいないと考える妻がいないことは明らかであり、
同様にN<48日目までの推論についての議論は無意味。

もし仮に妻達が何らかの方法で夫の不貞を察知できるなら
初日から全員不貞を知ってて法を無視している状態のはずだから、
即日皆殺しだな。

178 :132人目の素数さん:2013/05/10(金) 10:08:11.18 .net
もしかして、
「妻たちは村中の夫全員が不貞を働いているのを知っているが、自分の夫の
不貞をについては分からない」
というバカ妻揃いという想定?

179 :132人目の素数さん:2013/05/10(金) 10:23:15.54 .net
>>175
逆からたどっていけばいいんじゃないか?

議論を分かりやすくするために50人の妻の中から1人を選んで(仮に妻1と名づける)妻1の視点から考えることにする。
妻1には自分の夫以外の49人が不貞夫であることは分かっているので、そこから
自分以外の妻には少なくとも48人の不貞夫が見えていることが分かる。
そこで妻1は、議論を分かりやすくするために自分以外の49人の妻の中から1人を選んで(仮に妻2と名づける)
常に妻2の視点から考えることにしよう、と考えるだろう。そこで妻1は次のように考える。
妻2には自分の夫を除いて少なくとも48人が不貞夫であることは分かっているので、そこから
不貞夫を持つ48人の妻には少なくとも47人の不貞夫が見えていることが分かる。
そこで妻2は、議論を分かりやすくするために不貞夫を持つ48人の妻の中から1人を選んで(仮に妻3と名づける)
常に妻3の視点から考えることにしよう、と考えるだろう。そこで妻2は次のように考える、と妻1は考える。
妻3には自分の夫を除いて少なくとも47人が不貞夫であることは分かっているので、そこから
不貞夫を持つ47人の妻には少なくとも46人の不貞夫が見えていることが分かる。
そこで妻3は、議論を分かりやすくするために不貞夫を持つ47人の妻の中から1人を選んで(仮に妻4と名づける)
常に妻4の視点から考えることにしよう、と考えるだろう。そこで妻3は次のように考える、と妻2は考える、と妻1は考える。
……

頭が痛くなってきたのでここら辺でやめておく

180 :132人目の素数さん:2013/05/10(金) 11:25:15.50 .net
>>170の完璧な解法】
俺からすれば、50人全員が不貞夫であることは分かっているので
そこからその50人の妻には自身の夫を除く49人の不貞夫が見えていることが
俺には分かる。(その中の1人を妻1と名付ける)
妻1からすれば、自分の夫を除く49人が不貞夫であることは分かっているので
そこからその49人の妻には自身の夫を除く48人の不貞夫が見えていることが
妻1には分かる、ということが俺には分かる。(その中の1人を妻2と名付ける)
妻2からすれば、自分の夫を除く48人が不貞夫であることは分かっているので
そこからその48人の妻には自身の夫を除く47人の不貞夫が見えていることが
妻2には分かる、ということが妻1には分かる、ということが俺には分かる。(その中の1人を妻3と名付ける)
……
妻49からすれば、自分の夫を除く1人が不貞夫であることは分かっているので
そこからその1人の妻には自身の夫を除く0人の不貞夫が見えていることが
妻49には分かる、…、ということが妻1には分かる、ということが俺には分かる。(その1人を妻50と名付ける)
妻50からすれば、自分の夫を除く0人が不貞夫であることは分かっているので
そこから自分の夫が不貞夫でなければこの村には不貞夫はいないということが
妻50には分かる、ということが妻49には分かる、…、ということが妻1には分かる、ということが俺には分かる。
【以下につづく】

181 :132人目の素数さん:2013/05/10(金) 11:25:48.31 .net
>>180のつづき】
ところが、少なくとも一人の不貞夫がいることが判明してしまった。自分の夫が不貞夫でないとすると、これは矛盾である。
この時点で妻50は自分の夫が不貞夫であることが分かるので1日目に処刑するはずだろうことが
妻49には分かる、ということが妻48には分かる、…、ということが妻1には分かる、ということが俺には分かる。
ところが、誰も処刑されることなく2日目を迎えてしまった。自分の夫が不貞夫でないとすると、これは矛盾である。
この時点で妻49は自分の夫が不貞夫であることが分かるので2日目に処刑するはずだろうことが
妻48には分かる、…、ということが妻1には分かる、ということが俺には分かる。
……
ところが、誰も処刑されることなく49日目を迎えてしまった。自分の夫が不貞夫でないとすると、これは矛盾である。
この時点で妻2は自分の夫が不貞夫であることが分かるので49日目に処刑するはずだろうことが
妻1には分かる、ということが俺には分かる。
ところが、誰も処刑されることなく50日目を迎えてしまった。自分の夫が不貞夫でないとすると、これは矛盾である。
この時点で妻1は自分の夫が不貞夫であることが分かるので50日目に処刑するはずだろうことが
俺には分かる。
【証明終わり】

182 :132人目の素数さん:2013/05/10(金) 11:31:19.88 .net
1日目2日目……ってのはおかしい気がするけどなあ。
1日目に何も起きない段階で推論は完成して2日目に全員殺害になるんじゃないか?

183 :132人目の素数さん:2013/05/10(金) 11:46:44.95 .net
実際に50日経たないと矛盾してることは分からないよ

184 :132人目の素数さん:2013/05/10(金) 12:15:24.52 .net
女王の発言前後で状況が変わったことといえば、以下ぐらいしかないよな。

●発言前
・夫は、「自分の妻と他の夫が、村に不貞夫がいるかどうか知っているか」を知らない
●発言後
・夫は、「自分の妻と他の夫が、村に不貞夫がいることを知っている」ことを知った


女王が発言した内容は村の夫・妻は全員知ってるから、
「周知した」ということしか意味がない。

185 :132人目の素数さん:2013/05/10(金) 12:58:55.16 .net
>>170の50組の夫婦の質問文は
「女王が発言しました。どうなるでしょう」
だから、リンク先の解説が誤りで
「何も起こらない」がMS的正解かもしれないな。

186 :132人目の素数さん:2013/05/10(金) 13:42:22.28 .net
今となっては良く知られてるクイズの一種だから、ググれば色々と解説出てくると思うよ
wikipediaの共有知識なんかも要参照

「全員が知っている」、「全員が知っている、ということを知っている」、「全員が知ってる、ということを知っている、ということを知っている」・・・
という情報(知識)はそれぞれ別物であることがポイント
ただし >>170のリンク先の問題文では条件が足らないから、解説のような推論は成り立たない
(>>170の解説・答えを不自然・非現実的に感じるのは、勝手に不自然・非現実的な条件を仮定してるから)

187 :132人目の素数さん:2013/05/10(金) 14:23:05.04 .net
面白く脚色したつもりだろうが、つまらない上に曖昧さだけを表面化させた感じ
結果としてただのとんち、もしくは条件逆算問題

188 :あぼーん:あぼーん.net
あぼーん

189 :132人目の素数さん:2013/05/10(金) 21:20:58.71 .net
最初の段階で、全妻が「全妻が×夫は48人以上であることを知っている」ことを知っている。
従って、×夫が48人以下ならその妻は自分の夫が×夫であることがわかるので殺害するはずだが殺害されない。
すると2日目の朝には、全妻が「全妻が×夫は49人以上であることを知った」ことを知ることになる。
従って、×夫が49人以下ならその妻は自分の夫が×夫であることがわかるので殺害するはずだが殺害されない。
すると3日目の朝には全妻が×夫は50人いることを知ることになり、全妻が夫を殺害する。

女王がやってくるまえに全夫は殺害されているはず。

190 :132人目の素数さん:2013/05/10(金) 21:27:33.84 .net
>>189
>50組の夫婦のいる村の男全員が不貞をしています。
この文はこの問題の読者に対してであって、村の人がこのことを知っているわけではない。

村の人が分かっていることは
>女はみな、自分の夫以外の男が不貞をすれば即座にわかります。でも自分の夫が不貞をしてもわかりません。
>村の掟では不貞をはたらいた夫の妻は、夫を即日殺さなければなりません。
という2点だけ

191 :132人目の素数さん:2013/05/10(金) 21:45:20.59 .net
>>190
全妻には他の妻の夫が全員×夫だとわかっているのだから、
どの妻も「他の妻全員が少なくとも48人×夫がいることを知っている」と知ることになるだろ。

192 :132人目の素数さん:2013/05/10(金) 21:48:10.59 .net
>>190
> この文はこの問題の読者に対してであって、村の人がこのことを知っているわけではない。
当たり前だろ。知ってたら1日目で終わるわw

193 :132人目の素数さん:2013/05/10(金) 22:10:11.88 .net
>>191
事実A:「全妻には自分の夫以外の49人の×夫が見えている」

>>189の1行目は事実Aから導かれるが2行目では事実Aに反する仮定をしてるな

同じように見えても>>174は事実Aを使ってなくて一般のNに関する数学的帰納法を使っている

194 :132人目の素数さん:2013/05/10(金) 22:14:31.52 .net
>>193
>>174はいきなり事実に反する仮定をしてるじゃんか。

195 :132人目の素数さん:2013/05/10(金) 22:16:00.98 .net
> 不貞の夫が一人だけなら
この仮定は事実に反しないのか?

196 :132人目の素数さん:2013/05/10(金) 22:21:25.69 .net
>>194-195
そうじゃなくて、本題の50人っていうのをN人に一般化して問題を解いてるんだよ

197 :132人目の素数さん:2013/05/10(金) 22:25:23.40 .net
解けてねえって話だよ

198 :132人目の素数さん:2013/05/10(金) 22:31:23.21 .net
えっ、具体的な数値を一般化して解くのは数学ではよくある解法だと思うけど…
それとも>>174の証明は間違ってるってこと?

199 :132人目の素数さん:2013/05/10(金) 23:05:38.55 .net
>>174が正しかったら>>189も正しくね?
なんで、>>189に対しては事実に反する仮定をしているからダメって言って、
>>174には言わないんだ?
>>174の仮定は事実に反してるだろ?

200 :132人目の素数さん:2013/05/10(金) 23:14:00.89 .net
>>199
>>189は1行目では事実Aに基づいた仮定をしていて、2行目では事実Aに反する仮定をしてる
つまり、1行目と2行目とで相反する仮定を使ってる。それなのに1行目の結論を2行目に適応してるから矛盾なんだよ

201 :132人目の素数さん:2013/05/10(金) 23:15:04.89 .net
>>200
>>174もそうだよ。

202 :132人目の素数さん:2013/05/10(金) 23:15:56.14 .net
>>201
>>189の2行目がどうして従うのか分からないんだが、解説して

203 :132人目の素数さん:2013/05/10(金) 23:18:07.79 .net
>>201
>>174のどことどこが相反する仮定を使ってるの?

204 :132人目の素数さん:2013/05/10(金) 23:32:26.98 .net
女王の台詞が悪い気がする。

「夫が不貞を働いたと思う人は挙手してください」
と言う質問を繰り返したときに、何回目で不貞が
露呈するかという問題なら素直に理解できる。

205 :132人目の素数さん:2013/05/10(金) 23:55:32.02 .net
>>204
そうか?
その方式でも、本質的なところは変わってないように思うが

206 :132人目の素数さん:2013/05/11(土) 00:02:24.40 .net
問題:無理数の無理数乗で有理数となるものが存在することを示せ」
※高校数学の範囲で証明できます

207 :132人目の素数さん:2013/05/11(土) 00:29:31.52 .net
>>206
分からない問題はここに書いてね360
http://kamome.2ch.net/test/read.cgi/math/1316096657/418

208 :132人目の素数さん:2013/05/11(土) 00:32:31.87 .net
p=log(q)

209 :132人目の素数さん:2013/05/11(土) 07:43:01.55 .net
個人的にはウィキの「共有意識」の説明が分かりやすい
島民10人のうち、3人の目が青で7人の目が緑の場合、
7人には青い目の人が3人見えるが、3人には2人しか見えない

210 :132人目の素数さん:2013/05/11(土) 08:46:07.78 .net
事実に反する仮定って意味があるの?
「不貞夫が一人」は偽なんだから、「不貞夫が一人ならその妻は気づかない」も真になってしまわないの?

211 :132人目の素数さん:2013/05/11(土) 08:55:00.62 .net
Wikiの説明もそうだけど、本当に共有されているのは
「不貞夫が1人以上存在すること」ではなくて、
「お互いが不貞夫を何人いるはずだと思っているか」という
推論のステップであって、推論の同期をとることが本質的。

「不貞夫が1人以上存在する」という発言で、
全員の推論段階がN=1に同期すると言いたいのだろうが、
他人の思考が同期したことを確信できる情報量が無いと
自然な解釈とは思えないな。

212 :132人目の素数さん:2013/05/11(土) 12:03:26.60 .net
共通意識って、今月中に抜き打ちテストをやるっていう話に似てるね
抜き打ちだから、生徒が全く予期出来ないタイミングでやらなければならない
となると31日の実施は無理、何故なら30日が過ぎた時点で31日の実施が予期出来てしまうから
となると30日の実施も無理、何故なら29日が・・・・・・・・・ 結局テストを実施出来る日は存在しないという話

共通意識もテストの話も、理屈は分かるんだが釈然としないものが残るね、なにかがおかしい気がする
ああいう連鎖って本当に存在するのかなぁ

213 :132人目の素数さん:2013/05/11(土) 12:04:40.80 .net
>>212 訂正

共通意識 → 共有意識

214 :132人目の素数さん:2013/05/11(土) 14:22:13.97 .net
共有知識
http://ja.wikipedia.org/wiki/%E5%85%B1%E6%9C%89%E7%9F%A5%E8%AD%98

青い目の人が最低でも一人はいるというアナウンスは、青い目の人が4人以上いるケースでは必要ないと思う
むしろ必要なのは共通のゲーム開始時間

215 :132人目の素数さん:2013/05/11(土) 14:44:18.07 .net
>>210
仮定してるのは「不貞夫が一人」じゃないぞ
あくまでも仮定の大枠は「自分の夫は不貞でない(不貞夫は自分の夫以外の49人)」だ


>>212
>>186でも書いたけど共有知識の仮定って不自然で非現実的な仮定だから
その結果が不自然・非現実的に見えてしまっても当たり前
推論がちゃんと行われる為には
「全員頭がいい(演繹的に推論できる)」「全員頭がいいと知っている」「そのこと自体を知っている」「そのこと自体を(ry」・・・
という仮定などが必要だが、現実世界ではそんな知識はまず知り得ない

216 :132人目の素数さん:2013/05/11(土) 14:57:36.48 .net
>>214
「最低でも1人いる」というアナウンスは必要だよ
島の掟を「明日○月×日から施行する」などという設定にすれば、共通のゲーム開始時間を作れるが
アナウンスがなければ、元の問題の時と同じような推論はできない(仮定の矛盾を示せない)

217 :132人目の素数さん:2013/05/11(土) 16:26:59.16 .net
>>212
連鎖って厄介な問題だよね、人間の頭脳では捉えられないようになってるのかも
カントのアンチノミーに追加していいのかもしれん

2つの封筒問題スレ 4
http://uni.2ch.net/test/read.cgi/math/1319861681/
上のスレでも一時期連鎖が話題になってた
「二つの封筒を用意して、片方にはもう片方の二倍の金額を入れる、最小値は1円、
片方を開封した被験者にもう片方の金額がバレてはいけない」という問題
ここでも連鎖によって困ったことが起きる
15円30円のペアが無理なのは言うまでもない、もし被験者が15円を開封したらもう片方が30円だとバレてしまう
となると30円60円も無理、すでに15円30円があり得ないと分かってるんだから、
被験者が30円を開封した時点でもう片方が60円だとバレてしまう
となると60円120円も無理・・・・・・・・・・

共有意識も、抜き打ちテストも、二つの封筒も、全て連鎖が絡んでる

218 :132人目の素数さん:2013/05/11(土) 17:35:07.46 .net
現実の世界なら
「相手も知っているかもしない」
「相手に自分の考えが読まれてるかもしれない」
ぐらいのことを考えるのがやっと
不確かなことしか解らない
たからこそ相手の裏をかいたりもできるが、裏の裏をかかれる可能性もある

219 :132人目の素数さん:2013/05/11(土) 20:33:03.55 .net
永久に亀の後ろを走り続けてればいいと思うよ

220 :132人目の素数さん:2013/05/21(火) 04:03:46.25 .net
128×128のチェス盤からマス目を1個除いたものはL字牌で敷き詰められることを証明せよ

221 :132人目の素数さん:2013/05/21(火) 07:30:36.25 .net
3×2

222 :あぼーん:あぼーん.net
あぼーん

223 :132人目の素数さん:2013/05/21(火) 23:17:32.59 .net
3×2、5×9
2×2、5×5

224 :132人目の素数さん:2013/05/28(火) 21:00:46.76 .net
ベタかな。
(アナログ)時計で、夜の0時0分から、翌日の0時0分までに、長針と短針が重なるのは何回か?ただし0時0分は除くものとする。

225 :132人目の素数さん:2013/05/28(火) 21:24:30.12 .net
0<x<1440
6x-x/2=360n ∴x=720n/11
n=1〜21

226 :132人目の素数さん:2013/05/28(火) 22:02:30.58 .net
↓これって、オマイラが歌ってるんだよな?
http://www.youtube.com/watch?v=yxOiP5SxocY&sns=em

227 :132人目の素数さん:2013/05/29(水) 14:25:36.00 .net
太郎くんと花子ちゃんが商店街で買い物に行ったとする。
二人が帰りに廃校舎に遊びに行き二時間後に帰宅。
およそ三ヶ月後に花子ちゃんが吐き気をもようしたと仮定した場合
この問題の登場人物が三人になっている確率は?

228 :132人目の素数さん:2013/05/29(水) 21:39:05.48 .net
体K上の多項式 f(X) = X^3-3X-1 ∈ K[X] は、
K内に少なくとも1根を持つものとする。
このとき、fの重複度を込めた3つの根は
全てKに含まれることを示せ。

229 :132人目の素数さん:2013/05/31(金) 09:48:29.07 .net
>>228
ちょいとズルいかもしれんが

まず複素数で考えてみると、
X=2Yとおけば、f(X)=8Y^3-6Y-1
f(X)=0⇔4Y^3-3Y=1/2
Y=cos20゚,cos140゚,cos260゚はこれを満たす。(3倍角の公式 cos3θ=4(cosθ)^3-3cosθ より)
よって、f(X)の根は 2cos20゚,2cos140゚,2cos260゚。
ここで、α=2cos20゚とおくと、
2cos140゚=-2cos40゚=-2(2(cos20゚)^2-1)=-α^2+2
と書ける。

これを踏まえ、一般の体で考える。
f(X)のK内における根の一つをαとすると、
f(X)=(X-α)(X^2+αX+α^2-3) と因数分解できる。
g(X)=X^2+αX+α^2-3 とおく。g(X)がK内に一つ根を持てば、残り一つもKに含まれる。
g(-α^2+2)
=(-α^2+2)^2+α(-α^2+2)+α^2-3
=α^4-4α^2+4-α^3+2α+α^2-3
=α^4-α^3-3α^2+2α+1
=(α^3-3α-1)(α-1)
=0
より、-α^2+2はg(X)の根。
したがって、f(X)の根は全てKに含まれる。□

230 :132人目の素数さん:2013/05/31(金) 12:53:24.66 .net
なるほどー

231 :132人目の素数さん:2013/06/02(日) 05:19:51.90 .net
正八面体の一つの面を床に置いた時、真上から見たらこの図形はどう見えるか。[出典・T大]

232 :132人目の素数さん:2013/06/02(日) 20:29:34.74 .net
正六角形

233 :231:2013/06/04(火) 05:57:11.87 .net
>232
もう少し詳しく。

ちなみにT大は駒場にある大学ね。

234 :132人目の素数さん:2013/06/04(火) 08:11:07.45 .net
床と平行な正三角形とその辺それぞれにくっついた斜めの二等辺三角形
とでも言えばいいのか

235 :132人目の素数さん:2013/06/04(火) 09:48:53.11 .net
http://gascon.cocolog-nifty.com/photos/uncategorized/2009/08/22/c147_02.png

236 :233:2013/06/06(木) 04:23:32.40 .net
>234
まあ…。

正六角形の中に六芒星があるように見える、とかそんな感じか。実際には作図させる問題みたいだが。

237 :◆yEy4lYsULH68 :2013/06/06(木) 06:19:32.49 .net
馬鹿はその存在が無駄なんや。そやし馬鹿は居なくてもエエのやナ。

ケケケ狢

238 :132人目の素数さん:2013/06/06(木) 07:47:23.91 .net
>>236
>>235を見る気はないのか?

239 :◆yEy4lYsULH68 :2013/06/06(木) 09:11:27.26 .net
馬鹿板は無駄。



240 :132人目の素数さん:2013/06/06(木) 09:49:23.77 .net
6点(1,0,0),(0,1,0),(0,0,1),(-1,0,0),(0,-1,0),(0,0,-1)
平面x+y+z=-1を床とする。この平面上にない3点から床に下ろした垂線の足は
(1,0,0)→(1/3,-2/3,-2/3)
(0,1,0)→(-2/3,1/3,-2/3)
(0,0,1)→(-2/3,-2/3,1/3)
∴6点(1,0,0),(0,1,0),(0,0,1),(1/3,-2/3,-2/3),(-2/3,1/3,-2/3),(-2/3,-2/3,1/3)により囲まれる図形

241 :あぼーん:あぼーん.net
あぼーん

242 :132人目の素数さん:2013/06/06(木) 12:44:18.53 .net
3y+2= 2y+3  

それぞれ移項して、3y-3=2y-2

3(y-1) =2(y-1)

両辺 (y-1) で割って 、3=2

あれっ、 3=2に

243 :あぼーん:あぼーん.net
あぼーん

244 :132人目の素数さん:2013/06/06(木) 12:46:06.54 .net
三人の女性が3000円のゲームを買うことにした。

三人が1000円ずつ出し合い3000円を店員に渡したところ、奥に入った店員は主人から「少し古いので500円まけてやりな。」といわれた。

ところがこの店員は「500円では半端だ。三人なので300円まけたことにして、200円は俺がもらっておこう。」と考え、女性に300円返した。

仲良し三人組は100円ずつ分け合った。

三人は最初1000円ずつ出したがあとで100円返してもらったので、結局各人900円出したことになる。

支払った三人の900円の合計と店員のポケットに入れた200円を合計すると2900円になる。

100円はどこに消えたんだろうか?

245 :あぼーん:あぼーん.net
あぼーん

246 :132人目の素数さん:2013/06/06(木) 13:07:15.10 .net
意味不明な計算してるだけだ。あまりにも今さらだし。

247 :あぼーん:あぼーん.net
あぼーん

248 :132人目の素数さん:2013/06/06(木) 13:14:15.44 .net
2500(ゲームの代金)+200(俺の取り分)=3000(初めの支払い)-300(まけた代金)

249 :あぼーん:あぼーん.net
あぼーん

250 :132人目の素数さん:2013/06/06(木) 17:13:27.34 .net
>>242
0で割るなし

251 :132人目の素数さん:2013/06/06(木) 18:26:09.16 .net
>>242
計算の過程でどこかに偶然ゼロが含まれてしまうこともあるから注意しなくてはいけない、
という警告としての価値があるね、その書き込み
なかなか面白い

252 :236:2013/06/06(木) 19:03:23.78 .net
>238
文字化けしてて見れなかった。手書き画像をアップしてあるとか?

253 :132人目の素数さん:2013/06/06(木) 19:13:00.03 .net
>>252
http://blog-imgs-43.fc2.com/s/e/i/seishinnyc/c147_02.png

254 :132人目の素数さん:2013/06/07(金) 20:05:46.98 .net
数列
4,6,7,9,10,11,12,14,□,・・・
数学好きならすぐ分かるかな、有名だし

255 :132人目の素数さん:2013/06/07(金) 20:08:20.62 .net
つまらん

256 :132人目の素数さん:2013/06/07(金) 20:15:15.44 .net
数学好きだけどさっぱりわからんし聞いたことも見たこともない

257 :132人目の素数さん:2013/06/07(金) 20:23:13.60 .net
フィボナッチ数を除いた自然数列

258 :132人目の素数さん:2013/06/07(金) 20:47:40.02 .net
>>257正解!

259 :132人目の素数さん:2013/06/07(金) 21:02:33.88 .net
フィボナッチ数を除いた自然数列は、項番nの初等関数として表すことは出来るか?

260 :132人目の素数さん:2013/06/07(金) 21:37:43.62 .net
F[k]={(1+√5)^k/√5} ({m}:mに最も近い整数) を使って
nまでに何個フィボナッチ数が存在するかを求めて…みたいな?
うーん、しかしこれでは初等関数にはならないなぁ

261 :あぼーん:あぼーん.net
あぼーん

262 :252:2013/06/08(土) 05:05:22.79 .net
>253
ごめん、やっぱり文字化けしてる。
この携帯ダメだぁ〜。7年前に買ったやつだし。

263 :132人目の素数さん:2013/06/08(土) 12:36:01.57 .net
>>235>>253も文字化けなど見えん、図しかないぞ

264 :132人目の素数さん:2013/06/08(土) 14:11:36.56 .net
>>262
PCで見ようぜ…

265 :262:2013/06/09(日) 20:22:11.76 .net
>264
パソコンはネットに繋がっていない。

266 :132人目の素数さん:2013/06/12(水) 19:43:39.85 .net
>>220
2^n×2^nのチェス盤について考える
(i)n=1のとき、
L字牌1つで埋まる

(ii)n=kのとき成り立つと仮定する

n=k+1のとき、
2^(k+1)×2^(k+1)=4×2^k×2^kより、
2^k×2^kのチェス盤を四方に並べたものとして考える

ここで、2^(k+1)×2^(k+1)のチェス盤の中央(2^k×2^kのチェス盤の角が互いに接し合う場所)にL字牌を置くと、L字牌が重なっている2^k×2^kのチェス盤は1マス除かれた状態のため、仮定からL字牌で敷き詰められる
また、L字牌が重なっていない2^k×2^kのチェス盤から1マス抜けば、仮定からL字牌で敷き詰められる
(i),(ii)から数学的帰納法より成り立つ
したがってn=7のときの
128×128のチェス盤でも成り立つ

267 :132人目の素数さん:2013/06/14(金) 22:07:11.25 .net
aを定数として、次の不等式を解け。
ax−2<a^2・x^2−4<ax+2
[法政大]

268 :132人目の素数さん:2013/06/14(金) 22:22:36.79 .net
>>220>>266
立方体でも類似のことが云えるな。

269 :132人目の素数さん:2013/06/14(金) 22:54:22.94 .net
>>267
(3/2)^2<(ax-1/2)^2<(5/2)^2

270 :132人目の素数さん:2013/06/14(金) 23:29:54.62 .net
つまらん。

271 :132人目の素数さん:2013/06/14(金) 23:58:44.17 .net
計算ドリル問題のどこが面白いやら

272 :132人目の素数さん:2013/06/15(土) 00:09:51.04 .net
a=b
a^2=ab
a^2-b^2=ab-b^2
(a+b)(a-b)=b(a-b)
a+b=b
b+b=b
2b=b
2=1

なんかこれ思い出したわwwwww

273 :132人目の素数さん:2013/06/15(土) 00:24:06.66 .net
0で割るやつがあるか

274 :132人目の素数さん:2013/06/15(土) 00:25:52.10 .net
そんな大人ぶらなくたって・・・

275 :132人目の素数さん:2013/06/15(土) 07:07:56.30 .net
数的処理もここでいいのかな

8F建ての建物に設置されているエレベーターがいFから上昇して8Fに到着するまでの間に
A〜Eの5人がそれぞれ乗り降りをした
5人が次のように述べているとき1〜5の中で確実にいえるのはどれか
なお、同じ階である人が乗り、別の人が降りた場合、この2人は乗り合わせたことにはならない

A「私は乗った階から3つ上の階で降りた」
B「私は4Fで降りた。Aと同じ階で乗ったが、降りた階は異なる階だった」
C「私はAが降りた階で乗り、乗った階から2つ上の階で降りた」
D「私は乗った階から2つ上の階で降りた。私は誰とも乗り合わせなかった」
E「私は既に下の階から乗っていたAと乗り合わせCと一緒に降りた」

1 Aは6Fで降りた
2 Bは2Fで乗った
3 Cは7Fで降りた
4 Dは6Fで乗った
5 Eは4Fで乗った

よろしくお願いします

276 :132人目の素数さん:2013/06/15(土) 07:53:44.33 .net
>いF

277 :132人目の素数さん:2013/06/15(土) 07:56:13.49 .net
1Fのミスです
階と変換するのがめんどくてF使ってますが原文は全部階で統一してあります

278 :132人目の素数さん:2013/06/15(土) 08:15:40.33 .net
難しいな
どこがどう面白いのかさっぱりわからん

279 :132人目の素数さん:2013/06/15(土) 08:46:40.41 .net
(A) Ai + 3 = Ao
(B) Bo = 4, Bi <= 3, Bi = Ai, Ao ≠ Bo
(C) Ci = Ao, Co = Ci + 2
(D) Do = Di + 2, Di >= Ao,Bo,Co,Eo
(E) Ei > Ai, Eo = Co

(C)までの条件で
Ai 1 2 3
Ao 4 5 6
Bi 1 2 3
Bo 4
Ci 4 5 6
Co 6 7 8
となるが、(D)の条件でCi=Ao=4となり矛盾。

280 :132人目の素数さん:2013/06/15(土) 11:00:00.21 .net
1.
D.
2.
D.
3.
A,B.
4.
A,(E).
5.
A,E.
6.
C,E.
7.
C,E.
8.

281 :132人目の素数さん:2013/06/15(土) 11:13:41.18 .net
なお、同じ階である人が乗り、別の人が降りた場合、この2人は乗り合わせたことにはならない

282 :132人目の素数さん:2013/06/15(土) 11:35:29.56 .net
ちなみに答え1です
アプローチの仕方教えてください

283 :132人目の素数さん:2013/06/15(土) 12:14:17.02 .net
>>275
Bの証言から、Aは4Fで降りていない。
上とAとCの証言から、「Aが2F→5F、Cが5F→7F」または「Aが3F→6F、Cが6F→8F」。
上とDの証言から、「Aが3F→6F、Bが3F→4F、Cが6F→8F、Dが1F→3F」で確定。
上とEの証言から、「Eが4F、5F→8F」。乗った階は確定しない。
よって、1○ 2× 3× 4× 5×。

284 :132人目の素数さん:2013/06/15(土) 12:20:25.25 .net
>>283はちょっとだけ端折ってるけど、Aの証言から順に愚直に吟味するだけの問題じゃねえか。

Aの証言からAは1→4、2→5、3→6、4→7、5→8のいずれか。
以下、>>283と同様。

285 :132人目の素数さん:2013/06/15(土) 12:22:12.64 .net
スケジュール表を埋めるだけの作業だしな

286 :132人目の素数さん:2013/06/15(土) 12:40:25.20 .net
>>283-284
なるほど、解説きくとけっこうすんなりいくもんですね
ありがとうございます

287 :132人目の素数さん:2013/06/15(土) 15:10:48.18 .net
(D)は、Di >= Ao,Bo,Co,Eo
ともとれるが、Do <= Ai,Bi,Ci,Ei
にもなるのか...

288 :132人目の素数さん:2013/06/16(日) 06:51:59.79 .net
数学史上、一旦確立した定理が覆っちゃったことってありますか?

289 :132人目の素数さん:2013/06/16(日) 07:04:35.92 .net
「確立」とは?

290 :132人目の素数さん:2013/06/16(日) 08:00:28.57 .net
確立=学会が認定
学会すら無かった時代は対象外で

291 :132人目の素数さん:2013/06/16(日) 09:17:51.07 .net
近代では無いんじゃないか?
未確定なものは未確定として予想扱いにしてただろう。
誰かが言ったから採用なんてのはアリストテレスとかの時代じゃね?

292 :132人目の素数さん:2013/06/16(日) 12:56:02.72 .net
学会は認定なんかしないだろ
個々人が認めるだけさ

293 :132人目の素数さん:2013/06/16(日) 14:08:54.12 .net
宇宙定数・・・は物理か。

294 :132人目の素数さん:2013/06/16(日) 14:12:03.29 .net
クイックソートの最初の論文には誤りが有ったけど、
30年間、誤りが正されなかったんだっけ。

295 :132人目の素数さん:2013/06/16(日) 22:40:29.82 .net
数学基礎論の分野で何か無いかな

296 :132人目の素数さん:2013/06/18(火) 02:09:21.69 .net
公理が定理になることはある

297 :132人目の素数さん:2013/06/18(火) 19:59:22.83 .net
そんなのあったっけ?
ぱっと思いつかんのだが

298 :132人目の素数さん:2013/06/18(火) 22:10:48.18 .net
>>297
例えばヒルベルトの幾何学基礎論にある定理の一つ「1直線上に任意の4点が与えられたとき、これらの点をA,B,C,Dで表し、A#B#CかつA#C#DかつB#C#Dとすることが常に可能である(ただし、点Xが点Y,Zの間にある関係をY#X#Zで表す)」
というのは元々公理だったけど後に他の順序公理から導けることがわかったから定理になった

299 :132人目の素数さん:2013/06/20(木) 23:04:01.73 .net
>>267
 >>269 の続き...

 3/2 < |ax - 1/2| < 5/2,
∴ -5/2 < ax -1/2 < -3/2 または 3/2 < ax -1/2 < 5/2,
∴ -2 < ax < -1 または 2 < ax < 3,

・a>0 のとき
 -2/a < x <-1/a または 2/a < x < 3/a,

・a<0 のとき
 3/a < x < 2/a または -1/a < x < -2/a,

・a=0 のとき
 解なし。

300 :132人目の素数さん:2013/07/07(日) NY:AN:NY.AN .net
関数f(x)は、次の条件@、Aを満たしている。
@f'(0)=a
Aすべての実数x、yに対してf(x+y)=f(x)+f(y)

(1)f'(x)を求めよ。
(2)f(x)=f(1)xを示せ。
[大阪市大]

301 :132人目の素数さん:2013/07/08(月) NY:AN:NY.AN .net
1/17 = 0.058823529411....なのだが
588^2 + 2353^2 = 5882353 が成り立つことを計算せずに
1/17から説明しなさい。

302 :132人目の素数さん:2013/07/08(月) NY:AN:NY.AN .net
>>301
 1/17 なので、

 n = 6*(10^2 -2) = 588 とおくと、
 2353 = 4n+1,
 17n = (10^2 +2)(10^2 -2) = 10^4 -4,
 5882353 = (10^4 +4)n +1
     = (10^4 -4)n +8n +1
     = (17n)n +8n +1
     = n^2 + (4n+1)^2,

303 :132人目の素数さん:2013/07/10(水) NY:AN:NY.AN .net
これは面白い。
出典はどこ?

304 :132人目の素数さん:2013/07/10(水) NY:AN:NY.AN .net
2^29 は9桁の数で、各桁の数字がすべて異なる。
0〜9のうち、この数の桁に現れない数字を、2^29を直接書き下す以外の方法で決定せよ。

305 :132人目の素数さん:2013/07/10(水) NY:AN:NY.AN .net
(2^29の各桁の数字の和)=2^29≡(2^3)^9*4≡-4≡5 mod9
一方0+1+2+3+…+9=45≡0 mod9
∴現れない数字は4

306 :132人目の素数さん:2013/07/11(木) NY:AN:NY.AN .net
>>303
588^2+2352^2を計算しなさいという問題があり、成立の理由を調べたら17=4^2+1との関係がわかった。

307 :132人目の素数さん:2013/07/11(木) NY:AN:NY.AN .net
すばらしい炯眼

308 :132人目の素数さん:2013/07/20(土) NY:AN:NY.AN .net
f(x+1)g(x-1)-g(x+1)f(x-1)=1
任意のxに対して成り立つから、xをx+1、x-1に置換した
f(x)g(x-2)-g(x)f(x-2)=1
g(x)f(x+2)-f(x)g(x+2)=1
が成立する。両辺を引くと
f(x){g(x-2)+g(x+2)}-g(x){f(x-2)+f(x+2)}=0
よって、ある実数aに対して以下の式が成立する。
a*f(x)=f(x-2)+f(x+2)
a*g(x)=g(x-2)+g(x+2)

1. a≠2のとき
x^2-ax+1=0の2解をα、βとすると
f(x+2)-αf(x)=β{f(x)-αf(x-2)}
h(x)=f(x+2)-αf(x)とおくと
h(x)=βh(x-2)
h(x)=C4(√β)^x+C5(-√β)^x、C4,C5は定数 …@
f(x+2)-βf(x)=α{f(x)-βf(x-2)}
k(x)=f(x+2)-βf(x)とおくと
k(x)=αk(x-2)
k(x)=C6(√α)^x+C7(-√α)^x、C6,C7は定数 …A
@,Aから
(β-α)f(x)=C4(√β)^x+C5(-√β)^x-C6(√α)^x-C7(-√α)^x
f(x)=C0(√α)^x+C1(-√α)^x+C2(√β)^x+C3(-√β)^x、C0,C1,C2,C3は定数

2. a=2のとき
f(x+2)-2f(x)+f(x-2)=0
f(x+2)-f(x)=f(x)-f(x-2)
f(x+2)-f(x)=Cとすると
f(x)=C/2*x+C0+C1(-1)^x、C0,C1は定数

309 :132人目の素数さん:2013/07/20(土) NY:AN:NY.AN .net
>>308
> よって、ある実数aに対して以下の式が成立する。

なぜ?

310 :132人目の素数さん:2013/07/20(土) NY:AN:NY.AN .net
a=2のとき、を以下に訂正
f(x+2)-2f(x)+f(x-2)=0
f(x+2)-f(x)=f(x)-f(x-2)
f(x+2)-f(x)=C4+C5(-1)^x、C4,C5は定数とすると
f(x)-f(x-2)=C4+C5(-1)^(x-2)=f(x+2)-f(x)
ここで
f(x)=C4/2*x+C1+(C5/2*x+C3)(-1)^x、C1,C3は定数
とすると
f(x+2)-f(x)=C4/2*(x+2)+C1+(C5/2*(x+2)+C3)(-1)^(x+2)-(C4/2*x+C1+(C5/2*x+C3)(-1)^x)
=C4+C5(-1)^x
となるので、C0=C4/2, C2=C5/2として
f(x)=C0*x+C1+(C2*x+C3)(-1)^x

311 :132人目の素数さん:2013/07/20(土) NY:AN:NY.AN .net
>>309
a*f(x)=f(x-2)+f(x+2)かつa*g(x)=g(x-2)+g(x+2) ⇒ f(x){g(x-2)+g(x+2)}-g(x){f(x-2)+f(x+2)}=0
は自明。逆は知らない。

312 :132人目の素数さん:2013/07/20(土) NY:AN:NY.AN .net
逆が問題なわけだが

313 :132人目の素数さん:2013/07/20(土) NY:AN:NY.AN .net
f(x)(g(x-2)+g(x+2))=g(x)(f(x-2)+f(x+2))=bとすると
g(x)=b/(f(x-2)+f(x+2))
g(x-2)=b/(f(x-4)+f(x))
g(x+2)=b/(f(x)+f(x+4))
f(x)*(b/(f(x-4)+f(x))+b/(f(x)+f(x+4)))=b
f(x)*(f(x)+f(x+4)+f(x-4)+f(x))=(f(x-4)+f(x))(f(x)+f(x+4))
f(x)(f(x+4)+2f(x)+f(x-4))=f(x)^2+(f(x+4)+f(x-4))f(x)+f(x+4)f(x-4)
f(x)^2=f(x+4)f(x-4)

314 :132人目の素数さん:2013/07/20(土) NY:AN:NY.AN .net
a*f(x)=f(x-2)+f(x+2)
f(x+4)=a*f(x+2)-f(x)
f(x-4)=a*f(x-2)-f(x)
f(x+4)f(x-4)=(a*f(x+2)-f(x))(a*f(x-2)-f(x))
=f(x)^2+a*(f(x+2)+f(x-2))*f(x)+a^2*f(x+2)*f(x-2)
となりa=0?

315 :132人目の素数さん:2013/07/20(土) NY:AN:NY.AN .net
>>313
b→b(x)だった.…

316 :132人目の素数さん:2013/07/22(月) NY:AN:NY.AN .net
三角形の内部にあるn個の点によって、この三角形は2n+1個の領域に三角形分割されることを証明せよ

317 :132人目の素数さん:2013/07/22(月) NY:AN:NY.AN .net
え?

318 :132人目の素数さん:2013/07/22(月) NY:AN:NY.AN .net
またポエマーかよ。
今回はどんだけ後出しするのやら。

319 :132人目の素数さん:2013/07/22(月) NY:AN:NY.AN .net
普通に帰納法使うかすれば解けるんじゃね?
どこが面白い問題なんだか

320 :132人目の素数さん:2013/07/22(月) NY:AN:NY.AN .net
2回くらい後出しが必要かw

321 :132人目の素数さん:2013/07/22(月) NY:AN:NY.AN .net
多分、最初の三角形の頂点も含めて、どの3点も一直線上にはないものとする、
くらいは、出てくるかな

322 :132人目の素数さん:2013/07/22(月) NY:AN:NY.AN .net
エスパーしたところによれば、それは要らないと出た

323 :132人目の素数さん:2013/07/22(月) NY:AN:NY.AN .net
>>321
それ俺が昼ごろ書こうとしたが考えなおしたら不要だと気づいてやめた文言じゃないか

324 :132人目の素数さん:2013/07/24(水) NY:AN:NY.AN .net
a,b(≧2)を互いに素な整数とする。
整数m,n(≧0)がm+n=ab-a-bを満たすとき、
mとnのどちらか一方のみが
ax+by(x,yは非負整数)
という形で表せることを示せ。

325 :324:2013/07/24(水) NY:AN:NY.AN .net
m,nの≧0という条件は不要だった

326 :132人目の素数さん:2013/07/25(木) NY:AN:NY.AN .net
アナログで最強のソートはどれか考えたい。

トランプのようなカードに
1000以下の数字が一様平均ランダム&重複ありで書かれている。
全部で100枚程度ある。
数字の小さい順にソートするとき、
平均計算量が一番少なくなるのはどのアルゴリズムか?
道具はなくて広い部屋に裸で閉じ込められたみたいなシュールな状況を想像してほしい

327 :132人目の素数さん:2013/07/25(木) NY:AN:NY.AN .net
あ、床は自由に使ってよしで

328 :132人目の素数さん:2013/07/25(木) NY:AN:NY.AN .net
数字は1〜1000の自然数を想定
動きまわるのも、分類が多すぎるのも、作業効率かえって低くなりそうなんで

(1)1の桁だけでまず分類する
(2)分類し終わったら1の桁が、0が下〜9が上となるよう順に重ねる
(3)同じように上のカードから10の桁だけで分類する
(4)同じように分類し終わったら10の桁が、0が下〜9が上となるよう順に重ねる
(5)同じように100の桁だけで分類
(6)同じように100の桁が、0が下〜9が上となるよう順に重ねる
(7)1000だけ補正作業

ただし分担作業する場合は他の人もこの方法について理解している必要がある

329 :132人目の素数さん:2013/07/25(木) NY:AN:NY.AN .net
……(6)だけ9が下〜0が上でよかった

330 :132人目の素数さん:2013/07/25(木) NY:AN:NY.AN .net
人手でやるならバケットソート系列が良いだろう
経過が分かり易いしミスったときも挿入し易い
100枚程度なら手の届く範囲で並べられるから
メモリコストも気にしなくて良い

例えば>>328の方法を上の桁からやればいい

331 :132人目の素数さん:2013/07/26(金) NY:AN:NY.AN .net
>>328
上限が1000なら、壁から数字mm離して置いていけば、
1mのソート済みカード列が出来るな。

332 :132人目の素数さん:2013/07/26(金) NY:AN:NY.AN .net
プログラム的にもそれが最速だろうな

333 :132人目の素数さん:2013/07/26(金) NY:AN:NY.AN .net
右手にソート前、左手にソート済みを持ってバブルソートじゃない?
床に比べてメモリアクセス効率がいいぞ

334 :132人目の素数さん:2013/07/26(金) NY:AN:NY.AN .net
1mm単位で調整なんて俺にはそんな手早くできないが

335 :132人目の素数さん:2013/08/17(土) NY:AN:NY.AN .net
4面体の4つの面にそれぞれ0,1,2,3の数字が書かれてあり、
投げた時にそれぞれの面が下を向く確率は1/6,1/3,1/3,1/6とする。
このとき、下を向いた面に書かれている数を「出目」と呼ぶことにすると、
出目を2で割った余りが0,1になる確率はそれぞれ1/2であり、
出目を3で割った余りが0,1,2になる確率はそれぞれ1/3である。
この4面体は、出目を2および3で割った余りがそれぞれ等確率となる、
面の数が最小のサイコロである。

さて、今度は出目を2,3,5で割った余りがそれぞれ等確率となるものを作りたい。
ただし、出目となる数は整数であれば何でもよい。
また、それぞれの面が下を向く確率の比は自由に調整できるものとする。
面の数は最小でいくつだろううか。

336 :132人目の素数さん:2013/08/17(土) NY:AN:NY.AN .net
7面ではできない…と思うが…どうか

337 :132人目の素数さん:2013/08/19(月) NY:AN:NY.AN .net
8面でできた、1から順に
1/30, 1/10, 1/6, 1/5, 1/5, 1/6, 1/10, 1/30

338 :336:2013/08/19(月) NY:AN:NY.AN .net
>>337
1〜8でもできたのか……

339 :132人目の素数さん:2013/08/23(金) NY:AN:NY.AN .net
>>324

背理法による。
mもnも ax+by (x≧0, y≧0) の形で表わせたと仮定する。
m+nもそうだから、
 ab-a-b = ax+by (0≦x<b-1, 0≦y<a-1)
 ab = a(x+1) + b(y+1),
(a,b)=1 より
 x+1 ≡ 0 (mod b)、y+1 ≡ 0 (mod a)
 x+1 = kb、y+1 = La (k≧1, L≧1).
 ab = ab(k+L),
ab(≠0) で割って、
 1 = k+L ≧ 2,  (矛盾)
∴ m, n の一方は ax+by の形では表わせない。

340 :132人目の素数さん:2013/08/23(金) NY:AN:NY.AN .net
>>316

nについての帰納法による。

(1) n=1 ならば明らかに成立する。

(2) n-1 については命題が成り立つ、と仮定する。

・n番目の点Pnがいずれかの△の内部にあるとき
  → その△がPnにより3つの△に分割される。

・n番目の点Pnがいずれかの辺上にあるとき
  → その辺を共有する2つの△が、Pnにより4つの△に分割される。

・n番目の点が頂点と重なるとき
  → 命題を「n個の相異なる点により・・・・」と解するならば、この場合は生じない。

よってnについても成立する。

341 :132人目の素数さん:2013/08/24(土) NY:AN:NY.AN .net
nは正整数である。n×nのマス目があって、それぞれのマスに1,2…,n^2の数字が一つずつ記されている。
このとき、どのような数字の記し方についても、次の性質をもつ隣接したマスが存在することを示せ。
「隣接したマスの記されている数同士の差はnより小さい」

342 :132人目の素数さん:2013/08/24(土) NY:AN:NY.AN .net
>>339
m,nの一方がax+byの形で表せることの証明が必要なのでは

343 :132人目の素数さん:2013/08/25(日) NY:AN:NY.AN .net
>>335
8面でできることは連立方程式を解けば>>337のように出るんだろうけど、
7面で出来ないことの証明って簡単に出来るの?

344 :132人目の素数さん:2013/08/25(日) NY:AN:NY.AN .net
7面で1/5,1/5,1/5,1/5,1/5となるのは
1/5,1/5,1/5,1/5,a+b+cまたは
1/5,1/5,1/5,a+b,c+d。
1/5,1/5,1/5,1/5,a+b+cのとき
1/3<1/5+1/5なので1/3,1/3,1/3はできない。
1/5,1/5,1/5,a+b,c+dのとき
1/3,1/3,1/3にするには
1/5,1/5,1/5,2/15+1/15,2/15+1/15で
1/2,1/2はできない。

345 :132人目の素数さん:2013/08/25(日) NY:AN:NY.AN .net
(12,12,12,8,7,5,3,1)/60。
(12,12,11,9,8,4,3,1)/60。
(12,12,11,9,7,5,3,1)/60。
(12,12,11,8,7,5,4,1)/60。
(12,12,9,8,7,5,4,3)/60。
(6,6,6,4,4,2,1,1)/30。
(6,6,6,4,3,3,1,1)/30。
(6,6,6,4,3,2,2,1)/30。
(6,6,5,5,4,2,1,1)/30。
(6,6,5,5,3,3,1,1)/30。
(6,6,5,4,4,2,2,1)/30。
(6,6,5,4,3,3,2,1)/30。
(6,6,4,4,3,3,2,2)/30。

346 :132人目の素数さん:2013/08/26(月) NY:AN:NY.AN .net
>>341
147
582
936

347 :132人目の素数さん:2013/08/26(月) NY:AN:NY.AN .net
お前らの中にイケメンいない?
稼げるのかレポ頼むw
URL貼れないから
メーンズ ガーーデン
って検索して!
※正しいサイト名は英語です。

348 :132人目の素数さん:2013/08/26(月) NY:AN:NY.AN .net
nを正整数とする。
任意の2n-1個の整数があったとき、その中から和がnの倍数になるn個の整数が取りだせることを示せ。

349 :132人目の素数さん:2013/08/27(火) NY:AN:NY.AN .net
>>341
×「隣接したマスの記されている数同士の差はnより小さい」
○「隣接したマスの記されている数同士の差はn以上」
ではないか?

350 :132人目の素数さん:2013/08/31(土) NY:AN:NY.AN .net
test

351 :132人目の素数さん:2013/08/31(土) NY:AN:NY.AN .net
>>348

数学の部屋 → 『割り切れる?Part7』
 山梨県 Footmark さんからの問題です。高校生以上向き。
三重県からの解答を掲載。

352 :132人目の素数さん:2013/08/31(土) NY:AN:NY.AN .net
test

353 :132人目の素数さん:2013/08/31(土) NY:AN:NY.AN .net
4次元正多面体をカウントしる

354 :132人目の素数さん:2013/08/31(土) NY:AN:NY.AN .net
数学の挑戦!!!
「エンジニアなら、三分以内に解ける;建築家なら、三時間;医者なら、六時間;
会計士なら、三ヶ月; 弁護士なら、解けないかもしれない」という仮説があります。
皆さんはどのくらいの時間がかかりますか?
https://twitter.com/baiduime/status/373445009929805824/

問題の画像
http://pbs.twimg.com/media/BS6-QgTCIAA1pr-.jpg:large?.jpg



なぁ、お前らは正解分かる?何分で解いた?

355 :132人目の素数さん:2013/08/31(土) NY:AN:NY.AN .net
>>354
右から二列目の縦列だけに注目すれば答えは簡単だけど、
他の列は無視していいんだろうか?

356 :132人目の素数さん:2013/08/31(土) NY:AN:NY.AN .net
答えは任意の数、少なくとも91と答える奴はアホ

357 :132人目の素数さん:2013/08/31(土) NY:AN:NY.AN .net
>>354 の問題の画像

[2 3 4 15 12]
[3 4 5 28 20]
[4 5 6 45 30]
[5 6 7 66 42]
[6 7 8 ? 56]


 m-1, m, m+1, C[2m,2]=m(2m-1), m(m+1)

358 :132人目の素数さん:2013/08/31(土) NY:AN:NY.AN .net
何でわざわざ余分なのがつけてあるのだろうか。
OEISでも91の他はなかった。六角数がわかったくらい。

359 :132人目の素数さん:2013/08/31(土) NY:AN:NY.AN .net
「問題未定義、少数の強法則。」
と唱えるのに、数秒。
何秒かかるかは、滑舌しだい。

360 :132人目の素数さん:2013/09/01(日) 05:00:47.76 .net
>>351

2n−1個の整数の中に、余りが同じものがn個以上あれば、そこからn個を取り出すと和はnの倍数なので、命題は成立する。
よって、以下では、余りが同じものはn−1個以下とする。

nの因数についての帰納法による。

(1) nが素数のとき
2n−1個の整数をnで割った余りの順に並べ、x_1, x_2, ..., x_(2n-1) とする。

同じ余りがn個以上並ばないため、
 j-i ≧ n-1 ⇒ x_j - x_i はnで割リ切れない。

ここで、i=1,2,・・・・,n-1 に対して
 y_i = x_(n+i)- x_i ≠ 0 (mod n)
つまり、「非合同ペア」がn−1組できる。
 {x_1、x_(n+1)}
 {x_2、x_(n+2)}
  ・・・・・・・・
{x_(n-1)、x_(2n-1)}
各ペアから一方を選ぶやり方は
 {y_1、y_2、・・・・・、y_(n-1)}
の部分集合(φも含める)と対応しており 2^(n-1) とおりある。

361 :132人目の素数さん:2013/09/01(日) 05:07:42.87 .net
>>351

〔補題〕1≦k≦n-1 とする。
 {y_1、y_2、・・・・・、y_k} の部分集合(φも含める)について、要素の和をnで割ったときの余りを求めると、
(k+1) 種類以上ある。

(略証)
kについての帰納法による。
k=1 のときは φおよび{y_1} の2種があり、成立つ。
k-1 について成立つと仮定する。
 {y_1、 ・・・・、y_(k-1)} の部分集合について、要素の和をnで割った余りを求め、
 その集合を S_(k-1) とする。つまり、余りは #S_(k-1) 種類ある。
 #S_(k-1) = n ならば命題は成立する。
 #S_(k-1) < n ならば、上記の部分集合に y_k を加えたものを考える。
 nで割った余りは同数{#S_(k-1) 種類}だが、
 Sum{S_(k-1)~} = Sum{S_(k-1)} + y_k・#S_(k-1),
 y_k ≠ 0 (mod n)、 #S_(k-1) ≠ 0 (mod n)、nは素数だから、
 y_k・#S_(k-1) ≠ 0  (mod n)
 S_(k-1) と S_(k-1)~ は要素の数は同じだが、内容は異なる。
∴ S_(k-1)~ には S_(k-1) にない要素がある。
 S_k = S_(k-1) ∪ S_(k-1)~ ⊃ S_(k-1),
 #S_k ≧ #S_(k-1) + 1,     (略証終)

362 :132人目の素数さん:2013/09/01(日) 05:14:53.83 .net
>>351

∴ {y_1、y_2、・・・・・、y_(n-1)} の部分集合(φも含める)について、
 要素の和をnで割った余りを求めると、n種類すべてを含む。

 とくに -(x_1 + ・・・・・ + x_n) と同じ余りのものを含む。
∴ 和がnの倍数であるようなn個組の整数を取り出せる。

以上から、nが素数のとき、命題は成立する。

(2) nが合成数のとき。

nの素因数の一つをpとし、n=pmとする。

 素数の場合と同様にして、n−1個の整数の中から、和がpの倍数であるようなp個組の整数を除去する。
これは2m−1回繰り返すことができる。
その結果、和がpの倍数であるようなp個組が2m−1組できる。{最後にp-1個が残るが}

これらp個組の和をpで割った値を {z_1, z_2, ..., z_(2m-1)} とおく。
帰納法の仮定により、これら2m−1個の整数から、和がmの倍数であるようなm個を取り出せる。

よって、和がpmの倍数であるような、pm個を取り出すことも可能。

  (三重県 鳥居さんからの解答)

363 :132人目の素数さん:2013/09/01(日) 05:18:22.66 .net
>>351

(蛇足)
2n−2個の整数の中からn個を取り出してその和をnの倍数とすることは、一般には不可能である。
〔例〕{a,・・・・,a, a+1,・・・・,a+1} (各n-1個)

364 :132人目の素数さん:2013/09/05(木) 17:47:37.29 .net
任意の項数nの実数列には、単調増加または単調非増加な項数ceiling(√n)の部分数列があることを示せ。
ここで、ceiling(x)はx以上の整数の中で最小のものである。

365 :132人目の素数さん:2013/09/10(火) 18:10:59.61 .net
整数の数列 (a_1, a_2, …, a_n) で 1≦a_1≦2, 1≦a_2≦2a_1, …, 1≦a_(n-1)≦2a_n をみたすものの個数は、
整数N∈{0, 1, 2, …, 2^n-1} の 1, 2, 4, 8, …, 2^n-1 への分割の総数に等しいことを示せ。

例(n=2) #{(1, 1), (1, 2), (2, 1), (2, 2), (2, 3), (2, 4)} = #{21, 2, 111, 11, 1, φ}

366 :132人目の素数さん:2013/09/10(火) 19:01:04.73 .net
>>365 訂正 (2行目2個目の 2^n-1 → 2^(n-1) )

整数の数列 (a_1, a_2, …, a_n) で 1≦a_1≦2, 1≦a_2≦2a_1, …, 1≦a_(n-1)≦2a_n をみたすものの個数は、
整数N∈{0, 1, 2, …, 2^n-1} の 1, 2, 4, 8, …, 2^(n-1) への分割の総数に等しいことを示せ。

例(n=2) #{(1, 1), (1, 2), (2, 1), (2, 2), (2, 3), (2, 4)} = #{21, 2, 111, 11, 1, φ}

367 :132人目の素数さん:2013/09/16(月) 15:53:06.39 .net
>>335
一般にn_1,…n_mをどの2数も互いに素な2以上の自然数としたとき
これらの数について条件を満たすN=n_1+…+n_m-m+1面のサイコロを構成出来て
そのサイコロで出目がiとなる確率P(i)(0≦i≦N-1)は
P(i)=♯{(i_1,…,i_m)|0≦i_j≦n_j-1,i_1+…+i_m=i}/Π[k=1,m]n_k
で与えられることはわかった。
これが最小で一意だと思うが、それはうまく示せなかった。

368 :132人目の素数さん:2013/09/17(火) 09:54:07.50 .net
高校数学の質問スレPART356
http://uni.2ch.net/test/read.cgi/math/1378637354/283

283 名前:132人目の素数さん[sage] 投稿日:2013/09/13(金) 07:57:15.03
凸八角形Kがある。Kの頂点のうち適当な3点を結んで三角形を作ると
その面積はKの面積の ( ア )分の1以上にできる。

アに当てはまる最小の自然数はいくらか。

これはどのように考えればいいのでしょうか。
こんな問題初めてです。分野さえ分かりません。

369 :132人目の素数さん:2013/09/17(火) 10:17:23.36 .net
正八角形で1,3,6番目の頂点を結んだ場合を考えて、3 ( > 8-4√2 ) だろうか
なお間違っている可能性がかなりある。眠いし

370 :132人目の素数さん:2013/09/18(水) 08:12:17.52 .net
>>368以下と解釈。
任意の凸八角形Kに対して、Kの頂点のうち3点を結んで作られる三角形のうち
その面積が最大となるものは、少なくともKの面積の ( ア )分の1以上である。

>>369の例で3は可能。
八角形の頂点を一つ飛ばしで選んだ四角形と、
この四角形に外接し、かつ、八角形を内包する四角形との
入れ子で上限を見積もってみたけど、4は無理そうな感じだな。

371 :132人目の素数さん:2013/09/18(水) 11:17:35.51 .net
正八角形なら3は可能ってだけじゃないの?
4を考えてる意味もわからん。

372 :132人目の素数さん:2013/09/23(月) 12:00:00.03 .net
最大の三角形をABCとする。
Aを通りBCに平行な直線とBを通りACに平行な直線とCを通りABに平行な直線で
できる三角形をDEFとすると
K<DEF=4ABC。

373 :132人目の素数さん:2013/09/23(月) 12:57:54.63 .net
日能研の問題から応用問題。
半径の和が自然数Nであり、それぞれ自然数の半径a,b,cを持ち、
互いに重ならない3つの円を考える。
(1) N=20のとき、3つの円の面積の和の最小値と最大値を求めよ。
(2) 3つの円の面積の和の最大・最小をNを用いて表現せよ。
   なお、導出過程も記述せよ。

まあ、宮廷の大学入試レベルだな。

374 :132人目の素数さん:2013/09/23(月) 13:10:09.07 .net
どこが面白いのかさっぱりわからん

375 :132人目の素数さん:2013/09/23(月) 13:34:00.05 .net
書くのがかなりめんどい。というか俺はあきらめた。
受験生が「これなら分かるぞ」と思って解きはじめるが、
ごちゃごちゃしてきて投げてしまうパターン

376 :132人目の素数さん:2013/09/23(月) 13:52:18.12 .net
計算だけでやろうとするとゴチャゴチャするが
論理でやるとスッキリできる

377 :132人目の素数さん:2013/09/23(月) 16:05:09.99 .net
「論理でやる」とは?

378 :132人目の素数さん:2013/09/23(月) 18:02:18.00 .net
>>373
「自然数a,b,cがあり、a+b+c=Nとする。N=20の時、a^2+b^2+c^2の最大値と最小値を求めよ」
という問題と、本質的にどこが異なる?
なぜ、円が出てきて、互いに重ならないとかが出てくる?
問題に、記載されていない条件がなにか、抜けているのでは?

379 :132人目の素数さん:2013/09/23(月) 18:40:34.41 .net
>>378
Nの3に対する剰余で最大最小値が変わる
実数ならa=b=cで最大だが、自然数という制限が付いているからN=20の時には実現できない

380 :379:2013/09/23(月) 18:45:47.24 .net
すまん。何か勘違いしてた。
>>373とは別人です。
小学生向きの文章をそのまま改変したとか?

381 :373:2013/09/23(月) 19:49:20.29 .net
>>380
その通りです。日能研の文章をそのまま改変しただけ。
問題が面白くないor冗長なのはご愛嬌ってことで。
そういえば、日能研の文章には追加で、
最小となるa,b,cの組み合わせは何通りあるか、という問題もあったな。
どちらも、ガチャガチャ数え上げる計算しかできない小学生
にとっては、少し酷い問題だなと思ってしまった。

日能研の問題は数値をNに一般化すると、宮廷入試以上のレベルになる
ものが多い、という一例。
今回の回答の本質は>>379だね。最小は基本a=b=cだがNが3で
割り切れない場合の処理と、最大値の証明が大変。
正解は直感的に分かる人が多いが、それが正解だと示すのが面倒くさいので
後回しにされる問題だと思う。

382 :132人目の素数さん:2013/09/23(月) 20:39:54.06 .net
問題文が冗長はともかく、問題が面白くないのはご愛嬌とかスレタイ見ろよとしか言えないんだが。

383 :132人目の素数さん:2013/09/23(月) 22:30:18.97 .net
>>373

 a^2 + b^2 = (1/2)(a+b)^2 + (1/2)|a-b|^2,
より、a+b が一定ならば、|a-b| の大きい方が大きい。

(最小)
もし a-b≧2 ならば、
 (a-1)^2 + (b+1)^2 = (a^2 + b^2) -2(a-b-1)
          ≦ (a^2 +b^2) -2,
となるので、(a,b) は最小ではない。
∴ |a-b|≦1
同様にして |b-c|≦1、|c-a|≦1.
 {a,b,c} = {q, q+1} 
ただし、N = 3q + r, (0≦r<3)
 q が 3-r 個、q+1 が r 個.
 aa + bb + cc = (3-r)qq + r(q+1)^2
   = 3qq +r(2q+1)
   = ((3q+r)^2 -rr +3r)/3
   = (NN + r(3-r))/3,

(最大)
 {a,b,c} = {1,1,N-2} のとき
 aa + bb + cc = (N-2)^2 + 2,

384 :132人目の素数さん:2013/09/24(火) 22:44:30.99 .net
>>373

つまり

 最小値: π[ (NN+2)/3 ],

 最大値: π((N-2)^2 + 2),

385 :132人目の素数さん:2013/09/25(水) 23:15:26.07 .net
一見問題が面白いと思えなくても解き方が面白いなら許せるが、これはどうだろうか。

386 :132人目の素数さん:2013/09/26(木) 00:11:38.09 .net
問題、解法、結果、全てがつまらんな
問題のための問題としか思えん

387 :132人目の素数さん:2013/09/26(木) 22:00:00.28 .net
a(b)≧a(b+1)(1≦b<N)=>a(b)≦N−b。

J=煤i2^(a(b)))。

388 :132人目の素数さん:2013/09/29(日) 19:31:18.46 .net
>>386

ttp://modernfart.jp/2010/06/7564/

389 :132人目の素数さん:2013/10/12(土) 23:21:17.16 .net
凹型五等辺五角形は無限個存在する。

一つの凹型五等辺五角形をある次元の空間上の一点で表すとして、
ちかい形同士は近くに置いて、
できるだけ形の対称性が点列配置の対称性に対応するとすると
それら全ての集合は何次元のどんな形になるか?
(数学の問題としては記述が不正確だけどそこは許して)

390 :132人目の素数さん:2013/10/13(日) 03:29:31.73 .net
辺の長さを1に固定すると、自由度は隣り合う角度で二次元
最後の点を半径1の円の2交点のうちどちらにとるかで2通り
2つの正方形平面から最後の点まで辺が届かない、辺同士が交差、凸型、の
3領域を除くことになりそう

391 :132人目の素数さん:2013/10/13(日) 10:02:53.58 .net
有名だけど

次のようなゲームを考える
プレイヤーと司会者がおり、プレイヤーの前には3つのドアがあり、その奥には当たりが1つ、ハズレが2つ用意されている。
プレイヤーがドアを1つ選択する(この時点では開けない)。
司会者は正解のドアを把握しており(これについてプレイヤーは承知している)、
残された2つのうちハズレのドアを1つ開ける(2つともハズレの場合はランダム)。
司会者は「今なら選択を変更して構いませんよ?」とプレイヤーに問いかける。
さて、このときプレイヤーは最初の選択を変更するべきか、否か。

392 :132人目の素数さん:2013/10/13(日) 11:29:28.38 .net
ドアを選択した時点で、当たりであってもハズレであっても
変更するかしないかで等確率1/2で当たりハズレがあるので
選択を変更してもしなくても当たる確率は同じ。

393 :132人目の素数さん:2013/10/13(日) 11:38:49.14 .net
正解のドアを把握している司会者がハズレを1つ教えることがポイント

394 :132人目の素数さん:2013/10/13(日) 18:58:12.98 .net
>>392
ちょっと俺と賭けをしないか?

395 :132人目の素数さん:2013/10/13(日) 22:42:56.83 .net
0から999の整数を、三次元格子点(x,y,z)と次のルールで対応付ける
xは100の位の数字、yは10の位の数字、zは1の位の数字

問題:
0から999までの整数から三つの素数を選び、それに対応する三つの三次元格子点を結ぶと
正三角形を成したという。そのような3素数の選び方のうち、もっとも大きな正三角形を
成す組み合わせは何か?

396 : ◆2U88CDX3HuB9 :2013/10/14(月) 06:12:38.99 .net
ふむ…

397 :132人目の素数さん:2013/10/14(月) 10:10:44.53 .net
次の数列の□に当てはまる数はなんですか?
6 16 32 34 □

398 :132人目の素数さん:2013/10/14(月) 11:06:13.95 .net
881,991

399 :132人目の素数さん:2013/10/14(月) 12:59:42.51 .net
>>398 すべての正答を見つけただろうということは判る

400 :132人目の素数さん:2013/10/15(火) 00:40:11.62 .net
>>395

辺の長さって
 {113, 131, 311} なら 2√2
 {337, 373, 733} なら 4√2
 {199, 919, 991} なら 8√2
てな具合?

401 :132人目の素数さん:2013/10/15(火) 02:09:37.19 .net
そうですよ

402 :132人目の素数さん:2013/10/17(木) 02:54:05.34 .net
>>391
モンティ・ホール問題ね
さすがに有名すぎ

403 :132人目の素数さん:2013/11/27(水) 14:08:18.52 .net
Σ[n=1〜∞](n+m-1)Pm・(1-r)^n=m!/2r^m
を示せ
但しm∈N r∈Rで0<r<1

404 :132人目の素数さん:2013/11/29(金) 20:03:24.93 .net
>>403

与式を Q_m とおく。母関数は
 Σ[m=0,∞) Q_m/m!・s^m
 = Σ[m=0,∞) Σ[n=1,∞) P[n+m-1,m]/m! (1-r)^(n-1)・s^m
 = Σ[m=0,∞) Σ[n=1,∞) C[n+m-1,m] (1-r)^(n-1)・s^m
 = Σ[n '=0,∞) Σ[m=0,n '] C[n ',m] (1-r)^n '-m・s^m
 = Σ[n '=0,∞) (1-r+s)^n '
 = 1/(r-s)
 = (1/r)/{1-(s/r)}
 = (1/r)Σ[m=0,∞) (s/r)^m,

∴ Q_m = m!/r^(m+1),

405 :132人目の素数さん:2013/12/29(日) 03:11:04.44 .net
次の条件を満たす閉集合X[1],X[2],...と数列a[1],a[2],...は存在するか?
・各iについてa[i]は自然数でありX[i]はR^2内の正a[i]角形である
・ある有界集合Yがあって各iについてX[i]⊂Yとなる
・各iについてa[i+1]<2*a[i]
・各iについてX[i]⊂X[i+1]

406 :132人目の素数さん:2013/12/29(日) 10:28:59.66 .net
閉集合X[1],X[2],...は閉集合列X[1],X[2],...のこととして、さっぱりわからん
どこら辺がどう面白いのかが

407 :132人目の素数さん:2013/12/29(日) 20:36:59.30 .net
a[i+1]=a[i], X[i]=X[i+1] でいいだろ

408 :謹賀新年:2014/01/01(水) 01:42:27.70 .net
3つの皿に、それぞれいくつかの豆が入っている。
これらに対し、以下の1つの操作だけが許されている。

操作: 2つの皿を選びA,Bとする。
   AからBに、きっかりBの個数分だけ豆を移す。

  i.e. A,Bの豆をa個,b個(a≧b)としたとき、
   AからBにb個の豆を移して a-b個, 2b個とする。

3つの皿の初期状態がどのような個数であっても、
この操作を上手く繰り返すことにより、いずれかの皿を
空にすることができることを示せ。

409 :132人目の素数さん:2014/01/02(木) 06:11:35.70 .net
質問させてもらいます。

試行回数をn、的中率をp、回収率をk%とすると、
真の回収率=k × (p ± 2×平方根((1−p)×p/n) )/p

※1と2の真の回収率はそれぞれいくつになるのでしょうか?

※1 試行回数485   的中率5.8%     回収率181.3%

※2     485      11.5%       123.9%

410 :132人目の素数さん:2014/01/03(金) 11:01:08.09 .net
>>405 >>406
たとえば…
a[i]=2^i+1、半径1の円をC[0]として、任意の自然数iについて
C[i-1]に外接する正a[i]角形を周とする領域をX[i]、X[i]に外接する円をC[i]とすると、
C[i]の半径r[i]は,r[i]=Π{k=1,i}cos(π/(2^k+1))と表せる。
これでi→∞としてr[i]が有限値に収束するなら、これがその例になる。
r[i]は対数を取るとlog(cos(π/(2^i+1)))のΣとなるので、それを適当に評価すればいい。
面倒なので以下略

411 :132人目の素数さん:2014/01/03(金) 12:00:41.31 .net
答えじゃなくて、どこが面白いのかわからんだけなのだが

412 :132人目の素数さん:2014/01/03(金) 14:10:18.23 .net
Πが無理数であることの証明って出来ます?

413 :132人目の素数さん:2014/01/03(金) 15:09:37.38 .net
πが有理数であると仮定すると超越数であることと矛盾

414 :132人目の素数さん:2014/01/17(金) 23:00:17.08 .net
その各桁の数の立方の和に等しいような数が。ちょうど4個ある。
それらはいくつか。

古典的名著、コンスタンス・レイド『ゼロから無限へ』(芹沢正三訳、
講談社ブルーバックス、1971)より。

415 :132人目の素数さん:2014/01/18(土) 00:14:01.27 .net
意味が取れない????

416 :132人目の素数さん:2014/01/18(土) 00:34:16.34 .net
そのような数はせいぜい4桁なので虱潰しで

417 :132人目の素数さん:2014/01/18(土) 02:42:01.03 .net
>>414

1, 153, 370, 371, 407 (自然数を十進法で表わしたとき)

418 :132人目の素数さん:2014/01/18(土) 04:04:51.93 .net
スレチかもしれないですがスレ立てできなかったので貼らせていただきます
数学の課題です、お願いいたします

次の(i)(ii)を満たすDnを求めよ
(i)lim  Dn={(x,y)|x>0,y>0}
n→∞
(ii)lim ∬ (x-y)dxdy=2014
n→∞ Dn
ヒント
Dn{(x,y)|a <x<bn,c <y<dn}を予想して確かめる
      n    n
lim  a =0=lim c   lim   b =∞=lim  d
n→∞  n  n→∞ n n→∞  n  n→∞ n

419 :132人目の素数さん:2014/01/18(土) 04:13:19.71 .net
>>418
マルチポストはしない
既に質問スレがあるので個別の問題でスレ立てはしない
質問スレのテンプレを見て式を書き直せ
ここでの質問は取り下げて質問スレで親切な人を待て

420 :132人目の素数さん:2014/01/18(土) 16:50:30.58 .net
ろくに読みもしないで質問する奴って
よっぽど焦ってるんかな?

421 :132人目の素数さん:2014/01/18(土) 16:57:33.71 .net
続き

(4) その各桁の数の4乗の和に等しいような自然数が、ちょうど4個ある。
  それらはいくつか。

(5) その各桁の数の5乗の和に等しいような自然数が、ちょうど3個ある。
  それらはいくつか。

(6) その各桁の数のn乗の和に等しいような自然数がある。(n>5)
  それはいくつか。

422 :132人目の素数さん:2014/01/18(土) 17:04:52.39 .net
>>421

(4) 1, 1634, 8208, 9474

(5) 1, 4150, 4151

(6) 1  (n>5 または n=2)

かな?

423 :132人目の素数さん:2014/01/18(土) 17:13:28.88 .net
嘘問題。

424 :132人目の素数さん:2014/01/19(日) 00:00:00.38 .net
0,1,4150,4151,54748,92727,93084,194979。
0,1,548834。

425 :132人目の素数さん:2014/01/21(火) 00:00:00.19 .net
0,1,1741725,4210818,9800817,9926315,14459929。
0,1,24678050,24678051,88593477。
0,1,146511208,472335975,534494836,912985153。
0,1,4679307774。

426 :132人目の素数さん:2014/01/21(火) 11:40:37.94 .net
4^10+6^10+7^10+9^10+3^10+0^10+7^10+7^10+7^10+4^10=4679307774

427 :132人目の素数さん:2014/01/22(水) 00:00:00.41 .net
0,1,32164049650,32164049651,40028394225,42678290603
,44708635679,49388550606,82693916578,94204591914。
0,1。
0,1,564240140138。
0,1,28116440335967。
0,1。

428 :132人目の素数さん:2014/01/23(木) 01:38:33.94 .net
nn+98=x(75-n)、あるいはnn+98が75-nで割り切れる時のnを求める解法

429 :132人目の素数さん:2014/01/23(木) 07:08:02.28 .net
nn+98=-(75-n)(75+n)+5723

430 :132人目の素数さん:2014/01/26(日) 18:27:50.40 .net
直角三角形があって、その周りの長さが60インチ、
斜辺へ下ろした垂線の長さが12インチあるとき、
それぞれの辺の長さは?

431 :132人目の素数さん:2014/01/26(日) 18:38:08.83 .net
15, 20, 25

432 :132人目の素数さん:2014/02/16(日) 05:04:44.50 .net
この極限を求めよ
http://i.imgur.com/uttZYqn.png

433 :132人目の素数さん:2014/02/16(日) 05:27:15.70 .net
x^4-2x^3+x^2-2=0

434 :132人目の素数さん:2014/02/16(日) 09:53:23.77 .net
>>432
2

435 :132人目の素数さん:2014/02/16(日) 13:03:03.04 .net
…が含まれている数値をxと置いていいの?

436 :132人目の素数さん:2014/02/16(日) 15:45:08.31 .net
丁寧にやる時は再帰的に与えるだろうけどここで気張ることもあるまい

437 :132人目の素数さん:2014/02/16(日) 16:20:46.03 .net
…(√2+(√2+(√2+…
って外にも点々が続いてたら?

438 :132人目の素数さん:2014/02/16(日) 16:44:00.60 .net
そんな式を考えた奴が出てきたらそいつに確かめればよい

439 :132人目の素数さん:2014/02/16(日) 17:15:06.36 .net
漸化式でやろうとしたら
a(n+1)^2=2+a(n)で詰んだ
これ一般項出せるの?

440 :132人目の素数さん:2014/02/16(日) 17:23:30.45 .net
そもそもその数、n重の根号無しには表せんだろう

441 :132人目の素数さん:2014/02/16(日) 17:29:42.55 .net
一般項を知らなくても、初項を正の数とすれば2に収束することはわかる

442 :132人目の素数さん:2014/02/16(日) 18:22:32.76 .net
a(n)=√(2+√(a(n-1))

443 :132人目の素数さん:2014/02/16(日) 23:51:56.00 .net
丁寧に議論するなら
@漸化式から、有界と単調を言う
Aもし収束するならば、x^2=2+xを満たすxに収束する
ことを言えばよい

444 :132人目の素数さん:2014/02/17(月) 00:20:50.88 .net
やっぱり挟み撃ちか

445 :132人目の素数さん:2014/02/17(月) 02:49:33.00 .net
http://i.imgur.com/celz7Qf.jpg
なぜこうなる?

446 :132人目の素数さん:2014/02/17(月) 07:47:10.87 .net
お前の書く式の順番が意味分からんその理由から説明しろww

447 :132人目の素数さん:2014/02/17(月) 11:16:34.47 .net
>>445
左側のやり方で考えるなら、1/5が1より小さいことを考慮していないから間違えている。
右側のやり方についてはいったい何がわからんのかわからん。

448 :132人目の素数さん:2014/02/18(火) 02:48:03.06 .net
>>439
a_n=2cos(Θ_n) ,Θ_(n+1)=(Θ_n)/2 

449 :132人目の素数さん:2014/02/18(火) 08:05:42.80 .net
同じ式から左右で違う式展開をやって、
なぜ結果が異なるかっていう質問だったのか。

450 :132人目の素数さん:2014/02/21(金) 01:20:47.44 .net
x軸上の点(a,0)を中心とする半径r(r>0)の円が放物線y=x^2に接しているという。
aとrの関係を求む

451 :132人目の素数さん:2014/02/21(金) 03:14:55.58 .net
>>450
(放物線の接線の方程式と円の中心との距離)=r を、といたらいけそうだね

452 :132人目の素数さん:2014/02/21(金) 04:00:00.33 .net
16(a^2−r^2)^3+a^4−20a^2r^2−8r^4−r^2=0。

453 :132人目の素数さん:2014/02/21(金) 06:59:09.01 .net
>>451
いやいや^^;

454 :132人目の素数さん:2014/02/21(金) 17:53:12.86 .net
放物線y=x^2の(x,y)における接線は(0,-x^2)を通る

455 :132人目の素数さん:2014/02/26(水) 06:08:30.64 .net
2^a - 3^b = 1 をみたす自然数解の組 (a、b) をすべて求めよん。

456 :132人目の素数さん:2014/02/26(水) 16:08:28.61 .net
>>455
3^b≡1,3(mod 2^3)
よって(a,b)=(2,1)のみ

457 :132人目の素数さん:2014/02/26(水) 20:00:22.12 .net
2^a - 3^b = -1 をみたす自然数解の組 (a、b) をすべて求めよん。

458 :132人目の素数さん:2014/02/26(水) 22:08:42.57 .net
>>457
(log 3)/(log 2)の連分数展開より
(a,b)=(1,1),(3,2)以外に存在したとしても、人類の手には負えないものと思われる

459 :132人目の素数さん:2014/02/26(水) 22:32:27.97 .net
タオは使わんでもなんとかなる

460 :132人目の素数さん:2014/02/27(木) 07:45:50.44 .net
f(b)=3^b-1
f(b)=3*f(b-1)+2

461 :132人目の素数さん:2014/03/04(火) 20:19:21.37 .net
今年の一橋大学の数学第1問には感心した。
解答をまだ見ていない人、楽しめること請け合いまっせ。

a-b-8とb-c-8が素数となるような素数の組(a,b,c)をすべて求めよ。

462 :132人目の素数さん:2014/03/04(火) 21:03:44.22 .net
こういう整数問題を第1問に出されたら結構焦りそう

d=a-b-8,e=b-c-8とする。
また、pをある奇素数とする。

d=2のとき
 e=2のとき
  a,b=a-10,c=a-20は3で割った余りが異なる3つの数なので、
  いずれか1つは3の倍数。
  全て素数だから、この中で最小のc=3
  このときb=13,a=23となって条件を満たす。

 e=pのとき
  b-c=8+p(奇数)より、b,cの偶奇は異なる。
  b>cかつcは素数なのでc=2
  このとき a=20+p, b=10+p
  p≠3のとき、a,bのいずれか一方が6以上の3の倍数となるため不適。
  よってp=3であり、a=23,b=13

d=pのとき
 a-b=8+p(奇数)より、a,bの偶奇は異なる。
 a>bかつbは素数なのでb=2
 このときc=-6-e<0となって不適。

答 (a,b,c)=(23,13,3),(23,13,2)

463 :132人目の素数さん:2014/03/04(火) 21:30:12.88 .net
殆どの受験生は何が手がかりかも掴めずに途方に暮れただろうな。
理詰が好きな子は楽しんで解いたか。

464 :132人目の素数さん:2014/03/05(水) 01:45:49.26 .net
こういう手探りで解いていく問題大好き

465 :132人目の素数さん:2014/03/05(水) 06:58:08.90 .net
問題の発想はどこからだろ。
デザインとか符号理論?

466 :132人目の素数さん:2014/03/05(水) 21:14:40.12 .net
これa-bとb-cでも問題成り立つな
8という数に特に意味はなさそうだ

467 :132人目の素数さん:2014/03/05(水) 21:57:16.74 .net
>>439

・|a(1)| ≦ 2 のとき、
 a(n) = 2cos(α/(2^n)),
 ここに、cos(α/2) = a(1)/2,

・|a(1)| ≧ 2 のとき
 a(n) = 2cosh(β/(2^n)), (n>1)
 ここに、cosh(β/2) = |a(1)|/2,

468 :132人目の素数さん:2014/03/05(水) 22:49:53.76 .net
>>466
組の数を有限にするのには役にたっているかな。

469 :132人目の素数さん:2014/03/05(水) 22:55:27.27 .net
>>468
全て求め切ったと分らせるのには役に立っている、と言うほうがいいか。

470 :132人目の素数さん:2014/03/06(木) 23:48:47.65 .net
今年の東大の第四問がおもしろい
試験会場では解ききれなかったが、数Vのかなり深い所を聞いてきている

第四問
f(x)=(1-p)x+(1-x)(1-e^(-qx)) 0<p<1,p<q
(3)f(c)=c,0<c<1となるcが存在することを示せ

471 :132人目の素数さん:2014/03/06(木) 23:54:21.69 .net
良く練られた問題とは思うが、別に面白くも何ともない

472 :132人目の素数さん:2014/03/07(金) 00:35:47.17 .net
>>470
y=f(x)とy=xの交点が0<x<1の範囲にあることを示せばいい
おもしろいのはどの部分?

473 :132人目の素数さん:2014/03/07(金) 01:05:30.19 .net
それ単なる言い換えやん

474 :132人目の素数さん:2014/03/07(金) 07:48:41.83 .net
だ、か、ら、おもしろいのはどこだ、と聞いている

475 :TheLastManStudying:2014/03/07(金) 20:15:34.50 .net
最後のゆとり世代には、中間値定理が面白いのか。
来年は大変だな、気の毒に。

476 :132人目の素数さん:2014/03/08(土) 08:00:52.34 .net
今年の東大は第四問以外がつまらなさすぎたから、かえって第四問が面白く感じた

477 :132人目の素数さん:2014/03/08(土) 08:36:15.74 .net
今年の現役生は相当頭が悪い
中間値定理を知ってはいるが使える奴はほとんどいない
中高一貫の進学校でもこの現状

478 :132人目の素数さん:2014/03/08(土) 09:16:09.47 .net
読み流していたが、中間値定理が出るということは、
これは理系の試験だ!
東大理系二次で、こんな問題が出る時代になったのか。
少子化というのは、恐ろしいな。

479 :132人目の素数さん:2014/03/08(土) 09:22:36.81 .net
うるせえ!

480 :132人目の素数さん:2014/03/08(土) 09:34:08.95 .net
お前らゆとり貶して優越感浸るの好きだな

481 :132人目の素数さん:2014/03/08(土) 09:37:55.43 .net
で、どこがどう面白いの?

482 :132人目の素数さん:2014/03/08(土) 09:56:10.69 .net
改めて考えると全然面白い問題じゃなかった
ゆとり脳でした
ごめんなさい

483 :132人目の素数さん:2014/03/08(土) 10:05:08.13 .net
今年の2番の冒頭では
自然数(すなわち1以上の整数)
と記述してある。
おおっと思ったよ。「すなわち」だもんな。
0は自然数ですか、という連綿と続く遣り取りに業を煮やしたのかもしれない。
さて、これが「受験数学における自然数」の約束事に昇華するかどうか、興味深い。

484 :132人目の素数さん:2014/03/08(土) 10:06:39.14 .net
数学パズルとして面白い問題ではないってだけで
中間値の定理を面白いと感じたなら良いことだ

485 :132人目の素数さん:2014/03/08(土) 10:32:29.74 .net
ゆとりをなめんな

【サッカー/なでしこ】アルガルベカップ 日本、デンマークに1−0勝利! 岩渕の先制ゴールを守り今大会初白星[03/08]
http://hayabusa3.2ch.net/test/read.cgi/mnewsplus/1394208166/120

486 :132人目の素数さん:2014/03/08(土) 10:34:15.47 .net
120 名前:名無しさん@恐縮です[] 投稿日:2014/03/08(土) 01:29:52.55 ID:ivRovwLWI
>>103

我が国については、国際的に最上位レベルにある子どもの学力と対照的に、大人の理解度は下位に位置しており、極めて特徴的である。
我が国では、(略)、関心の低い大人の影響で子どもの関心が低下する(平成18年版 科学技術白書)

ユトリ世代  2位/25カ国
大人     22位/25カ国

ttp://www.mext.go.jp/b_menu/hakusho/html/hpaa200601/001/002/0401/1-2-54.gif


ユトリは成人力調査でも高い学力を持っているとわかりました

487 :132人目の素数さん:2014/03/08(土) 13:31:11.47 .net
いつまで「ゆとり」ネタに頼ってんだ?

488 :132人目の素数さん:2014/03/08(土) 15:15:00.55 .net
「ゆとり」は、もう終わるが、
少子化は、益々悪化してゆく。
学校を減らさなければ、
教育水準の低下は止められない。

489 :132人目の素数さん:2014/03/09(日) 01:16:53.62 .net
減らさず全部で少数精鋭やれば低下せんだろ

490 :132人目の素数さん:2014/03/09(日) 02:09:28.44 .net
もっと問題を

491 :132人目の素数さん:2014/03/09(日) 17:38:08.61 .net
次の極限を求めよ:
Σ(n=1~∞)√(n)*e^(-n)

492 :132人目の素数さん:2014/03/09(日) 20:00:31.35 .net
極限?

493 :132人目の素数さん:2014/03/09(日) 20:15:26.46 .net
ごめん
極限値といえばいいのかな?

494 :132人目の素数さん:2014/03/09(日) 20:17:45.63 .net
Σ(n=1~∞)

ここの意味がよくわからんけど?

495 :132人目の素数さん:2014/03/09(日) 20:46:52.60 .net
何度もごめんなさい
lim_[N→∞]Σ(n=1~N)√(n)*e^(-n) の値を求めて欲しい
ということです

496 :132人目の素数さん:2014/03/09(日) 22:28:55.76 .net
つまり無限級数でしょ?

497 :132人目の素数さん:2014/03/09(日) 22:30:34.00 .net
なんでそんな基本的な表記の事で突っかかってんのか、わからんわ

498 :132人目の素数さん:2014/03/10(月) 08:34:08.54 .net
いや、こういう表記あんまり見ないからさ
何か特殊な意味とか有るのかなと思って

499 :132人目の素数さん:2014/03/10(月) 11:19:17.87 .net
>>498
普通に見ますが

500 :132人目の素数さん:2014/03/10(月) 13:20:04.91 .net
ただの無限級数を極限とか書くからさ

501 :132人目の素数さん:2014/03/10(月) 13:21:26.25 .net
>>499
>【掲示板での数学記号の書き方例】
http://mathmathmath.dotera.net/
>●数列和・数列積:Σ_[k=1,n]a(k), Π_[k=1,n]a(k) (← "Σ"は「しぐま」,"Π"は「ぱい」で変換可.)
>●極限:lim_[x→∞]f(x) (← "∞"は「むげんだい」で変換可.)

少なくとも君の使った表記法を知らないからといって責められることではないね

502 :132人目の素数さん:2014/03/10(月) 13:21:50.89 .net
どうでもいいことグダグダいってんなよ

503 :132人目の素数さん:2014/03/10(月) 13:26:42.26 .net
なんでそんな基本的な表記の事で突っかかってんのか、わからんわ

"突っかかってんのか"、わからんわ

504 :132人目の素数さん:2014/03/10(月) 13:32:04.62 .net
Z会の天才問題集より
http://i.imgur.com/WonIBRr.jpg

505 :132人目の素数さん:2014/03/10(月) 15:53:46.95 .net
1.640205705728237058203865285315382948349514749938706030136522526234759357847017216022108728859728527 +
1.361230730112066360252141136119566081774341077796194978801633686001519877697193958458861004952824615 I

506 :132人目の素数さん:2014/03/10(月) 19:21:12.19 .net
数学板なのに>>491>>504にまともに答えられる人はいないの?

507 :132人目の素数さん:2014/03/10(月) 19:30:14.32 .net
>>506
あなたはそれら両方の出題者?

508 :132人目の素数さん:2014/03/10(月) 19:53:37.98 .net
表記の意味を尋ねても答えてくれないので
問題の解きようがありません

509 :132人目の素数さん:2014/03/10(月) 19:56:05.39 .net
>>501を見る限り、意味を理解しながら嫌がらせしてるようにしか見えないけど

510 :132人目の素数さん:2014/03/10(月) 20:25:49.59 .net
>>506
面白くないんじゃないの

511 :132人目の素数さん:2014/03/10(月) 20:48:59.89 .net
>>509
結局、無限級数をあらわしてるってことでいいの?

512 :132人目の素数さん:2014/03/10(月) 22:37:02.40 .net
491の出題者ですが、ただ無限級数の値を求めて欲しいということだけです

513 :132人目の素数さん:2014/03/11(火) 01:18:49.42 .net
504の答えは[5/2-{187^(1/3)}/2]^(1/3)≒- 0.710877で合ってる?

514 :132人目の素数さん:2014/03/11(火) 01:21:55.98 .net
アスペがうるさいスレ

515 :132人目の素数さん:2014/03/11(火) 01:25:56.20 .net
間違えた

[5/2-{189^(1/3)}/2]^(1/3)=[5/2-3/2*7^(1/3)]^(1/3)≒0.71751

516 :132人目の素数さん:2014/03/11(火) 01:38:48.42 .net
>>504
もっと問題を出せよ!

517 :132人目の素数さん:2014/03/11(火) 02:07:53.70 .net
>>515
ちがうよ

518 :132人目の素数さん:2014/03/11(火) 08:42:58.06 .net
a=cos(2π/7),b=cos(4π/7),c=cos(8π/7),α=a^(1/3),β=b^(1/3),γ=c^(1/3),
s=α+β+γ,t=αβ+βγ+γα,x=s^(1/3)とおく

a+b+c=-1/2,ab+bc+ca=-1/2,abc=1/8

s(ss-3t)=-2,2t(2tt-3s)=-5

4xxx-30xx+75x+32=0
s=x^(1/3)=[5/2-3/2*7^(1/3)]^(1/3)

519 :132人目の素数さん:2014/03/11(火) 08:44:37.02 .net
間違えた


a=cos(2π/7),b=cos(4π/7),c=cos(8π/7),α=a^(1/3),β=b^(1/3),γ=c^(1/3),
s=α+β+γ,t=αβ+βγ+γα,x=s^3とおく

a+b+c=-1/2,ab+bc+ca=-1/2,abc=1/8

s(ss-3t)=-2,2t(2tt-3s)=-5

4xxx-30xx+75x+32=0
s=x^(1/3)=[5/2-3/2*7^(1/3)]^(1/3)

520 :132人目の素数さん:2014/03/11(火) 09:02:04.34 .net
流石にこれは酷いのではないか。
これを放置するのが、最近の管理方針か?

521 :132人目の素数さん:2014/03/11(火) 09:03:11.38 .net
>>504
a=cos(2π/7), b=cos(4π/7), c=cos(8π/7)
a+b+c=-1/2
ab+bc+ca=-1/2
abc=1/8

522 :132人目の素数さん:2014/03/11(火) 14:35:44.48 .net
放置で充分だろ

523 :132人目の素数さん:2014/03/11(火) 17:09:19.60 .net
なんか問題でもあるのか?

524 :132人目の素数さん:2014/03/11(火) 18:41:34.25 .net
放置以外に有効策があるなら列挙してくれ
話はそれからだろう

525 :132人目の素数さん:2014/03/11(火) 19:58:57.97 .net
>>519 の補足

 s(ss-3t) +3u = a+b+c = -1/2,
 ttt -3stu +3uu = ab+bc+ca = -1/2,
 u = (abc)^(1/3) = 1/2,
より
 sssu - ttt = 1/4,
したがって
 sss = [5 - 3・7^(1/3)]/2,
 ttt = [4 - 3・7^(1/3)]/4,

526 :132人目の素数さん:2014/03/12(水) 17:48:54.13 .net
xy平面上において
(k-1,0)と(k,0)とを結ぶ経路(k=1,2,…,n)
(k-1,1)と(k,1)とを結ぶ経路(k=1,2,…,n)
(k,0)と(k,1)とを結ぶ経路(k=1,2,…,n)
を考える
各経路はそれぞれ1/2の確率で閉鎖される
このとき,(0,0)から出発して(0,1)へ行ける確率を求めよ

527 :132人目の素数さん:2014/03/12(水) 17:57:01.28 .net
経路が閉鎖されたらジャンプして行けばいいので、求める確率は1

528 :132人目の素数さん:2014/03/12(水) 18:31:53.05 .net
ひんがら目気色悪すぎこっち見んな死ね。ひんがら目気色悪すぎこっち見んな死ね。ひんがら目気色悪すぎこっち見んな死ね。
ひんがら目気色悪すぎこっち見んな死ね。ひんがら目気色悪すぎこっち見んな死ね。ひんがら目気色悪すぎこっち見んな死ね。
ひんがら目気色悪すぎこっち見んな死ね。ひんがら目気色悪すぎこっち見んな死ね。ひんがら目気色悪すぎこっち見んな死ね。
ひんがら目気色悪すぎこっち見んな死ね。ひんがら目気色悪すぎこっち見んな死ね。ひんがら目気色悪すぎこっち見んな死ね。
ひんがら目気色悪すぎこっち見んな死ね。ひんがら目気色悪すぎこっち見んな死ね。ひんがら目気色悪すぎこっち見んな死ね。
ひんがら目気色悪すぎこっち見んな死ね。ひんがら目気色悪すぎこっち見んな死ね。ひんがら目気色悪すぎこっち見んな死ね。

529 :132人目の素数さん:2014/03/12(水) 18:34:43.87 .net
527がドヤ顔で俺マジウケるレス返したわーと思っている確率なら1だろうな。

530 :132人目の素数さん:2014/03/12(水) 18:42:39.82 .net
ポエムにマジレスされて涙目wwwww

531 :132人目の素数さん:2014/03/12(水) 20:35:32.97 .net
lim_[N→∞]Σ(n=1~N)√(n)*e^(-n)
を求めよ

532 :132人目の素数さん:2014/03/12(水) 21:03:38.76 .net
ぞれじゃフォーマット厨は満足しないぞ

533 :132人目の素数さん:2014/03/12(水) 23:22:42.96 .net
>>531の値は求まるの?
∫[0,1]e^(-x^2)dxとかが出てきたんだけど

534 :132人目の素数さん:2014/03/13(木) 14:46:46.98 .net
それぐらいできるだろ

535 :132人目の素数さん:2014/03/13(木) 17:50:46.50 .net
>>534
e^(-x^2)の原始関数って初等関数で表せられないんでしょ?
それじゃ∫[0,1]e^(-x^2)dxの値は求まらないじゃん

536 :132人目の素数さん:2014/03/13(木) 18:26:33.34 .net
そこで数値解析ですよ

537 :132人目の素数さん:2014/03/13(木) 18:28:12.91 .net
求まる求まらないを語るときは、どの空間での話なのかをだな……

538 :132人目の素数さん:2014/03/13(木) 18:57:02.82 .net
結局のところ>>531の答えは何なの?

539 :132人目の素数さん:2014/03/13(木) 19:12:19.05 .net
ttp://www.wolframalpha.com/input/?i=sum+sqrt%28n%29*e%5E%28-n%29%2C+n%3D1+to+infinity

540 :132人目の素数さん:2014/03/13(木) 19:46:55.82 .net
これって"問題"になっているって言えるの?

541 :132人目の素数さん:2014/03/13(木) 22:34:41.22 .net
n人が100点満点のテストをしたとき、平均点が50点となった
このとき、ある1人の生徒の偏差値が得点以上となる確率を求めよ。

(※点数は連続(実数)で、それぞれの生徒について、0〜100点まで得点を取る確率は一様とする)

542 :132人目の素数さん:2014/03/14(金) 23:04:08.26 .net
ちょっと変えてみた。

4人が100点満点のテストをしたとき、平均点が50点となった
このとき、ある1人の生徒の偏差値が得点と等しくなる確率を求めよ。

(※点数はもちろん整数で、それぞれの生徒について、0〜100点まで得点を取る確率は一様とする)

543 :132人目の素数さん:2014/03/14(金) 23:23:54.95 .net
面白くもなんともない上に酷い問題だな

544 :132人目の素数さん:2014/03/15(土) 00:09:53.92 .net
>>531 >>539

0.7072407184868037907468779143806467104165083549257855 〜 √(1/2)

545 :132人目の素数さん:2014/03/15(土) 04:54:07.20 .net
√(1/2)〜0.70710678118

546 :132人目の素数さん:2014/03/15(土) 05:25:27.53 .net
n-n^2 が最大となるnの値を求めよ。

547 :132人目の素数さん:2014/03/15(土) 10:08:32.40 .net
作成途中か?

548 :132人目の素数さん:2014/03/15(土) 13:02:33.22 .net
>>546
n=0.5

549 :132人目の素数さん:2014/03/15(土) 13:30:05.68 .net
>>546
もはや中学レベルの問題

550 :132人目の素数さん:2014/03/15(土) 17:13:45.10 .net
>548
それどうやって解くんですか?

551 :132人目の素数さん:2014/03/15(土) 19:03:55.50 .net
n-n^2=-n(n-1)より
0と1の中間に軸があることは自明

552 :132人目の素数さん:2014/03/15(土) 20:48:36.24 .net
>>531

Σ[n=1, ∞) (√n)*e^(-n)
 〜∫[1/2, ∞) (√x)*exp(-x)dx
 = 0.7100910583 〜 √(1/2),

553 :132人目の素数さん:2014/03/15(土) 20:58:20.53 .net
ax^2 + bx + c = 0 の解を求めよ

554 :132人目の素数さん:2014/03/15(土) 21:57:31.33 .net
pを無理数とします。数列{a_n}をa_n=(p*nの小数部分)で定めます。
区間[0,1]に含まれる任意の区間[a,b]に対して、[a,b]∋a_nとなる自然数nが無限個存在することを示しなさい。

555 :132人目の素数さん:2014/03/15(土) 22:11:31.41 .net
そんなの[0,1]×[0,1]でトーラス作って葉層構造を考えれば自明じゃん

556 :132人目の素数さん:2014/03/15(土) 22:15:24.82 .net
自明ではないと思う
そこをきちんと表現して欲しかったんだけど

557 :132人目の素数さん:2014/03/16(日) 03:32:03.33 .net
>>553
条件不足

558 :132人目の素数さん:2014/03/16(日) 03:50:45.95 .net
√2+√3>π
を示せ

559 :132人目の素数さん:2014/03/16(日) 10:26:24.33 .net
>>553
x=(-b±√(b^2-4ac))/(2a)

560 :132人目の素数さん:2014/03/16(日) 10:33:30.44 .net
>>553
i)a!=0のとき→x=(-b±√(b^2-4ac))/(2a)
ii)a=0, b!=0のとき→x=-c/b
iii)a=b=0, c!=0のとき→不能
iv)a=b=c=0のとき→不定

561 :132人目の素数さん:2014/03/16(日) 10:41:40.53 .net
>>560
おっとb^2-4acの正負によっても場合分けが必要だった
まあ、めんどいからいいやw

562 :132人目の素数さん:2014/03/16(日) 12:50:51.75 .net
>>553
x=(-b±√(b^2-4ac))/(2a)
x=(2c)/(-b±√(b^2-4ac))
の使える方

563 :132人目の素数さん:2014/03/16(日) 13:48:45.88 .net
>>558
ttp://d.hatena.ne.jp/haruya12/20120314

564 :132人目の素数さん:2014/03/17(月) 00:26:53.17 .net
こっちにもいろいろある
√2+√3>πの証明
http://www2.ocn.ne.jp/~mizuryu/toukou2/toukou56.html

565 :132人目の素数さん:2014/03/18(火) 07:59:49.58 .net
何かもっとパズルっぽい問題無いの???
1,1,5,8で10を作れみたいなやつとか

566 :132人目の素数さん:2014/03/18(火) 08:35:50.36 .net
>>565
パズル
http://ikura.2ch.net/puzzle/

567 :132人目の素数さん:2014/03/18(火) 12:26:47.18 .net
4つの4を使って149を作ってください
ただし使っていい記号は以下の通り
•四則演算(+-×÷)
•括弧()
•小数点 (例 .4=0.4)
•根号(√ )
•階乗( ! )
•指数( ^ )

568 :132人目の素数さん:2014/03/18(火) 12:35:06.51 .net
どこがどう面白いのかさっぱり分からん

569 :132人目の素数さん:2014/03/18(火) 14:00:00.25 .net
√(√(√((√(4)/.4)^(4!))))+4!=149。

570 :132人目の素数さん:2014/03/20(木) 15:55:16.50 .net
a[n+1]=√{(1+a[n])/2}
b[n+1]=(1-a[n])b[n]
a[1]=0,b[1]=1

数列a[n],b[n]の一般項を求めよ

571 :132人目の素数さん:2014/03/20(木) 15:57:17.88 .net
つまらん

572 :132人目の素数さん:2014/03/21(金) 19:30:02.31 .net
a>0、b>0、c>0、d>0、abcd=1のとき、(1/a)+(1/b)+(1/c)+(1/d)+9/(a+b+c+d)≧25/4 を証明せよ

573 :132人目の素数さん:2014/03/21(金) 20:57:35.99 .net
>>572
分からない問題はここに書いてね388
http://ai.2ch.net/test/read.cgi/math/1391965739/872
高校数学の質問スレPART368
http://ai.2ch.net/test/read.cgi/math/1393860594/615
知恵袋
ttp://detail.chiebukuro.yahoo.co.jp/qa/question_detail/q1185526616

574 :132人目の素数さん:2014/03/22(土) 18:06:21.04 .net
数列a(n)は、以下の漸化式を満たす
a(n+3)=-a(n+2)+2a(n+1)+8a(n),
a(1)=a(2)=a(3)=1
このとき、a(n)のすべての項は平方数であることを証明せよ

575 :132人目の素数さん:2014/03/22(土) 20:00:07.95 .net
x^2+3x+4=0の2解 α,β
a(n)=2^(n+1)/7-α^n/7-β^n/7

576 :132人目の素数さん:2014/03/22(土) 20:40:27.12 .net
数列 b(n) を b(n+2)=b(n+1)-2b(n), b(1)=b(2)=1 とおくと、
すべての自然数 n で a(n)=(b(n))^2 となることを、帰納法で示せばいい。

577 :132人目の素数さん:2014/03/22(土) 22:57:51.74 .net
>>554の出題者だがこの問題についてはすでにワイルの均等分布定理という定理があるらしい
これを知っている人からすれば私の問題は全く面白くなかったでしょう
本当に申し訳ない

578 :132人目の素数さん:2014/03/22(土) 23:04:00.71 .net
>>572
これはググれば答えがヒットするが、かなりの難問

579 :132人目の素数さん:2014/03/22(土) 23:10:36.14 .net
自演乙

580 :132人目の素数さん:2014/03/23(日) 01:33:01.01 .net
>>572
ラグランジュ未定乗数法を使えば解けるが…
不等式の証明で解析を使うのはイケナイことだけど

581 :132人目の素数さん:2014/03/25(火) 19:22:28.36 .net
逝けない女だと他人は言ふけれど、イイじゃないの、(略証)ならば。

582 :132人目の素数さん:2014/03/25(火) 20:00:04.61 .net
>>574

xx-x+2=0 の2解
 γ = (1-i√7)/2,
 δ = (1+i√7)/2,
を使えば、
 b(n) = (δ^n - γ^n)/(i√7),
 2 = γδ,
 α = γγ,
 β = δδ,

583 :132人目の素数さん:2014/03/25(火) 20:41:29.08 .net
>>576

 b[n] = 2^{(n-1)/2}・U_n(1/√8),
ここに U_n は第2種チェビシェフ多項式。
 sin(nθ) = (sinθ)U_n(cosθ),

584 :132人目の素数さん:2014/03/30(日) 23:38:20.60 .net
nを正の整数とする。
3点(0,0)、(n√2,0)、(0,n√3)を頂点に持つ三角形の内部にある格子点の数をnで表せ。

585 :132人目の素数さん:2014/03/31(月) 05:02:01.95 .net
([n√2]+2)([n√3]+2)/2

586 :132人目の素数さん:2014/03/31(月) 05:10:40.80 .net
n=1でもう違った

587 :132人目の素数さん:2014/03/31(月) 23:45:33.87 .net
(0,0)、(√2,0)、(√2,√3)、(0,√3)を4頂点にもつ長方形自体、
内部の格子点は(1,1)の1点だけだから、対角線に関して対称じゃないんだよね。
これ解けるんだろうか?

588 :132人目の素数さん:2014/03/31(月) 23:57:46.81 .net
数論に出てくる名前のついているようなナントカ数の類が現れそうだな

589 :132人目の素数さん:2014/04/01(火) 06:03:07.64 .net
√3+√2>√6

590 :132人目の素数さん:2014/04/01(火) 06:33:50.07 .net
x>0, y>0
x√3+y√2<6n

591 :132人目の素数さん:2014/04/01(火) 08:32:04.24 .net
訂正 x√3+y√2<n√6

592 :132人目の素数さん:2014/04/02(水) 01:13:56.89 .net
>>584
近似式:{√(3/2)}n^2-{√(5/2)-0.008}n

593 :132人目の素数さん:2014/04/03(木) 13:55:43.07 .net
次の漸化式:a_1=p,a_(n+1)=-1+([1/a_n]+1)*a_n
で表される数列{a_n}は0に収束することを示せ
ただしpは無理数である.また実数xに対して[x]でxの整数部分を表すものとする

594 :132人目の素数さん:2014/04/03(木) 17:45:11.50 .net
-1+([1/(-1)]+1)(-1)=-1

595 :132人目の素数さん:2014/04/03(木) 18:06:45.47 .net
ただしpは無理数である.

596 :132人目の素数さん:2014/04/03(木) 19:05:01.35 .net
↑バカ

597 :132人目の素数さん:2014/04/03(木) 20:19:57.09 .net
負の数だと成り立たない気がするような
もし間違ってたら馬鹿と罵ってもらってかまわん

598 :132人目の素数さん:2014/04/03(木) 20:47:06.65 .net
p は -1 より大きい無理数とすれば成り立つようだ

599 :132人目の素数さん:2014/04/03(木) 22:18:20.77 .net
593の出題者です
はじめp=√2で成立したから多分無理数ならなんでも大丈夫なんだろなって思っていたのでpが正の場合しか考えていませんでした
"ただしpは無理数である"→"ただしpは正の無理数である"と訂正します
>>598さんの言うように正よりもっと範囲を拡張できるのかもしれませんが,まだ私自身検討中です

600 :132人目の素数さん:2014/04/04(金) 00:40:05.28 .net
極限lim(n→∞)tan{2^(1/n) nπ}を求めよ

601 :132人目の素数さん:2014/04/04(金) 00:47:09.76 .net
tan(πlog2)

602 :132人目の素数さん:2014/04/04(金) 18:48:58.18 .net
>>593,599
a_n > 1 のとき a_(n+1) = a_n - 1 だから
0 < a_n < 1 のときを考えればよい
x = 1/a_n とする
x の小数部分を {x} と書く ({x} = x - [x])
a_(n+1) = ([x] + 1)/x - 1 = (1 - {x})/x
1/a_(n+1) = x/(1 - {x}) = x + x{x}/(1 - {x})
x > 1, 0 < 1-{x} < 1 だから
1/a_(n+1) > x + {x} = [x] + 2{x}
1/a_(n+1) > [1/a_n] + 2{1/a_n}
つまり 1/a_(n+1) は 1/a_n より整数部分が大きいか、小数部分が2倍以上

1/a_n (n = 1,2,...) の小数部分は 0 になることはないので、
上から明らかに 1/a_n → ∞ (n→∞)

603 :132人目の素数さん:2014/04/04(金) 22:44:10.22 .net
>>600
え?

604 :abc:2014/04/11(金) 15:37:39.19 .net
突然ですが、平方根などの根の計算方法を発見しましたけど、どうしたらよいか分かりません。誰か教えて下さい。複雑な計算や難しい理論を必要とせず、微分積分も使いません。ネットで調べても同じものは無いようです。

605 :132人目の素数さん:2014/04/11(金) 16:30:10.68 .net
近所の3流以下の大学数学教授にメール、という考えがちらついた

606 :132人目の素数さん:2014/04/11(金) 17:00:47.48 .net
ポエムスレで発表すればいいよ

607 :132人目の素数さん:2014/04/11(金) 17:13:15.08 .net
適当な学会に入って論文投稿すればー
金払えば入会できるぞ

608 :132人目の素数さん:2014/04/11(金) 22:22:08.68 .net
近隣の中高教師の勉強会に相談してみては?
あなたの県名+数学+指導法+研究会 でggr,

609 :132人目の素数さん:2014/05/03(土) 19:46:07.96 ID:CJIvXJKsu
>>564

(2k-1)(2k+1) = 2kk +2kk -1 = 2kk(2k+1) -2kk(2k-1) -1,
を kk(2k-1)(2k+1) で割って
1/kk = 2/(2k-1) -2/(2k+1) -1/{kk(2k-1)(2k+1)},
k=1〜∞ でたして6倍すると
π^2 = 12 - Σ[k=1,∞) 6/{kk(2k-1)(2k+1)}
 ≦ 12 -2 -1/10 -2/105
 ≦ 12 -2 -1/10 -19/1000
 = 9.881

 √24 = 24/√(25-1) > 24/(5-0.1) > (24/5*5)*(5+0.1) = 0.96*5.1 = 4.896
(√2 +√3)^2 = 5 +√24 > 5 + 4.896 = 9.896

610 :132人目の素数さん:2014/05/03(土) 20:29:41.33 ID:CJIvXJKsu
>>563

0<θ<π/2 のとき、マクローリン展開から
 sinθ > θ - (1/6)θ^3,
 sinθ > θ - (1/6)θ^3,
 tanθ > θ + (1/3)θ^3,
辺々たすと
 2sinθ + tanθ > 3θ,
これは Snellius-Huygensの不等式として知られている。

この不等式で θ= π/4 - π/6 = π/12 として
 sinθ = sin(π/4 -π/6) = (√3 -1)/√8,
 tanθ = tan(π/4 -π/6) = 2-√3,
を使えば
 2(4-√2 -2√3 +√6) > π,

一方、
 √2 + √3 -2(4-√2 -2√3 +√6) = (√2 -1)^2・(2-√3)^2・(√3 -√2) > 0,
なので、(1)とあわせて
 √2 + √3 > π.

611 :132人目の素数さん:2014/04/20(日) 01:06:46.65 .net
幅→0の長方形

612 :132人目の素数さん:2014/04/20(日) 01:23:59.44 .net
>>611


613 :132人目の素数さん:2014/04/20(日) 03:52:09.18 .net
細い長方形なら格子点沢山覆えるだろう

614 :132人目の素数さん:2014/04/20(日) 09:38:28.99 .net
どんな形状であってもn+1個の格子点を含むように配置できる
と読むのであろう。

615 :132人目の素数さん:2014/04/20(日) 11:09:51.99 .net
配置は平行移動だけ? 回転も含まないと無理?

616 :132人目の素数さん:2014/04/20(日) 13:15:50.69 .net
ある点P,Qのx座標の差・y座標の差がいずれも整数であるとき、「PとQは同値である」ということにする。


問の平面図形をA、その面積をS(A)とする。
また、0 ≦ x < 1, 0 ≦ y < 1 に対して、
f(x,y) = [Aの内部にある点で、点 (x,y) と同値であるものの個数]
とする。
すると、S(A) =∫[0,1]∫[0,1] f(x,y) dxdy が成り立つ。

また、S(A) < n より、
f(x,y) ≧ n+1 を満たすような (x,y) が必ず存在する。
(「常にf(x,y) ≦ nが成り立つ」と仮定すると S(A) ≦ ∫[0,1]∫[0,1] n dxdy = n となり矛盾)

そのような (x,y) を一つ取り、点 (x,y) が原点にくるように図形を平行移動させると、
A内部には原点と同値な点 (すなわち格子点) がn+1個以上含まれることになる。

617 :609:2014/04/20(日) 13:40:56.94 .net
>>616
おお、定式化するとそういう風に証明するんでしょうね。

私が見た解説は、以下のようなものでした。
(1) 平面図形Aを、格子の升目の上に適当に置く。
(2) Aが含まれる1x1の升目を、バラバラに切り取る。
(3) 升目を全部重ねる。
(4) 升目の何処かの座標には、元Aの領域の重なりがn+1以上の箇所がある。
_(そうでなければ、Aの面積がn以下になるため)
(5) その座標が格子点になるように、平行移動すれば良い。

618 :132人目の素数さん:2014/04/21(月) 05:40:12.75 .net
コインを投げて表が出れば1点を加え、裏が出れば1点引く
ただし、0点の場合は引かない
初めの持ち点は0点とする
n回投げたとき、持ち点がk点となる確率を求めよ

答え
C[n,(n+k)/2](1/2)^n (n+kが偶数)
C[n,(n+k+1)/2](1/2)^n (n+kが奇数)

らしいんだが解き方分かる人いるかな

あと、単位円に内接する正n角形の頂点から3点選んでできる三角形の面積の期待値

619 :132人目の素数さん:2014/04/21(月) 07:29:49.30 .net
>>618
そもそも誤答じゃね?(n,k)=(2,0)とかどうよ
>ただし、0点の場合は引かない

|sin(2πu/n)+sin(2πv/n)-sin{2π(u+v)/n}|/2 (0<u,v),(u+v<n)の期待値あたりか?めんどくさ

620 :132人目の素数さん:2014/04/21(月) 21:10:23.10 .net
n=2,k=0だと表裏と裏裏で1/2
C[2,(2+0)/2](1/2)^2=1/2だが

621 :132人目の素数さん:2014/04/21(月) 22:08:16.13 .net
          __ノ)-'´ ̄ ̄`ー- 、_
        , '´  _. -‐'''"二ニニ=-`ヽ、
      /   /:::::; -‐''"        `ーノ
     /   /:::::/           \
     /    /::::::/          | | |  |
     |   |:::::/ /     |  | | | |  |
      |   |::/ / / |  | ||  | | ,ハ .| ,ハ|
      |   |/ / / /| ,ハノ| /|ノレ,ニ|ル' 
     |   |  | / / レ',二、レ′ ,ィイ|゙/   
.     |   \ ∠イ  ,イイ|    ,`-' |      
     |     l^,人|  ` `-'     ゝ  |        
      |      ` -'\       ー'  人           私は死なないわよ。
    |        /(l     __/  ヽ、            でも最近一寸太ったかしら。
     |       (:::::`‐-、__  |::::`、     ヒニニヽ、           Windows ver.10 で    
    |      / `‐-、::::::::::`‐-、::::\   /,ニニ、\            元の痩せた姿にしてよね。
   |      |::::::::::::::::::|` -、:::::::,ヘ ̄|'、  ヒニ二、 \              
.   |      /::::::::::::::::::|::::::::\/:::O`、::\   | '、   \
   |      /:::::::::::::::::::/:::::::::::::::::::::::::::::'、::::\ノ  ヽ、  |
  |      |:::::/:::::::::/:::::::::::::::::::::::::::::::::::'、',::::'、  /:\__/‐、
  |      |/:::::::::::/::::::::::::::::::::::::::::::::::O::| '、::| く::::::::::::: ̄|
   |     /_..-'´ ̄`ー-、:::::::::::::::::::::::::::::::::::|/:/`‐'::\;;;;;;;_|
   |    |/::::::::::::::::::::::\:::::::::::::::::::::::::::::|::/::::|::::/:::::::::::/
    |   /:::::::::::::::::::::::::::::::::|:::::::::::::::::::::O::|::|::::::|:::::::::::::::/

622 :618:2014/04/21(月) 22:32:22.29 .net
>>620
すまん間違えた

623 :132人目の素数さん:2014/04/22(火) 12:20:00.40 .net
数学的帰納法。

624 :132人目の素数さん:2014/04/22(火) 23:35:36.42 .net
教科書傍用の下みたいな練習問題
Σ[m=1→n]{Σ[l=1→m](Σ[k=1→l])}
これを式の意味を解釈して簡単に計算できないかな

たとえばΣ[k=1→n](k-1)(n-k)は
(k-1)(n-k)は1〜nの整数の中から3個取り出す方法のうち
2番目に大きい数字がkとなるような取り出しかただから
Σ[k=1→n](k-1)(n-k)=C[n,3]

625 : ◆BhMath2chk :2014/04/23(水) 00:00:00.94 .net

□■
■□■
□■□■
■□■□■
□■□■□■
■□■□■□■
□■□■□■□■

              ■
            ■  □
          ■  ■  □
        ■  ■  □  □
      ■  ■  ■  □  □
    ■  ■  ■  □  □  □
  ■  ■  ■  ■  □  □  □
■  ■  ■  ■  □  □  □  □

626 :132人目の素数さん:2014/05/04(日) 02:42:40.97 .net
test

627 :132人目の素数さん:2014/05/04(日) 02:53:40.74 .net
 >>563

0<θ<π/2 のとき、マクローリン展開から
 sinθ > θ - (1/6)θ^3,
 sinθ > θ - (1/6)θ^3,
 tanθ > θ + (1/3)θ^3,
辺々たすと
 2sinθ + tanθ > 3θ,
これは Snellius-Huygensの不等式として知られている。

この不等式で θ= π/4 - π/6 = π/12 として
 sinθ = sin(π/4 -π/6) = (√3 -1)/(2√2),
 tanθ = tan(π/4 -π/6) = 2-√3,
を使えば
 4{(√3 -1)/√2 +(2-√3)} > π,

√2 + √3 = 4{(√3 -1)/√2 +(2-√3)} + (√2 -1)^2・(2-√3)^2・(√3 -√2)
> 4{(√3 -1)/√2 +(2-√3)}
> π,

628 :132人目の素数さん:2014/05/04(日) 23:07:43.00 .net
a_0=0, a_1=1, a_(n+2)=a_(n+1)+a_n (n=0,1,2,…)とする。
(1)lim(n→∞)a_n/a_(n-1)を求めよ。
(2)(1)で求めた値をzとする。z^x(xは整数)はxが十分に大きいとき、ほぼ整数となる
ことを示せ。

629 :132人目の素数さん:2014/05/13(火) 00:05:49.55 .net
(1)(1+√5)/2
(2)b[n]=[z^n+1/2],z^n=b[n]+c[n]とする
このとき、-1/2≦c[n]<1/2…@
また、z^(n+2)=z^(n+1)+z^n,
c[2]=(-3+√5)/2,c[3]=-2+√5

2≦nでc[n]/c[n+1]=-zを数学的帰納法で示す
まずn=2のとき成り立つ
n=kで成り立つとする
z^(k+2)=z^(k+1)+z^k=b[k+1]+b[k]+c[k+1]+c[k]
c[k+1]+c[k]についてc[k]とc[k+1]は異符号で@より-1/2<c[k+1]+c[k]<1/2
よってc[k+2]=c[k+1]+c[k]
=(1-z)c[k+1]
c[k+1]/c[k+2]=1/(1-z)=-z
よってn=k+1で成り立つ

これらよりc[n]=c[2]/(-z)^(n-2)
となり示される

630 :132人目の素数さん:2014/05/23(金) 20:35:05.31 ID:giHHhsRIW
開き括弧'('と閉じ括弧')'のみからなる記号列
(ただし'('と')'が正しく対応付けられるもの)
があるとする。
この記号列のある部分に対し、
(X(Y)Z)→(XYZ)(XYZ)...(XYZ)
という置き換えを考える。
(X(Y)Z)の外側および内側の括弧はそれぞれ対応する括弧であるものとし、
X,Y,Zはそれぞれ任意の記号列(長さ0でもよい)とする。
(XYZ)(XYZ)...(XYZ)は、(XYZ)を任意個(0個でもよい)並べたものである。
このような置き換えを無限に繰り返し行うことは不可能であることを示せ。

631 :132人目の素数さん:2014/05/23(金) 20:37:21.90 .net
開き括弧'('と閉じ括弧')'のみからなる記号列
(ただし'('と')'が正しく対応付けられるもの)
があるとする。
この記号列のある部分に対し、
(X(Y)Z)→(XYZ)(XYZ)...(XYZ)
という置き換えを考える。
(X(Y)Z)の外側および内側の括弧はそれぞれ対応する括弧であるものとし、
X,Y,Zはそれぞれ任意の記号列(長さ0でもよい)とする。
(XYZ)(XYZ)...(XYZ)は、(XYZ)を任意個(0個でもよい)並べたものである。
このような置き換えを無限に繰り返し行うことは不可能であることを示せ。

632 :132人目の素数さん:2014/05/23(金) 20:54:04.69 .net
>>628 (2)
{(1+√5)/2}^n+{(1-√5)/2}^n は、{1,3,4,7,11,18,...}という整数値を取り、
(1-√5)/2=-0.618...なので、{(1-√5)/2}^nは、nが大きくなるとどんどん小さくなる
ことより、題意は示される。

633 :132人目の素数さん:2014/05/24(土) 17:35:50.66 .net
>>631
記号列を成す、全ての開き括弧“(”、及び、閉じ括弧“)”に対し、
次のルールで「深さ」という値プロパティを与えることとする
・“(”に対しては、「注目している記号より左側の全ての“(”の数」−「注目している記号より左側の全ての“)”の数」
・“)”に対しては、対応する“(”と同じ値

ところで、「置き換え」ルール:(X(Y)Z)→(XYZ)(XYZ)...(XYZ) を適用すると、Y内部の「深さ」は置き換え前に比べ、1減る。
元々の記号列は有限個からなるものなので、「最大の深さ」が存在するため、無限に行うことはできない。

634 :132人目の素数さん:2014/05/24(土) 18:50:08.43 .net
>>633
XとZの内部の深さは変わらないので、最大の深さは変わらない場合もある。
よってこれだけでは証明になっていないと思われるが。

635 :132人目の素数さん:2014/05/24(土) 21:02:16.76 .net
>>634
なるほど、空振りなら、無限回可能ということですね

では、この修正ではどうでしょう。

一番最初に、(X(Y)Z)型の部分列全てに対して、({X}(Y){Z})と、仮想括弧{}を補ってしまいます。
そして、仮想括弧を通常の括弧と同一視した状態で、「深さ」を考えることにします。すると、
>>(X(Y)Z)→(XYZ)(XYZ)...(XYZ)
という置き換えで、X,Y,Z の(修正版の)深さは、1ずつ減ることになります。

636 :132人目の素数さん:2014/05/24(土) 21:24:58.94 .net
>>635
(A(B)C(D)E)という部分列があるとき、
({A}(B){C(D)E})
({A(B)C}(D){E})
という2通りの仮想括弧の付け方がある。
上の説明だと、この場合の考え方が分からない。

637 :132人目の素数さん:2014/05/24(土) 21:37:32.88 .net
>>635
そもそも、例えば((()))に仮想括弧を付けて({(}(){)})とすると
括弧の対応関係がクロスした状態になってしまう。
これはマズいのでは。

638 :132人目の素数さん:2014/05/24(土) 22:04:00.24 .net
グラフ木と対応させればいいんじゃないかな

((()))(()())()なら


●●●
┃┣┛
●●  ●
┣┻━┛

みたいな

639 :132人目の素数さん:2014/05/24(土) 22:12:01.78 .net
なるほど、確かにその通りです。では、素直にいきます。これではどうでしょう。

記号列を食べる関数を考えます。
その関数は、>>633の方法の深さを全ての括弧についてチェックし、

深さ0の括弧のペアの数は、○個
深さ1の括弧のペアの数は、△個
...
と言うように、深さと、その括弧の数を返します。

そして、この返り値は、次の方法で比較可能で、
最大の深さの大小、同じなら、その深さの数の大小、
同じなら、次の深さの大小、同じならその深さの数の大小、...
で判断します。
この関数を使えば、置き換え前と、置き換え後を比べると、必ず小さくなっていきます。

640 :132人目の素数さん:2014/05/24(土) 22:19:30.43 .net
>>637
仮想括弧は、「置き換え」に対応させて考えていたものなので、
そのようなクロスは、題意から除かれています。

641 :132人目の素数さん:2014/05/24(土) 22:23:48.80 .net
>>637
失礼、よく読むと、そのようなクロスは、題意から除かれて「いない」んですね。

642 :132人目の素数さん:2014/05/24(土) 22:33:53.50 .net
>>639
X=Y=空列,Z="()"として
(()())→(())(())(())
という置き換えを行うと、最大の深さ2の括弧が2個から3個に増える。

>>640
ちょっとよく分からない。
((()))は置き換えの対象になる記号列だと思うんだけど。
X,Y,Zで表される記号列は、必ずしもその内部だけで
括弧の整合性が取れている必要は無い。

643 :132人目の素数さん:2014/05/24(土) 22:35:44.64 .net
>>641
そういうこと。

644 :132人目の素数さん:2014/05/25(日) 09:17:59.51 .net
>>642
「深さ」の他に、「並列度」とでも言うべき値も考えると、どうだろうか?
直接の「親」に当たる括弧の中に、自分と同じ「深さ」をもつ括弧がいくつかあるか、
それを「並列度」とします。
家系図なんかに例えると、「深さ」は「世代」に、「並列度」は「兄弟の数」に相当します。

>> X=Y=空列,Z="()"として
>> (()())→(())(())(())
>> という置き換えを行うと、最大の深さ2の括弧が2個から3個に増える。
深さ2,並列度2の括弧が二つあったものが、置き換え後は、
深さ2,並列度1の括弧が三つ(or任意個)と数えることになります。

645 :132人目の素数さん:2014/05/25(日) 16:53:53.63 .net
次の方程式が表す図形を座標平面に図示せよ。(ただしひとつの平面に書き込むこと)

x^2+y^2=1

x^2+y^2=4

y=±x (−4≦x≦−3,3≦x≦4)

y=0 (−4≦x≦−3,3≦x≦4)

x=0

646 :132人目の素数さん:2014/05/25(日) 17:16:00.99 .net
この類か
https://www.wolframalpha.com/input/?i=graph+hatune+miku+curve

647 :132人目の素数さん:2014/05/25(日) 19:06:43.66 .net
>>644
((())())→((()))((()))
深さ3並列度1が1個→深さ3並列度1が2個

648 :132人目の素数さん:2014/05/25(日) 20:21:30.19 .net
っつうかグラフ木から順序数に対応付けすればいいだけじゃん
そうすれば置き換えによって順序数は必ず減少するんだから

649 :132人目の素数さん:2014/05/25(日) 20:54:39.85 .net
具体的に

650 :132人目の素数さん:2014/05/25(日) 23:44:50.14 .net
>>631
なかなかいい問題やねw 出典が知りたいw

>>638のような木構造で考えると、「置き換え」による操作は以下の通り
・根と一致しない部分木を1つ指定する。ただし、部分木は2以上の高さを持つものとする
 (X(Y)Z)
・部分木に属する任意の頂点を1つ消去し、頂点の子以下の部分木をもとの頂点の親に接続する
 (X(Y)Z) --> (XYZ)
・変形した部分木を任意個複製する
 (XYZ) --> (XYZ)...(XYZ)

あとは木の複雑度を数に対応付けて、それらが単調減少することを示せばおk
数列 a_n の一般項を (外側から n 番目の括弧の組の数) で (その内側にある括弧の組の数)を割った値
とすれば、変形によりある p, q (p<q)について a_p が増えて a_q が減るので収束が示せる


注意すべきは、>>637のように X, Y, Z が外側と同じレベル以下の括弧を含む場合で
この場合は無限に増殖できることが示せる

例えば (())(()) で、X="", Y="", Z=")(()" とおくと
 (())(()) --> ()(()) が任意個
となって、1操作につき3個以上増やせば操作が無限に行える

651 :132人目の素数さん:2014/05/26(月) 00:02:26.41 .net
具体的に

652 :132人目の素数さん:2014/05/26(月) 00:09:55.58 .net
> 数列 a_n の一般項を (外側から n 番目の括弧の組の数) で (その内側にある括弧の組の数)を割った値
の部分は、分母を (その内側にある(n+1)番目の括弧の組の数) としても同じ結果になる

>>644の言葉を借りれば、全体について「並列度」を「子供の数の平均」と定義し直して0世代目から並べるイメージ

複雑度が上昇しないことは示せても、最終的に ()()...() の形に収束することは示せないので
厳密な証明には別のアプローチが必要になりそう

あと、具体例を無理に想像するとアッカーマン関数のように急激に増加するのでおすすめしない

653 :132人目の素数さん:2014/05/26(月) 00:41:37.68 .net
>629
> (X(Y)Z)の外側および内側の括弧はそれぞれ対応する括弧であるものとし

仮定から、この操作が可能ならばX,Y,Zにまたがる括弧の組は無い。
従ってこの操作で生成される(XYZ)内の括弧の組は(X(Y)Z)より一つ少なく、
かつ、(XYZ)をいくつ繋げても(XYZ)をまたぐ括弧の組は生まれない。
ゆえに(XYZ)の繰り返し回数が有限ならばこの操作は有限回で収束する。

654 :132人目の素数さん:2014/05/26(月) 01:10:19.38 .net
具体的に書かないのは反論させないためか。

655 :132人目の素数さん:2014/05/26(月) 01:12:04.91 .net
((()))=(X(Y)Z)
X=(
Y=
Z=)

656 :132人目の素数さん:2014/05/26(月) 07:20:59.41 .net
>>652=>>652です

>>653
> 仮定から、この操作が可能ならばX,Y,Zにまたがる括弧の組は無い。
「X,Y,Zにまたがる」を「X,Y,Zとその外側にまたがる」
と言いかえれば成り立ちますね。

確かに、外側同士が「対応する括弧」ですから
選んだ部分列の内側に低レベルの括弧は存在しないといえます。

657 :132人目の素数さん:2014/05/26(月) 12:57:51.49 .net
何度かトライ(631,633,637,642)しましたが、結局、

記号列を食べるある関数F[]を用意し、それが、
F[A(X(Y)Z)B] = F[A(XYZ)B] + α
F[A(XYZ)(XYZ)...(XYZ)B] = F[A(XYZ)B] +β
 ただし、常に、α>β≧0  (「任意個」のβが積み重なっても、αより小さい)
を満たせばよいということですよね。

そのようなF[]が存在するのは確かっぽいけど、具体的な中身は、当初の予想とは異なり面倒そうです。

658 :132人目の素数さん:2014/05/26(月) 13:05:54.05 .net
具体的に書こうとしないからはっきりしないが
(()()())()()
->
(()())(()())(()())()()
が反例じゃないか。

659 :132人目の素数さん:2014/05/26(月) 13:58:04.27 .net
>>658は確かに、>>652>>652の反例になってますね
(単純に平均値を取っただけでは、ゴミを巻き込むことで
評価関数が 3/3 --> 6/5 と増えてしまう)

出題者の>>657さんは解決に近づいているようなので
本職の数学者の降臨を待ちつつ様子見

660 :132人目の素数さん:2014/05/26(月) 14:13:52.78 .net
出題者は>>631だが

661 :132人目の素数さん:2014/05/26(月) 20:25:57.86 .net
>>650
一応自作なので出典は無し。
同じような問題はどこかにあるかも。

>>659
>>657は出題者ではないよ。

662 :132人目の素数さん:2014/05/26(月) 21:23:06.93 .net
これは「ヒドラゲーム」と同じ類の問題だな
下のリンク先にグラフ木と順序数との対応付けの方法が載ってる

http://math.andrej.com/2008/02/02/the-hydra-game/
http://ja.googology.wikia.com/wiki/%E3%83%92%E3%83%89%E3%83%A9%E3%82%B2%E3%83%BC%E3%83%A0

663 :132人目の素数さん:2014/05/29(木) 09:53:05.43 .net
>>465
> 面積nを超える平面図形は、内側(境界含む)に
> n+1個の格子点を含むように配置できることを示せ。
>
> ってのが面白かった。

ブリクフェルトの定理。有界がいる。

664 :132人目の素数さん:2014/06/05(木) 03:17:39.63 .net
四角形の4辺と2本の対角線の長さが全て奇数であるものは存在しないことを証明せよ。

665 :132人目の素数さん:2014/06/14(土) 11:48:01.30 .net
四角形の頂点をそれぞれabcdとしたとき、の辺の長さab, bcと対角線の長さacには
ab^2 + bc^2 = ac^2の関係があり ab,bcを奇数とすると、ab^2、bc^2はそれぞれ
奇数であるから、ac^2は偶数なっちゃうよ。
acを奇数とするとac^2は奇数だから。ab,bc,acがすべて奇数であるこたーないってこと?

666 :132人目の素数さん:2014/06/14(土) 12:23:54.72 .net
長方形でない四角形もあるだろ

667 :132人目の素数さん:2014/07/20(日) 17:44:57.73 .net
1/4の確率で当たりが出ると言われているクジがあります
100人がそれぞれ100回挑戦したところ、確率以上の頻度で当たりを引けたのは10人だけでした

実際の当たりが出る確率はどのくらいだと推定できますか?

情報はこれだけで実際の10000回の試行の具体的な結果は入手できないものとします

668 :132人目の素数さん:2014/07/20(日) 17:55:02.82 .net
出来ます。

669 :132人目の素数さん:2014/07/20(日) 19:52:36.97 .net
最尤法でいい?

670 :132人目の素数さん:2014/07/20(日) 19:55:27.77 .net
しかもなんだか正規分布で近似したい気分だけどいい?

671 :132人目の素数さん:2014/08/02(土) 22:20:47.73 .net
好きな検定を選べ

672 :132人目の素数さん:2014/08/22(金) 18:36:15.96 .net
age

673 :◆2VB8wsVUoo :2014/08/22(金) 18:38:11.38 .net
>>4
>4 名前:KingMathematician ◆LoZDre77j4i1 :2014/08/21(木) 19:27:24.63
> とりあえず, vector analysis から修得しようか.
>



痴漢・逮捕・痴漢・逮捕・痴漢・逮捕・痴漢・逮捕・痴漢・逮捕・痴漢・逮捕・痴漢・逮捕・痴漢・逮捕・痴漢・逮捕・痴漢・逮捕
逮捕・痴漢・逮捕・痴漢・逮捕・痴漢・逮捕・痴漢・逮捕・痴漢・逮捕・痴漢・逮捕・痴漢・逮捕・痴漢・逮捕・痴漢・逮捕・痴漢
痴漢・逮捕・痴漢・逮捕・痴漢・逮捕・痴漢・逮捕・痴漢・逮捕・痴漢・逮捕・痴漢・逮捕・痴漢・逮捕・痴漢・逮捕・痴漢・逮捕
逮捕・痴漢・逮捕・痴漢・逮捕・痴漢・逮捕・痴漢・逮捕・痴漢・逮捕・痴漢・逮捕・痴漢・逮捕・痴漢・逮捕・痴漢・逮捕・痴漢
痴漢・逮捕・痴漢・逮捕・痴漢・逮捕・痴漢・逮捕・痴漢・逮捕・痴漢・逮捕・痴漢・逮捕・痴漢・逮捕・痴漢・逮捕・痴漢・逮捕
逮捕・痴漢・逮捕・痴漢・逮捕・痴漢・逮捕・痴漢・逮捕・痴漢・逮捕・痴漢・逮捕・痴漢・逮捕・痴漢・逮捕・痴漢・逮捕・痴漢
痴漢・逮捕・痴漢・逮捕・痴漢・逮捕・痴漢・逮捕・痴漢・逮捕・痴漢・逮捕・痴漢・逮捕・痴漢・逮捕・痴漢・逮捕・痴漢・逮捕
逮捕・痴漢・逮捕・痴漢・逮捕・痴漢・逮捕・痴漢・逮捕・痴漢・逮捕・痴漢・逮捕・痴漢・逮捕・痴漢・逮捕・痴漢・逮捕・痴漢
痴漢・逮捕・痴漢・逮捕・痴漢・逮捕・痴漢・逮捕・痴漢・逮捕・痴漢・逮捕・痴漢・逮捕・痴漢・逮捕・痴漢・逮捕・痴漢・逮捕
逮捕・痴漢・逮捕・痴漢・逮捕・痴漢・逮捕・痴漢・逮捕・痴漢・逮捕・痴漢・逮捕・痴漢・逮捕・痴漢・逮捕・痴漢・逮捕・痴漢
痴漢・逮捕・痴漢・逮捕・痴漢・逮捕・痴漢・逮捕・痴漢・逮捕・痴漢・逮捕・痴漢・逮捕・痴漢・逮捕・痴漢・逮捕・痴漢・逮捕
逮捕・痴漢・逮捕・痴漢・逮捕・痴漢・逮捕・痴漢・逮捕・痴漢・逮捕・痴漢・逮捕・痴漢・逮捕・痴漢・逮捕・痴漢・逮捕・痴漢
痴漢・逮捕・痴漢・逮捕・痴漢・逮捕・痴漢・逮捕・痴漢・逮捕・痴漢・逮捕・痴漢・逮捕・痴漢・逮捕・痴漢・逮捕・痴漢・逮捕
逮捕・痴漢・逮捕・痴漢・逮捕・痴漢・逮捕・痴漢・逮捕・痴漢・逮捕・痴漢・逮捕・痴漢・逮捕・痴漢・逮捕・痴漢・逮捕・痴漢

674 :132人目の素数さん:2014/08/29(金) 01:38:17.28 .net
∫[1、∞] x^(-x) = Σ[n=1 to ∞] n^(-n) を証明せよ。


( ゚∀゚) プウ
ノヽノ) =3'A`)ノ ヒャー
  くく へヘノ

675 :132人目の素数さん:2014/09/10(水) 14:34:02.33 .net
自然数aに対し[a]をaの桁数とする。
つまり[100]=3
[[10の1000乗]]=[1001]=4

ここで非常に大きい自然数aに関して考える。
aに[]を幾重にも重ねた結果はじめて1になった時、[]を重ねた数を{a}とする。
つまり
{10の10乗}=2     [[10の10乗]]=1より
{10の1000乗}=3    [[[10の1000乗]]]=1より

では{グラハム数}の値は?

676 :132人目の素数さん:2014/09/10(水) 15:07:55.48 .net
>>675
その中括弧には意味があるのかね、少尉!

677 :132人目の素数さん:2014/09/10(水) 15:51:37.78 .net
だいぶ大雑把に見積もると、「桁数を取る」ってlogを一回かけるようなもんだから
[a↑↑b] ≒ a↑↑(b-1)
{a↑↑b} ≒ b
(ただし↑↑はテトレーション)
くらいか。この程度だとグラハム数にかけても大して小さくならなそうだな

678 :132人目の素数さん:2014/09/10(水) 17:50:30.90 .net
グラハム数って宇宙の全ての物質をインクに変えても表記出来ないんですよね。
では宇宙の全ての物質をインクに変えたらどのぐらいの値が表記できるのでしょうか?
またそれはグラハム数に比べどのくらい小さい値なのかを教えてください。

679 :132人目の素数さん:2014/09/10(水) 18:27:00.76 .net
宇宙全体の素粒子の数〜10^80
つまり、そろばんを二つ並べれば表現できる

680 :132人目の素数さん:2014/09/10(水) 20:58:13.34 .net
円の内部に点Pをとり、円をPを通る4本の直線(45°で交わる)で8つの領域に分ける。
1個おきに選んで2組に分けるとき、どちらも面積の和が等しいことを示せ。

681 :132人目の素数さん:2014/09/11(木) 21:52:08.71 .net
>>680
妄想も大概にせいよ

682 :132人目の素数さん:2014/09/13(土) 18:12:00.12 .net
コインを今からn(100以上の自然数)回投げる。
その時、表が出た回数が0.6nを超える、もしくは裏が出た回数が0.6nを超える確率f(n)を求めよ。

683 :132人目の素数さん:2014/09/13(土) 20:09:00.97 .net
Σ[k=[0.6n]+1,n]C[n,k](1/2)^n

684 :132人目の素数さん:2014/09/14(日) 10:38:24.78 .net
1 だろ。
必ずどっちかは、0.6 n を超える。

685 :132人目の素数さん:2014/09/14(日) 12:48:04.52 .net
これはひどい

686 :132人目の素数さん:2014/09/14(日) 17:08:51.80 .net
それしかないんか

687 :132人目の素数さん:2014/09/14(日) 22:07:28.97 .net
>>684
?????

688 :132人目の素数さん:2014/09/14(日) 22:54:21.83 .net
すみません>>683の数式がわからない高校生なのですが、
nが100の場合と1000の場合のそれぞれのf(n)の数値を教えて下さい

689 :132人目の素数さん:2014/09/14(日) 23:25:22.23 .net
表と裏が0.5n回ずつ出るケースでは、表裏どちらも0.6nを超えてない。
>>684は大間違い。

690 :132人目の素数さん:2014/09/15(月) 04:04:03.51 .net
>>683 訂正
Σ[k=ceil(0.6n),n]C[n,k](1/2)^n

691 :132人目の素数さん:2014/09/24(水) 11:40:39.01 .net
円周率が超越数である事を示せ

692 :132人目の素数さん:2014/09/24(水) 13:46:16.54 .net
ググって読め

693 :132人目の素数さん:2014/09/24(水) 14:28:52.57 .net
πが超越数でないと仮定する。
しかしこれは「Wikipedia 超越数」に書いてあることに矛盾する。
従ってπは超越数である。

_人人人人人_
> 証明終了 <
 ̄Y^Y^Y^Y^Y ̄

694 :132人目の素数さん:2014/09/24(水) 14:53:08.08 .net
>>693
個人的にフィールズ賞をあげたい

695 :132人目の素数さん:2014/09/25(木) 12:41:37.93 .net
フールズ賞

696 :132人目の素数さん:2014/09/25(木) 18:16:43.81 .net
自然数nについて、2から数えてn個目の素数をf(n)とする。
例:f(3)=5
ここで、lim n→∞ f(n+1)-f(n)を求め、証明せよ。

697 :132人目の素数さん:2014/09/25(木) 18:20:41.48 .net
仮に問題を解く知識がなかったとしても、文章からポエムなことがすぐわかるね

698 :132人目の素数さん:2014/09/25(木) 18:53:04.64 .net
すべてはまぼろし

運営乙

699 :132人目の素数さん:2014/09/25(木) 21:24:18.53 .net
差が7000万以下の素数のペアなら無限に存在する事が証明されたそうだから
素数定理と合わせて答えが得られる

700 :132人目の素数さん:2014/09/25(木) 21:49:47.09 .net
(´・∀・`)ヘー

701 :132人目の素数さん:2014/09/25(木) 22:44:11.32 .net
>>699
ま、まじで!?
それって凄いことだよね

702 :132人目の素数さん:2014/09/25(木) 23:06:38.25 .net
>>696
>>699により、liminf_[n→∞](f(n+1)−f(n))≦7000万
一方で、
ttp://en.wikipedia.org/wiki/Cram%C3%A9r%27s_conjecture
により、limsup[n→∞](f(n+1)−f(n))/(log f(n))=+∞
特に limsup[n→∞](f(n+1)−f(n))=+∞

以上より、lim_[n→∞](f(n+1)−f(n)) は存在しない

703 :132人目の素数さん:2014/09/26(金) 02:21:49.29 .net
おみごと

704 :132人目の素数さん:2014/09/26(金) 11:54:08.94 .net
「sup」の読み方って「スープ」でいいの?

705 :132人目の素数さん:2014/09/26(金) 12:04:13.01 .net
死ね

706 :132人目の素数さん:2014/09/26(金) 12:36:33.67 .net
What can't Japanese do?
Who can do something?
Who should die?
Why should someone die?

707 :132人目の素数さん:2014/09/26(金) 12:54:33.29 .net
>>704
上付きならsuperscript, 上限ならsupremumだからいいんじゃない

708 :132人目の素数さん:2014/09/26(金) 14:16:35.68 .net
>>706
数学板だからってその英語力はないだろう

709 :132人目の素数さん:2014/09/26(金) 16:37:16.80 .net
>>708
間違っていないと思うけれども

710 :132人目の素数さん:2014/09/26(金) 21:42:06.00 .net
知りたい事があります。
f(1)=1
f(2)=2の2乗
f(3)=3の3の3乗乗
f(4)=4の4の4の4乗乗乗

こういう関数って書き方あるんですか?

711 :132人目の素数さん:2014/09/26(金) 21:49:13.76 .net
http://i.imgur.com/UUqpdzA.png

712 :132人目の素数さん:2014/09/26(金) 21:49:53.60 .net
>>710
クヌースの矢印 でぐぐれ

713 :132人目の素数さん:2014/09/26(金) 21:50:43.21 .net
ビットマップ描きにしては上手いじゃん

714 :132人目の素数さん:2014/09/28(日) 13:13:55.47 .net
5000年ごとぴったりの周期で噴火する火山が50個ある。
これから一番速く噴火する火山はあと何年で噴火する?
各火山の噴火タイミングと観測者のいる時刻は全て独立と仮定する。

715 :132人目の素数さん:2014/09/28(日) 14:04:22.85 .net
Σ[k=1,5000]k*(4999/5000)^(k-1)*Σ[i=1,5000]C[5000,i](1/5000)^i*(4999/5000)^(5000-i)

716 :132人目の素数さん:2014/09/28(日) 14:09:40.80 .net
Σ[k=1,5000]k*(4999/5000)^(50(k-1))*Σ[i=1,50]C[50,i](1/5000)^i*(4999/5000)^(50-i)

717 :132人目の素数さん:2014/09/28(日) 16:46:27.80 .net
すごい。なんかちょっと意味分かんないけど、すごいありがとうございます

718 :132人目の素数さん:2014/09/30(火) 01:12:59.02 .net
夏の夜の
太鼓囃子か
本降りの
トタン屋根打つ
けたたましさよ

719 :132人目の素数さん:2014/10/03(金) 00:52:03.45 .net
お願いします。
10000円を5%と6%の定期にあずけて受け取った利息が575円
この場合10000円をどのような割合で預けたかわかりますか?
お願いします。

720 :132人目の素数さん:2014/10/03(金) 02:12:11.77 .net
マルチうざい

721 :132人目の素数さん:2014/10/11(土) 07:23:34.22 .net
重さ1の球を三角錐状に3n+1段積んだとき、最下段の中心の球にかかる重さはいくらか。

722 :132人目の素数さん:2014/10/13(月) 17:25:24.88 .net
抜いても問題ないから0

723 :132人目の素数さん:2014/10/13(月) 19:36:48.32 .net
>>722
想定外の回答だったので条件を加える。
各々の球は、その下段にある3つの球に均等に力を及ぼす。
求めるものは、上の3つの球から加わる力の合力とする。

724 :132人目の素数さん:2014/10/13(月) 21:50:13.31 .net
上に3つ球がない場合もある

725 :132人目の素数さん:2014/10/13(月) 23:58:57.08 .net
>>724
n=0の場合を例外として除けば、最下段の中心の球には必ず3個の球が上に接してる。

726 :132人目の素数さん:2014/10/22(水) 15:53:28.85 .net
逆三角錐?

727 :132人目の素数さん:2014/10/22(水) 16:50:13.21 .net
中心に位置する球のみではなく一般的な場合を考えてしまった

728 :謹賀新年:2015/01/06(火) 23:46:57.78 ID:Cy8xuL1q.net
数列a(n)を以下で定める。

a(0)=0
a(1)=1
a(n+1)は、次の2条件を満たす最小の自然数とする。
・ a(n)より大
・ {0,1,,,n}からどのようにi,j(i<j)を選んでも、
 a(i),a(j),a(n+1)は等差数列にはならない。


[1] a(2),a(4),a(8)はいくつか。
[2] a(1000)はいくつか。

729 :132人目の素数さん:2015/01/07(水) 23:38:15.33 ID:RnUWTRzD.net
1000=2^3*(1+2^2*(1+2*(1+2*(1+2*(1+2*(1+0))))))

730 :132人目の素数さん:2015/01/10(土) 23:38:55.08 ID:XQc9ebMD.net
>>728
a[2]=3, a[4]=9, a[8]=27
a[1000]=29484

731 :132人目の素数さん:2015/01/11(日) 02:30:38.05 ID:sMhzxOIW.net
a(2)〜a(n-1)の値を出すことなしにa(n)を直接求めるにはどうすればいい?

732 :132人目の素数さん:2015/01/30(金) 12:45:53.43 ID:g9X/7gWf.net
https://www.toshin.com/sp/concours/
積分区間に∞が出てくるから、高校の範囲を超えてない?

('A` ) ブリブリッ
ノヽノ) =3'A`)ノ ギャ‐ッ
  くく へヘノ

733 :132人目の素数さん:2015/01/30(金) 20:50:19.92 ID:LFwsDtx3.net
高偏差値の生徒を集めたコースでは、
生徒が退屈して勉強を投げ出さないように、
たまに受験の出題範囲を越える問題も必要。
過保護な気もするが、サービス業だからね。

734 :132人目の素数さん:2015/02/24(火) 00:39:09.81 ID:qhrgXP8L.net
nHrを重複組合せの数とするとき、Σ[k=0 to r] nHk を簡単にせよ。

735 :132人目の素数さん:2015/02/28(土) 20:11:08.53 ID:VJZV6J0I.net
3以上の自然数nに対し、[n]をn-n未満の最大の素数と定める。
lim n→∞ [n]は発散か収縮か?そしてそれを証明せよ。

736 :132人目の素数さん:2015/02/28(土) 20:16:07.99 ID:dv/joUsD.net
>>735
収束
何故ならn-nは0だから

737 :132人目の素数さん:2015/02/28(土) 20:52:49.91 ID:VJZV6J0I.net
n-(n未満の最大の素数)です!

738 :132人目の素数さん:2015/02/28(土) 20:56:14.67 ID:D9aq9ikO.net
構文解釈はエスパー4級くらいだな
どこがどう面白いのかはエスパー不能

739 :132人目の素数さん:2015/02/28(土) 21:15:11.03 ID:zNiNlMKA.net
[奇素数+1]=1
[奇素数+2]=2
[奇素数+3]=3or1
[奇素数+4]=4or2
...
素数は無限にあり、このような増加と1への急落を無限に繰り返すので、収縮も収束もしない。発散。

740 :132人目の素数さん:2015/03/01(日) 18:17:51.24 ID:zELQ+vft.net
>>734
(1+r)/n((n+r)C(r+1))

741 :132人目の素数さん:2015/03/05(木) 00:16:52.34 ID:RwdUyX8p.net
>>739
双子素数予想は、解かれたんだっけ?

742 :132人目の素数さん:2015/03/05(木) 08:03:48.63 ID:e6CPYyVp.net
大川研究室っていう数ヲタの問題サイトって、どっかに移転したの?

743 :132人目の素数さん:2015/03/06(金) 00:13:31.73 ID:o/UgraHB.net
Σ[k=0 to n] (-1)^k {Σ[m=0 to k] C[2n+1, 2m+1]・C[n-m, n-k] } (sinθ)^{2k+1} を簡単にせよ。
ただし、C[n, r] は二項係数を表す。

744 :132人目の素数さん:2015/03/15(日) 15:54:44.22 ID:JATPX9sE.net
lim[n→∞] C[2n, n]/(4^n) の値を求めよん。

745 :132人目の素数さん:2015/03/16(月) 00:32:55.90 ID:gTNQV16n.net
f(x)を実数係数の多項式とする。
(1) f(x-1)f(x+1) = f((x^2)/2) をみたすf(x)を求めよ。
(2) {f(x)}^2 = f(x^2) をみたすf(x)を求めよ。
(3) f(x-1)f(x) = f(x^2) をみたすf(x)を求めよ。
(4) x・f(x-1) = (x-8)・f(x) をみたすf(x)を求めよ。

746 :132人目の素数さん:2015/03/16(月) 05:52:03.85 ID:gTNQV16n.net
x^4 + x^3 - 1 = 0 の4解を a、b、c、dとする。
ab、ac、ad、bc、bd、cd を解にもつ6次方程式を作れ。

747 :132人目の素数さん:2015/03/17(火) 18:08:39.22 ID:jPmGUsBF.net
>>746
これの解説をお願いします。

748 :132人目の素数さん:2015/03/20(金) 00:23:42.49 ID:4Uq6RMP/.net
>>745
すべて求めよと言っていないので、(1)-(4)はいずれも
f(x)=0
でOK

749 :132人目の素数さん:2015/03/20(金) 00:54:13.29 ID:X+uuAe1T.net
かなしいですね

750 :132人目の素数さん:2015/03/20(金) 01:03:56.82 ID:++NwwdBp.net
単行式なのですがそれは

751 :132人目の素数さん:2015/03/20(金) 12:58:07.34 ID:fIrLFwn8.net
これとか
http://suseum.jp/gq/question/2342

752 :132人目の素数さん:2015/03/20(金) 18:52:20.58 ID:5RzcZ7l8.net
>>746
(x-ab)(x-ac)(x-ad)(x-bc)(x-bd)(x-cd)=0

753 :132人目の素数さん:2015/03/20(金) 19:43:02.74 ID:GT36UrAI.net
>>748>>752
揚げ足取りだけは一流だな。

>>746はa、b、c、dを使わずに表せってことだろ

754 :132人目の素数さん:2015/03/20(金) 19:45:03.23 ID:4kPH0DVg.net
┌─────────────
|シラネーヨ
└─v────────────
  ∧ ∧
  ( ´ー`)∧ ∧
   \ < (´ー`) チラネーヨ /|
    \.⊂ヽ \/|____//
      \ ⊂ _,ノ   /
       ∪∪ ̄∪∪

755 :132人目の素数さん:2015/03/21(土) 01:34:54.56 ID:9ngVyk5j.net
>>746
x^6+x^4+x^3-x^2-1=0
ですかね

756 :132人目の素数さん:2015/03/21(土) 01:44:43.42 ID:/GyhVn2u.net
>>755
どうやったん?

757 :132人目の素数さん:2015/03/21(土) 02:04:25.36 ID:9ngVyk5j.net
4次方程式と6次方程式の解と係数の関係

758 :132人目の素数さん:2015/03/21(土) 02:26:45.29 ID:/GyhVn2u.net
計算しんどくない?

759 :132人目の素数さん:2015/03/21(土) 13:59:48.72 ID:vzbJx1C7.net
簡単にする方法なさそうだなー

760 :132人目の素数さん:2015/03/21(土) 15:31:31.03 ID:mH++Bi5J.net
問題の6個に a2乗,b2乗,c2乗,d2乗 を加えた
解を持つ10次方程式を経由したら?

761 :132人目の素数さん:2015/03/21(土) 17:27:58.58 ID:vzbJx1C7.net
どうなるの?

762 :132人目の素数さん:2015/03/21(土) 18:03:31.86 ID:vzbJx1C7.net
>>756
(x−ab)(x−ac)(x−ad)(x−bc)(x−bd)(x−cd)
=x^6−(ab+ac+ad+bc+bd+cd)(x^5+(abcd)^2 x)
+((a+b+c+d)(abc+abd+acd+bcd)−abcd)(x^4+abcd x^2)
−((abc+abd+acd+bcd)^2+((a+b+c+d)^2−2(ab+ac+ad+bc+bd+cd))abcd)x^3
+(abcd)^3

763 :132人目の素数さん:2015/03/21(土) 20:23:19.46 ID:ba+CbCKG.net
16次式経由のやつだけど
計算をあんまりやらないようにしようとしたけど、余計にめんどうだったかな?

f(x) = (x-a)(x-b)(x-c)(x-d) = x^4+x^3-1 とおくと、
a+b+c+d = abcd = -1, ab+ac+ad+bc+bd+cd = abc+abd+acd+bcd = 0,
(x-aa)(x-bb)(x-cc)(x-dd) = f(√x)f(-√x) = (x^2 + x√x - 1)(x^2 - x√x - 1) = x^4 - x^3 - 2x^2 + 1,
(x-aa)(x-ab)(x-ac)(x-ad) = a^4 f(x/a) = x^4 + ax^3 - a^4.
g(x) = (x-ab)(x-ac)(x-ad)(x-bc)(x-bd)(x-cd) = x^6 + px^5 + qx^4 + rx^3 + sx^2 + tx - 1 とおくと、
(定数項は (-1)^6 ab ac ad bc bd cd = (abcd)^3 = -1)
g(x)^2 (x^4 - x^3 - 2x^2 + 1) = Π[s,t∈{a,b,c,d}](x-st) = (x^4 + ax^3 - a^4)…(x^4 + dx^3 - d^4).
左辺のxの15, 14, 13, 2, 1乗の係数は、それぞれ 2p-1, p^2+2q-2p-2, 2pq+2r-p^2-2q-4p, t^2-2s-2, 2t,
右辺のxの15, 14, 13, 2, 1乗の係数は、それぞれ -1, 0, 0, 0, 0 であるから、 p=0, q=1, r=1, s=-1, t=0.
よって、求める方程式 g(x) = 0 は、 x^6 + x^4 + x^3 - x^2 - 1 = 0.

764 :132人目の素数さん:2015/03/21(土) 20:41:35.81 ID:/GyhVn2u.net
>>762
その変形を楽に見抜く方法ってあるん?

765 :132人目の素数さん:2015/03/22(日) 00:04:50.27 ID:281gzaF2.net
a>b>0のとき、∫[0、2π] log(a + b cos t)dt を計算せよ。
https://www.toshin.com/sp/concours/mondai/mondai_23.php

模範解答見たけど、こういう解法しかないの?

766 :132人目の素数さん:2015/03/22(日) 01:56:34.78 ID:zrXoqD+e.net
>>746
対称性を崩して ab,ac,ad と bc,cd,db で分けてみた。
やっぱ邪道かな・・・


x^4 + x^3 - 1 = (x-a) (x^3 + (a+1) x^2 + a(a+1)x + a^2(a+1))

右辺の2つめの因子を g(x) とおく。
g(x) = (x-b)(x-c)(x-d)

b+c+d = -(a+1)
bc+cd+db = a(a+1)
bcd = -a^2(a+1)
また、a^4 + a^3 - 1 = 0 より a^3(a+1) = 1

これらを用いて計算すると、

(x-ab)(x-ac)(x-ad) = a^3 g(x/a) = x^3 + a(a+1)x^2 + x + a^2

(x-bc)(x-cd)(x-db) = x^3 - (bc+cd+db)x^2 + bcd(b+c+d)x - (bcd)^2 = x^3 - a(a+1)x^2 + (a^2(a+1)^2)x - a(a+1)

(x-ab)(x-ac)(x-ad)(x-bc)(x-cd)(x-db)

= (x^3 + a(a+1)x^2 + x + a^2) (x^3 - a(a+1)x^2 + (a^2(a+1)^2)x - a(a+1))

= x^6 + x^4 + (-a(a+1)+(a+1)^2-a(a+1)+a^2)x^3 - x^2 - 1

= x^6 + x^4 + x^3 - x^2 -1

767 :132人目の素数さん:2015/03/22(日) 02:21:45.31 ID:6+nWisbd.net
>>765
f(p)=∫[0,π]log(1 + p cos x)dx , ただし0≦p≦1 とpの関数とみて、fをpで微分すると
f'(p)=∫[0,π]cos x dx/(1+p cosx)=(1/p)∫[0,π](1 - 1/(1+p cosx))dx
=(1/p)[x-{2/√(1-p^2)} arctan {√{(1-p)/(1+p)} tan(x/2)}]|_x=0~π
=π/p - π/{p√(1-p^2)}
pで積分し、p(0)=0を使って積分定数を定めると
f(p)=πlog{{1+√(1-p^2)}/2} 残りの整形はご自由に

768 :132人目の素数さん:2015/03/22(日) 12:44:37.54 ID:281gzaF2.net
>>765
模範解答の下から4行目から次の行への計算について説明お願いします。

 lim (1/n) log(α^n - (-β)^n) = log α

もうひとつ、α^n+(-β)^n は α^n-(-β)^n のミスですよね?

769 :132人目の素数さん:2015/03/23(月) 12:56:30.55 ID:p5MCw5RS.net
>>768
誤植大杉…
cos(s+t)+isin(s+t)のカッコが抜けてるし、
指摘の通りα^n+(-β)^n は α^n-(-β)^nだし、
「正の実数だから」の後で同じ計算を繰り返す意味がわからないし、
「区分求積法より」の後の計算の2行目で2πk/nとすべきところが2πknになってるし。

ちなみに
(1/n) log(α^n - (-β)^n) = (1/n) log(α^n (1-(-β/α)^n))
= log(α (1-(-β/α)^n)^(1/n)) → logα (n→∞)

770 :132人目の素数さん:2015/03/23(月) 13:02:22.15 ID:p5MCw5RS.net
>「正の実数だから」の後で同じ計算を繰り返す意味がわからないし
大事なことなので2回言ったのにそこが間違ってるっていう

771 :132人目の素数さん:2015/03/23(月) 13:28:15.33 ID:HdRm6+fd.net
>>765
これってαをこう置くのが定石なの?
答えありきで解答作ってるようで嫌だな

772 :132人目の素数さん:2015/03/23(月) 13:48:58.91 ID:EE/K0nhY.net
>>763>>766
おぬしら出来るな…
>>764
>>762 はひたすら計算しただけだからなー
やってると何となくコツが分かってくるけど説明できん

773 :132人目の素数さん:2015/03/23(月) 16:01:09.73 ID:jxu6JoMO.net
>>769
> = log(α (1-(-β/α)^n)^(1/n)) → logα (n→∞)

(1-(-β/α)^n)^(1/n) → 1 を言うところは明らかなんですか? それとも、はさみうちですか?

774 :132人目の素数さん:2015/03/23(月) 16:29:46.04 ID:z4CvzBPe.net
logα+(1/n)*log(1-(-β/α)^n) とすれば明らか

775 :132人目の素数さん:2015/03/24(火) 05:34:05.75 ID:079t8NQi.net
Lim [n → +0] x^(x^(x^x)) の値は?

776 :132人目の素数さん:2015/03/24(火) 09:55:32.81 ID:qu2ThwLP.net
lim[n→+0]x^x=1であることを利用して
それも1

777 :132人目の素数さん:2015/03/24(火) 12:20:21.32 ID:079t8NQi.net
>>751
解答が投降された
http://suseum.jp/gq/question/2342

778 :132人目の素数さん:2015/03/24(火) 21:41:56.07 ID:079t8NQi.net
l'Hopitalを使って、
lim[n→+0] x log x = lim[n→+0] (log x)/(1/x) = lim[n→+0] (1/x)/(-1/x^2) = lim[n→+0] (-x) = -∞

lim[n→+0] x^x = lim[n→+0] e^(x log x) = 1
lim[n→+0] x^(x^x) = lim[n→+0] e^(x^x log x) = 0
lim[n→+0] x(x^(x^x)) = lim[n→+0] e^(x^(x^x) log x) = ?

もう少し詳しく教えてください。

779 :132人目の素数さん:2015/03/24(火) 21:51:07.44 ID:JcRQw3cR.net
n?

780 :132人目の素数さん:2015/03/24(火) 21:51:42.11 ID:079t8NQi.net
書き間違いに気づいた。 lim[n→+0] じゃなくて lim[x→+0]

781 :132人目の素数さん:2015/03/28(土) 19:38:59.13 ID:QWWPkj1G.net
正三角形でない△ABCの辺AB、AC上にそれぞれ点M,Nがあり、
MB=BC=CNが成り立っている。△ABCの内心をI,外心をOとするとき、直線MNと直線IOは直交することを示せ。

782 :132人目の素数さん:2015/03/28(土) 20:32:33.57 ID:md2el3MA.net
AB,ACの中点を順にD,Eとすれば
OM^2-ON^2=OD^2+DM^2-OE^2-EN^2=OB^2-BD^2+DM^2-OC^2+CE^2-EN^2
=CE^2-EN^2-BD^2+DM^2=(CE+EN)(CE-EN)-(BD+DM)(BD-DM)=CN(2×CE-CN)-BM(2×BD-BM)
=BC(AC-BC)-BC(AB-BC)=BC(AC-AB)
また、IからBCに下ろした垂線の脚をFとすれば
IM^2-IN^2=IC^2-IB^2=CF^2-BF^2=BC(CF-BF)=BC{(BC+AC-AB)/2-(BC+AB-AC)/2}
=BC(AC-AB)
したがってOM^2-ON^2=IM^2-IN^2よりOI⊥MN

783 :132人目の素数さん:2015/04/03(金) 13:51:31.88 ID:utncgUtV.net
1/(1 - x - x^2 -x^3 -x^4 - x^5 - x^6) を無限級数の和の形に表せ。

784 :132人目の素数さん:2015/04/03(金) 17:58:40.95 ID:spUwRspF.net
お断りします

785 :132人目の素数さん:2015/04/04(土) 11:05:17.28 ID:8Zlf06VH.net
      ハ,,ハ
     ( ゚ω゚ )  お断りします
    /    \
  ((⊂  )   ノ\つ))
     (_⌒ヽ
      ヽ ヘ }
 ε≡Ξ ノノ `J

      ハ,,ハ ハ,,ハ
     ( ゚ω゚ )゚ω゚ )  お断りします
    /    \  \    お断りします
  ((⊂  )   ノ\つノ\つ))
     (_⌒ヽ ⌒ヽ
      ヽ ヘ } ヘ }
  ε≡Ξ ノノ `Jノ `J

    お断りします
        お断りします
            お断りします
      ハ,,ハ ハ,,ハ ハ,,ハ ハ,,ハ
     ( ゚ω゚ )゚ω゚ )゚ω゚ )゚ω゚ )  お断りします
    /    \  \  \  \    お断りします
  ((⊂  )   ノ\つノ\つノ\つノ\つ))   お断りします
     (_⌒ヽ ⌒ヽ ⌒ヽ ⌒ヽ       お断りします
      ヽ ヘ } ヘ }  ヘ } ヘ }
  ε≡Ξ ノノ `Jノ `J ノ `J ノ `J

786 :132人目の素数さん:2015/04/04(土) 16:32:47.38 ID:UyAiDdms.net
3辺が整数で、3辺の合計がnとなる三角形はいくつあるか?

ちなみに、nが1から10、および、100近辺(98〜102)での数は下記の通り
0,0,1,0,1,1,2,1,3,2,...,200,217,208,225,217,...

787 :132人目の素数さん:2015/04/04(土) 17:51:53.17 ID:OS+l0M6R.net
https://oeis.org/A005044
ふむ

788 :132人目の素数さん:2015/04/04(土) 22:40:41.16 ID:O7VVhNug.net
では、つづき
>>786の答が、 1/((1-x^2)(1-x^3)(1-x^4)) を母関数とする数列に現れる理由を述べよ。
>>786の答が、[(n^2+6)/12] - [(n+2)/4]*[n/4] で与えられることを証明せよ。
(ただし、[x]はxを超えない最大の整数を表す。)

789 :132人目の素数さん:2015/04/04(土) 22:58:17.03 ID:oohQJNWL.net
二問目はあんまり面白そうじゃないけど、一問目は面白そう

790 :132人目の素数さん:2015/04/05(日) 01:13:37.32 ID:Q5kF3thj.net
(11^n)/(n^4+n^3+n^2+n+1) が整数となるような正の整数nを全て求めよ

791 :132人目の素数さん:2015/04/05(日) 08:17:35.87 ID:/oMIQUac.net
n=0

792 :132人目の素数さん:2015/04/05(日) 12:54:41.15 ID:w4uWJ6PQ.net
n=3以外にないことを証明できるか、って話だと思うのだが、答えは用意されてるのかな?

793 :132人目の素数さん:2015/04/07(火) 05:37:09.79 ID:qLgpdS7q.net
   ,.__y⌒'ー---┐                         _へ、_、__,.ヘ、___
  /         \                       r´ -┼‐ ナ丶  
  }  (  聞 ど  l        .<´  ̄ ̄ `> 、    ┌´ (才    tナ 
 f´   )  こ う  ヽ      / 丶   /  ,   \    ヽ   '´   /   
 l   (  え も    } .n⌒l   ,  /! /{ /ヽ ヽ   }   つ  /へノ 
 {   )  ん       l |l |n  /  / j/ ーヘ{´ l || |   {  -‐ァ  -‐ァ、、
 }   (  な     {    | {  j」_ /  /        `}ノ! /   〈   (,__  (,__
. )   !     ,f   / j  /ヘ/   l ミミ      l/}    }   r‐、     
└-、         ,/⌒ヽ人  ゝ_|   | , ,     ミミ./ 廴_   )    f´    
    ゝ 、_,,,,,,,...-ゝ   〈ヽ二/__」   |    /`¬ , , | |  {__ノ})    ゚
       〈        `Tチ´‐ |   |>-| /_,ノ  \   く└----、-、 __,,.、
       \  ー‐――┘ヽ  |   ヽ、l 丶イ  | |ト、   ヽ.厂
         ` <        \\_  \ ̄l  //' | ) /
             ` < _r―ァ'  \  /、| //} /∠ト、
                ヽ. く ___//\」'/  {丶┬―┘
                      ヾ¬ー'^ヽ ー}_}ー ヽ 丶---― 、_
                   \    〉 /``マ^{、    __   } } }⌒\
                         \ `Tノ  ○卜>'´  \ノ ̄ ⌒ヽJ
                      /  ○     丶       \

794 :132人目の素数さん:2015/04/09(木) 01:00:59.84 ID:zJtxSOMb.net
まだ解けないのかw

795 :132人目の素数さん:2015/04/09(木) 03:19:18.00 ID:lnRigAFy.net
>>794
   ,.__y⌒'ー---┐                         _へ、_、__,.ヘ、___
  /         \                       r´ -┼‐ ナ丶  
  }  (  聞 ど  l        .<´  ̄ ̄ `> 、    ┌´ (才    tナ 
 f´   )  こ う  ヽ      / 丶   /  ,   \    ヽ   '´   /   
 l   (  え も    } .n⌒l   ,  /! /{ /ヽ ヽ   }   つ  /へノ 
 {   )  ん       l |l |n  /  / j/ ーヘ{´ l || |   {  -‐ァ  -‐ァ、、
 }   (  な     {    | {  j」_ /  /        `}ノ! /   〈   (,__  (,__
. )   !     ,f   / j  /ヘ/   l ミミ      l/}    }   r‐、     
└-、         ,/⌒ヽ人  ゝ_|   | , ,     ミミ./ 廴_   )    f´    
    ゝ 、_,,,,,,,...-ゝ   〈ヽ二/__」   |    /`¬ , , | |  {__ノ})    ゚
       〈        `Tチ´‐ |   |>-| /_,ノ  \   く└----、-、 __,,.、
       \  ー‐――┘ヽ  |   ヽ、l 丶イ  | |ト、   ヽ.厂
         ` <        \\_  \ ̄l  //' | ) /
             ` < _r―ァ'  \  /、| //} /∠ト、
                ヽ. く ___//\」'/  {丶┬―┘
                      ヾ¬ー'^ヽ ー}_}ー ヽ 丶---― 、_
                   \    〉 /``マ^{、    __   } } }⌒\
                         \ `Tノ  ○卜>'´  \ノ ̄ ⌒ヽJ
                      /  ○     丶       \

796 :132人目の素数さん:2015/04/09(木) 04:09:56.59 ID:lnRigAFy.net
周長一定の四角形の面積が最大となるのは、どんな四角形か?

797 :132人目の素数さん:2015/04/09(木) 13:39:12.47 ID:Izmp8vVm.net
でかい球面に貼った四角形の外側

798 :132人目の素数さん:2015/04/11(土) 12:54:57.96 ID:+tjCWHi7.net
円周率を小数点以下無限桁表記した時の何かしらの限界や法則を挙げて証明して下さい。
例えば円周率をどんなに表記したとしても同じ数値が1兆桁並ぶことは絶対にない、とか。
どんなに科学技術が進歩しても1京桁を検出することは絶対に不可能である、とか。

799 :132人目の素数さん:2015/04/11(土) 13:21:04.76 ID:mMf2cKf/.net
すべての数列を含むという円周率。「93138」は何桁目か
http://wc2014.2ch.net/test/read.cgi/math/1383139090/

800 :132人目の素数さん:2015/04/11(土) 14:20:30.00 ID:jsKmQUrg.net
10進で0〜9の数字全部が現れる

801 :132人目の素数さん:2015/04/11(土) 14:44:28.28 ID:I8SR0eto.net
ランダムな数列を無限に続けていけば、その中にあらゆる数列が現れる
これは間違いない
数列の中に93138を探す場合、
9が十個あれば、その内の一つは次の数字が3であろう
93が十個あれば、その内の一つは次の数字が1であろう・・・・・・

よって円周率がランダムかつ無限であると証明出来ればいい

802 :132人目の素数さん:2015/04/11(土) 16:14:48.87 ID:ObjKH11z.net
数列がランダムであるとはどういうことですか?

803 :132人目の素数さん:2015/04/11(土) 16:31:37.82 ID:N6l/fVa9.net
「正規数」でググれ

804 :132人目の素数さん:2015/04/11(土) 22:55:37.16 ID:fx2VCN3p.net
>>801
>ランダムな数列を無限に続けていけば、その中にあらゆる数列が現れる
>これは間違いない

それは間違いありなんだよなぁ……
93138を含まない、ランダムな数列を無限に続けることは可能だ。

805 :132人目の素数さん:2015/04/12(日) 01:43:01.70 ID:81lWzDTc.net
条件がついてる時点でランダムではないのでは
「1〜6がランダムに出るサイコロ(ただし5は出ない)」みたいな
自己矛盾してる

806 :132人目の素数さん:2015/04/12(日) 01:46:47.61 ID:jt35tDJl.net
それだけ「ランダム」という言葉は曖昧で、多様な定義があり得るということだよ

807 :132人目の素数さん:2015/04/12(日) 01:58:44.28 ID:2Ah7yMen.net
見当外れなレスが続いてますな。
>>804が正解。

808 :132人目の素数さん:2015/04/12(日) 02:27:00.09 ID:GwBqgrgC.net
前に出た数に相関があるならランダムの定義から外れる
例えば93138が出ないなら9313と8には確たる相関が存在し真のランダムではない

809 :132人目の素数さん:2015/04/12(日) 09:12:26.92 ID:swt4Y7uQ.net
前に出た数に相関があるとはどういうことですか?

810 :132人目の素数さん:2015/04/14(火) 13:40:01.21 ID:3fJdIw1N.net
定義を知らん奴が多いな

811 :132人目の素数さん:2015/04/15(水) 13:04:51.38 ID:5dcIs34t.net
5×5のマス目の各マスを、以下の条件を満たすように白色か黒色のいずれかに塗り分けることはできるか?

条件:2×2のマス目からなる部分のうち、どの相異なる2つ(重なっていても構わない)に対しても、それらの塗り分けられた色のパターンは異なる。

812 :132人目の素数さん:2015/04/15(水) 15:34:41.11 ID:h3Iy/EL0.net
2×2の塗り分けで作れるパターンは16種類
5×5のマス目の中にある、チェックすべき2×2の小ブロックも16カ所
つまり、生成可能な16パターン全てを1回づつ含むようにしなければならなく、無駄は一切許されない。
白と黒については、対称性が要求されるが、用意されているのは25マスで、同数を配置することはできない。
よって不可能

813 :132人目の素数さん:2015/04/15(水) 15:45:53.82 ID:PYF8sig5.net
>>811
でけた。
例)
11001
11001
00110
10011
00110

814 :132人目の素数さん:2015/04/15(水) 16:01:57.45 ID:h3Iy/EL0.net
そっか、角、辺、内部で、重みが違うから、内部での個数の違いを、辺で回復可能なんだ

815 :132人目の素数さん:2015/04/15(水) 16:50:26.90 ID:PYF8sig5.net
ついでにプログラムで全数検索してみた
正解は800通り。
対称性はなさそうだから、
回転や裏返しを同一視すると
ちょうど100通り。

816 :132人目の素数さん:2015/04/15(水) 16:52:19.82 ID:nFwX0VZA.net
ざっとプログラムで書き出してみたら800通りだった……って言いにきたけど被ったか。一致したから正しいんだろう
http://www.dotup.org/uploda/www.dotup.org266478.txt.html

817 :132人目の素数さん:2015/04/15(水) 17:55:30.00 ID:pTJpgUpM.net
4×4のトーラスでもできるのかな?

818 :132人目の素数さん:2015/04/15(水) 18:26:07.41 ID:a2ce/Zbt.net
□□□■
□□■□
■□■■
□■■■

参考
□□□■□
□□■□□
■□■■■
□■■■□
□□□■□

819 :132人目の素数さん:2015/04/15(水) 22:40:30.91 ID:nFwX0VZA.net
□■□□
□■■■
■■■□
□□■□
平行移動するとこうなって美しいな

820 :132人目の素数さん:2015/04/17(金) 06:48:38.60 ID:SjXFQsSJ.net
以下の条件(i),(ii),(iii)を全て満たすような整数の組(a,b,c,d)は存在するか.
(i)  1≦a<b<c<d≦90
(ii) a,b,c,dはいずれも30でない
(iii) sin a°sin d°=sin b°sin c° 

821 :132人目の素数さん:2015/04/17(金) 09:07:39.26 ID:IWpT5Gby.net
例:(18,24,48,78)

822 :132人目の素数さん:2015/04/17(金) 09:17:00.54 ID:IWpT5Gby.net
a+b+c+d<180となる例を探す。
a+b+c+d+2e=180とすると、
sin a°・sin d°=sin b°・sin c°なら当然
sin a°・sin d°・sin e°=sin b°・sin c°・sin e°
が成り立つので、
△ABCの内部に点Dがあって
∠ABD=∠DBC=e、∠BCD=a、∠DCA=b、∠CAD=d、∠DAB=c
となる整角三角形が存在する。
で、某書籍でそんな整角三角形を探してみた結果。

823 :132人目の素数さん:2015/04/17(金) 10:25:45.55 ID:IWpT5Gby.net
(6,12,24,54)
(9,12,48,81)
(12,18,42,84)
(15,18,54,75)
(18,24,48,78)

824 :132人目の素数さん:2015/04/17(金) 11:35:54.50 ID:IWpT5Gby.net
a+b+c+d>180なら
(24,27,63,84)
(48,54,66,84)
もあるな

825 :132人目の素数さん:2015/04/17(金) 12:43:59.66 ID:ewACgWf5.net
4人以上の人が円形に並んでいる。
各人はグー、チョキ、パーのいずれかの手を出しており、連続して並ぶ3人組全てについて、その3人でジャンケンをしたと考える。
このとき、3人のうち1人だけ勝っている組の個数と、1人だけ負けている組の個数は、3で割った余りが等しいことを示せ。

826 :132人目の素数さん:2015/04/17(金) 22:52:58.16 ID:GO4SxuzC.net
>>825
ゴリ押しの証明なら。

f:{ 0, 1, 2 }^3 → Z^3 を以下のように定義する。
まず、グーを0, チョキを1, パーを2で表現する。
a,b,c∈{ 0, 1, 2 } に対して、

f(a,b,c)=
(abcがあいこのとき) (1,0,0),
(abcが1人だけ勝ちのとき) (0,1,0),
(abcが1人だけ負けのとき) (0,0,1)

と定義する。次に、n≧4として、a_1,…,a_n∈{ 0, 1, 2 }が任意に与えられたとき、

S({a_i}_{i=1}^n)=
f(a_1,a_2,a_3)+f(a_2,a_3,a_4)+…+f(a_{n-2},a_{n-1},a_n)+f(a_{n-1},a_n,a_1)+f(a_n,a_1,a_2)

としてS({a_i}_{i=1}^n)を定義する。
S({a_i}_{i=1}^n)の2番目・3番目の成分がmod 3で等しいことを言えばよい。
n≧4に関する数学的帰納法で示す。

827 :132人目の素数さん:2015/04/17(金) 22:56:51.73 ID:GO4SxuzC.net
n=4のときは、(a_1,a_2,a_3,a_4)の全てのパターン(3^4通り)で確かめればよいので省略する。
n=kのとき成り立つとして、n=k+1のときは、まずS({a_i}_{i=1}^k)=(x, y, z)と置けば、
帰納法の仮定からy≡z (mod 3)である。また、簡単な計算により、

S({a_i}_{i=1}^{k+1})=
S({a_i}_{i=1}^k)−f(a_{k−1},a_k,a_1)−f(a_k,a_1,a_2)+f(a_{k−1},a_k,a_{k+1})+f(a_k,a_{k+1},a_1)+f(a_{k+1},a_1,a_2)

となることが分かる。

−f(a_{k−1},a_k,a_1)−f(a_k,a_1,a_2)+f(a_{k−1},a_k,a_{k+1})+f(a_k,a_{k+1},a_1)+f(a_{k+1},a_1,a_2)=(s, t, u)

と置くとき、S({a_i}_{i=1}^{k+1})=(x+s, y+t, z+u) となるので、y+t≡z+u (mod 3)が示せれば
数学的帰納法が達成される。y≡z (mod 3)だったから、t≡u (mod 3)が示せればよい。
(s, t, u) は a_{k−1}, a_k, a_{k+1}, a_1, a_2 だけで決定されるので、3^5 通りの全てのパターンで
t≡u (mod 3)となることを確かめれば終わる。この確認作業は省略する。以上より、>>825が成り立つ。

828 :132人目の素数さん:2015/04/17(金) 23:16:39.19 ID:DPDyr9MG.net
値はすべて mod 3で考える。
手の値をグー、チョキ、パーそれぞれ、1、0、−1とし、3人の手の値の和をゲーム値とよぶことにすると、
ゲーム値はアイコが0、ひとり勝ちが1、ひとり負けが−1になる。
ひとり当たり3回ジャンケンしたことになるので、全試合のゲーム値の総和は0。したがって >>825 がいえる。

829 :132人目の素数さん:2015/04/18(土) 00:15:54.24 ID:aEqgPHQP.net
乗法単位元が存在しない代数系Aにおいて、a∈Aに対する a^0 は何を表しているか?

例えばZで整数を表すとき、 A=2Z なら 2^0 はAの中の何を表している何? という問だ。

830 :132人目の素数さん:2015/04/18(土) 00:57:23.92 ID:iCoFEU/Y.net
代数系ってなに?

831 :132人目の素数さん:2015/04/18(土) 08:33:06.51 ID:aEqgPHQP.net
表現がアホだった。
(乗法単位元の存在を定義に含めない)環ということにしてくれ。

832 :132人目の素数さん:2015/04/18(土) 15:17:54.67 ID:Rj0rrVNG.net
nを3以上の整数とする.円周上にn個の石が並べられて輪を構成している.各石は白石か黒石のいずれかである.今,これらの石からなる輪から,以下の操作によって新たにn個の石からなる輪を作り出す:

操作:隣り合った2つの石からなる組全てに対して,それらが同じ色の石ならば白石を,異なる色の石ならば黒石を,円周上で2つの石の間に1個ずつ並べた後,それまで輪を構成していたn個の石を全て取り除く.

このとき,以下の条件を満たすnを全て求めよ:

:条件:最初に並べられたn個の石の色がどうであっても,上記の操作を有限回繰り返すことで,輪を構成する全ての石が白石になる.

833 :132人目の素数さん:2015/04/18(土) 21:11:12.02 ID:2y8DecAj.net
基準の位置を決める。
今その石が何色であろうと、その操作を2回行って、次にその場所に置かれる石の色は、
今の両隣の石の色が同じなら白、異なるなら黒になる
従って、4操作後、つまり、次の次にその場所に置かれる石の色は、今左に2個隣の石と
右に2個隣の石が同じなら白、異なるなら黒になる。
置かれている石の数が2n個なら、左にn個隣の石=右にn個隣の石=調度反対側の石であり、
同一の石なので、2n回操作すると、必ず白になる
つまり、nが偶数なら、ok

834 :132人目の素数さん:2015/04/18(土) 22:03:05.01 ID:PxtoJVc8.net
よく考えれば、置かれている石の数が奇数個でも、
(2×置かれている石の数)操作後の基準位置の石の色は、
「左に(置かれている石の数)個隣の石」と「右に(置かれている石の数)個隣の石」
の色が同じなら白、異なるなら黒となるが、この二つは=基準の石、つまり、同一の石なので
必ず白になりますね。つまり、条件無し(=3以上の整数全て)

835 :132人目の素数さん:2015/04/19(日) 02:46:50.86 ID:+TZRPpO4.net
3個なら○●●→●○●で無限ループになりますな。
5個だと●●○○○→○●○○●→●●○●●→○●●○○でやはり無限ループ。
自動的に、3の倍数や5の倍数は、必ず無限ループになる配置が存在することになる。

まだ詰めてないけど、条件を満たすnは2のべき乗ではないかと予想。

836 :132人目の素数さん:2015/04/19(日) 02:55:07.30 ID:+TZRPpO4.net
nが奇数の場合について考える。
最終的に全て白石になる状態から逆算して、出発点となりうるパターンを考えると、
直前は全黒か全白。
全黒の直前は、白黒交互に配置されている状況だが、nが奇数だとそれはありえない。
したがって、全白に収束する初期状態は、全白と全黒のみ。
よって、nが奇数の場合は、全白に収束しない初期状態が必ず存在する。

837 :132人目の素数さん:2015/04/19(日) 03:00:12.68 ID:+TZRPpO4.net
nが3以上の奇数pを約数として持つ場合、
p個からなるパターンを繰り返した配置にすれば、n=pの場合と同様の挙動となるので
白のみでも黒のみでもないp個からなるパターンを繰り返した初期配置からは全白には到達しえない。

よって、条件を満たすnは、少なくとも2のべき乗である必要がある。
(あとは、十分性の確認)

838 :132人目の素数さん:2015/04/19(日) 03:42:04.44 ID:+TZRPpO4.net
議論を簡単にするため、石の配置は、前回の配置の石を取り除いた後に
反時計回りに石の間隔の半分だけ回転するものとする。
(したがって、ある時点で時計回りに見て石Xの次が石Yだとすると、
1回操作後は、XとYが同じ色ならXの位置に白石が置かれ、
違う色ならXの位置に黒石が置かれることになる。)

白を0、黒を1とし、kを0以上の整数とすると、
時点Tから2^k回操作後の位置Pにおける石の色は、
時点Tにおける、位置Pの石の色と、位置Pから時計回りに2^k個離れた位置の石の色の
排他的論理和になることを、数学的帰納法で示すことができる。

従って、n=2^k(k=2,3,…)ならば、2^k回の操作で必ず全て白石となる。

839 :132人目の素数さん:2015/04/19(日) 04:20:02.86 ID:YQQ6oYuU.net
(x^(-1/2)+x^(1/2))^(2n)を展開したときの係数は、x^(-n)とx^(n)の係数が1であるのを除いて、
全て偶数だと思い込んでいました。831と832は全くの間違いです。
先頭と最後以外全て偶数になるのは、2^n乗の時だけのようです。

840 :132人目の素数さん:2015/04/19(日) 16:00:46.88 ID:YYGac0T3.net
2種類の円順列を考える。
ひとつは、列の長さが m、総和が n、要素が非負整数のもの。
これらの集合を A(m,n)とする。
もうひとつは、列の長さが m、総和が n、要素が0と1だけのもの。
これらの集合を B(m,n)とする。
例:
A(5,3)={30000, 21000, 20100, 20010, 20001, 11100, 11010}
A(3,5)={500, 410, 320, 302, 311, 221}
B(8,3)={11100000, 11010000, 11001000, 11000100, 11000010, 10101000, 10100100}
B(8,5)={11111000, 11110100, 11110010, 11101100, 11101010, 11100110, 11011010}
(円順列は開いて表わした。1234, 2341, 3412などは同じ円順列。)

もう2種類、数列を考える。こちらは円順列ではない。
ひとつは、列の長さが m、総和が nの倍数、n-1≧a[1]≧a[2]≧…≧a[m]≧0 をみたすもの。
これらの集合を S(m,n)とする。
もうひとつは、列の長さが m、総和が nの倍数、n-1≧a[1]>a[2]>…>a[m]≧0 をみたすもの。
これらの集合を T(m,n)とする。
例:
S(5,3)={22221, 22200, 22110, 21111, 21000, 11100, 00000}
S(3,5)={442, 433, 410, 320, 311, 221, 000}
T(3,8)={763, 754, 710, 620, 530, 521, 431}
T(5,8)={76542, 76210, 75310, 74320, 65410, 65320, 64321}

次を示してください。
#A(m,n) = #A(n,m) = #B(m+n,n) = #B(m+n,m) = #S(m,n) = #S(n,m) = #T(n,m+n) = #T(m,m+n)
ここで、#XはXの要素の個数を表わす。

できればA(またはB)とS(またはT)の間の全単射による組合せ論的証明を見つけてほしいです。俺が知らないので。

841 :132人目の素数さん:2015/04/20(月) 16:30:15.01 ID:OClVmyMl.net
平面上の無限個の点からなる集合Sは、以下の条件を満たす。

条件:Sに含まれる任意の2点間の距離は整数

このとき、Sの全ての点は同一直線上にあることを示せ。

842 :132人目の素数さん:2015/04/20(月) 20:26:57.57 ID:29zkaLXd.net
もう締め切りを過ぎたんで貼る
https://www.toshin.com/sp/concours/

この手の問題のお決まりの解決方法があったら教えてクリリン!

843 :132人目の素数さん:2015/04/21(火) 14:07:11.97 ID:UggWszef.net
>>840
横の長さm,縦の長さnの碁盤目状の道の左上端から右下端への最短ルートを考え,
さらにそのルートを無限個,終点と始点を連結してつなげたものを考える。
その無限経路の種類数を,平行移動して一致するものは同一視してカウントしたものが

#A(m,n) = #A(n,m) = #B(m+n,n) = #B(m+n,m) = #S(m,n) = #S(n,m) = #T(n,m+n) = #T(m,m+n)

に相当する。

例えば,(m,n)=(5,3)のとき,(→→↓→↓↓↓↓)の繰り返しという経路を考えたとき
・(→,↓)を(1,0)または(0,1)と対応付けたものがB(m+n,n),B(m+n,m)の要素
・↓の後に→が何個並ぶかをリストアップした(10002)という円順列がA(m,n)の要素
・→の後に↓が何個並ぶかをリストアップした(014)という円順列がA(n,m)の要素
・経路上のある→の直後からk番目に出現する↓までの間に出現する→の個数をk=1〜mについて調べ
 それを逆順に並べると(22221)(11110)(20000)のn個の数列が得られるが,
 このうち合計がnの倍数となるのは(22221)のみで,これがS(m,n)の要素
・経路上のある↓の直後からk番目に出現する→までの間に出現する↓の個数をk=1〜nについて調べ
 それを逆順に並べると(440)(433)(322)(211)(100)のm個の数列が得られるが,
 このうち合計がmの倍数となるのは(433)のみで,これがS(n,m)の要素
・経路上のある地点から,各→までの距離を逆順に並べると
 (310)(720)(761)(650)(754)(643)(532)(421)のm+n個の数列が得られるが,
 このうち合計がm+nの倍数となるのは(754)のみで,これがT(n,m+n)の要素
・経路上のある地点から,各↓までの距離を逆順に並べると
 (24567)(13456)(02345)(12347)(01236)(01257)(01467)(03567)のm+n個の数列が得られるが,
 このうち合計がm+nの倍数となるのは(754)のみで,これがT(n,m+n)の要素

844 :132人目の素数さん:2015/04/21(火) 14:11:22.25 ID:UggWszef.net
>>843
最後の3行修正
・経路上のある地点から,各↓までの距離を逆順に並べると
 (76542)(65431)(54320)(74321)(63210)(75210)(76410)(76530)のm+n個の数列が得られるが,
 このうち合計がm+nの倍数となるのは(76542)のみで,これがT(m,m+n)の要素

845 :132人目の素数さん:2015/04/21(火) 15:01:21.40 ID:UggWszef.net
>>840
ところで、SとTに関して、mとnは互いに素という制約はついてませんでしたか?

846 :132人目の素数さん:2015/04/21(火) 16:05:46.81 ID:HCzWEoa0.net
>>840,843
すみません。昔のメールをそのまま持ってきたもので、その後訂正したのを忘れてました。
次のように T(m,n)を変更してください。そうすれば、m,nが互いに素でなくても大丈夫だと思います。
>>843でのSやTの対応は、条件を満たすものが存在して、しかも一意的である必要がありますが、
それはどう示すのでしょうか?

×
もうひとつは、列の長さが m、総和が nの倍数、n-1≧a[1]>a[2]>…>a[m]≧0 をみたすもの。
これらの集合を T(m,n)とする。


もうひとつは、列の長さが m、総和を nで割ったときの余りが、mが奇数のときは 0、mが偶数のときは m/2 、
n-1≧a[1]>a[2]>…>a[m]≧0 をみたすもの。これらの集合を T(m,n)とする。

あと
×A(3,5)={500, 410, 320, 302, 311, 221}
○A(3,5)={500, 410, 401, 320, 302, 311, 221}

847 :132人目の素数さん:2015/04/21(火) 22:28:35.01 ID:UggWszef.net
>>846 (日付が変わる前に)

>>843でのSやTの対応は、条件を満たすものが存在して、しかも一意的である必要がありますが、
>それはどう示すのでしょうか?
それを示すために「互いに素」の条件が必要だという話をしているのですが。
そんなに難しい話ではないので、ご自分でお考え下さい。

それと、T(m,n)の定義の変更は、あまり意味がない気がします。
どちらにせよ互いに素でないとダメという状況になんら変わりはありません。

848 :132人目の素数さん:2015/04/22(水) 02:40:23.26 ID:3m567tgt.net
>>831
乗法単位元の存在しない環は乗法単位元付きの環に埋め込める
http://ja.wikipedia.org/wiki/%e6%93%ac%e7%92%b0

849 :132人目の素数さん:2015/04/22(水) 07:45:14.78 ID:rwqRr9Hn.net
さしあたってそういうのはどうでもいいんだ。

850 :132人目の素数さん:2015/04/22(水) 09:14:01.65 ID:YyVmQC12.net
どうでもよくはないだろう。

単位元を添加した拡大環上で累乗を定義するとき、
慣例どおりにやると0乗の値は1とするから、
部分環では0乗が閉じていないことになる。

851 :132人目の素数さん:2015/04/22(水) 09:25:51.71 ID:NCPVJCXm.net
で?

852 :132人目の素数さん:2015/04/22(水) 09:48:58.86 ID:YyVmQC12.net
何でも好きに定義したらよいが、
単位元を持たない環では
指数法則を満たす0乗は定義できない。
そんだけのこと。

853 :132人目の素数さん:2015/04/22(水) 16:14:59.43 ID:pi/Si0Dq.net
>>847
m,nが互いに素でないときは、>>843の対応がうまくいかないだけで、
#A(m,n) = #B(m+n,n) = #S(m,n) = #T(n,m+n) などは常に成り立ちます。
すべての場合に成り立つ一対一対応を見つけてほしいのです。

854 :132人目の素数さん:2015/04/22(水) 16:46:33.06 ID:ZZFcCH0d.net
>>853
そうなんですね
じゃあ当方ではわかりません
こちらの思いついた方法はmとnが互いに素の場合にのみうまくいくというだけですから。

それが分かっていたのなら
>>843でのSやTの対応は、条件を満たすものが存在して、しかも一意的である必要がありますが、
>それはどう示すのでしょうか?
という聞き方はミスリードだとは思いますが…

855 :132人目の素数さん:2015/04/22(水) 18:15:51.27 ID:ZZFcCH0d.net
B(m,n)から得られる全ての順列(円順列でない)の集合をP(m,n)
P(m,n)のうち右端が1のものの集合をQ(m,n)とするとき
Q(m+n,n)の1つの要素について、その中の0を右側から順にたどり、それより左に1が何個あるかを並べたもの の集合をX(m,n)
P(m+n,n)の1つの要素について、その中の1を右側から順にたどり、それらの順列内の場所を左端を0として表したもの の集合をY(n,m+n)
とすると、
N = #B(m+n,n) として
#X(m,n) = #Q(m+n,n) = nN
#X(n,m) = #Q(m+n,m) = mN
#Y(n,m+n) = #Y(m,n+m) = #P(m+n,n) = #P(m+n,m) = (m+n)N
となります。

で、
集合Q(m+n,n)を「円順列として同じ」という同値類に分割したときの代表元の集合がB(m+n,n)となっており、Q(m+n,n)とX(m,n)が1対1対応しているのですが、
集合X(m,n)を「別の切り口」でn個ずつの同値類に分割したときの代表元の集合がS(m,n)となるような「切り口」を探す
という方向性の方が有意義なのではないかと。

つまり、X(m,n)とB(m+n,n)の間と、X(m,n)とS(m,n)の間には、それぞれn対1の対応関係があるが、
B(m+n,n)とS(m,n)の間に意味のある1対1対応はないのではないかということです。

同様に、Y(n,m+n)とB(m+n,n)、Y(n,m+n)とT(n,m+n)の間にも、それぞれ異なる
m+n対1の対応関係があるのではないでしょうか。

(mとnが互いに素の場合は、たまたま同じ切り口でも成立しただけ、ということで。)

856 :132人目の素数さん:2015/04/22(水) 18:28:09.14 ID:ZZFcCH0d.net
言うまでもないと思いますが
ここで定義したX(m,n)は、S(m,n)の定義から総和の条件を除いたもの、
Y(m,n)はT(m,n)の定義から総和の条件を除いたものと一致します。

857 :132人目の素数さん:2015/04/22(水) 23:12:08.67 ID:ZZFcCH0d.net
あ、全然違うな…すみません >>855>>856はキャンセル。
>#X(m,n) = #Q(m+n,n) = nN
>#X(n,m) = #Q(m+n,m) = mN
>#Y(n,m+n) = #Y(m,n+m) = #P(m+n,n) = #P(m+n,m) = (m+n)N
とか言ってる時点で、暗黙のうちに「mとnは互いに素」を使ってた。

858 :132人目の素数さん:2015/04/24(金) 16:11:21.13 ID:Zj1CkDBa.net
以下の条件(1)(2)をともに満たすような正の整数からなる数列{a_n}が存在することを示せ。
 (1)任意の正の整数nに対して、Σ_{k=1}^{n} {a_k}^3は平方数
(2){a_n}の階差数列は全ての項が相異なる平方数

859 :132人目の素数さん:2015/04/24(金) 23:22:40.02 ID:GW+jCLBg.net
a_n=F[1]^2+F[2]^2+F[3]^2+...+F[n]^2=F[n]*F[n+1]; F[1]=1,F[2]=1,F[n]=F[n-1]+F[n-2]

860 :132人目の素数さん:2015/04/24(金) 23:45:20.84 ID:oCBbq7+G.net
へー(´・ω・`)

861 :132人目の素数さん:2015/04/25(土) 04:22:50.58 ID:zOxI0rTh.net
F[n+2]^2-F[n-1]^2
=(F[n+2]+F[n-1])*(F[n+2]-F[n-1])
=(F[n+1]+F[n]+F[n-1])*(F[n+1]+F[n]-F[n-1])=(2F[n+1])*(2F[n])=4*F[n+1]*F[n]
両辺に、(F[n+1]*F[n])^2を掛けると
(F[n+2]*F[n+1]*F[n])^2-(F[n+1]*F[n]*F[n-1])^2=4*F[n+1]^3*F[n]^3
F[0]=F[2]-F[1]=0に注意して、和を考えると、
Σ[k=1,n]F[k]*F[k+1]={F[n+2]*F[n+1]*F[n]/2}^2

862 :132人目の素数さん:2015/04/25(土) 04:25:01.99 ID:0fursNQo.net
×:Σ[k=1,n]F[k]*F[k+1]={F[n+2]*F[n+1]*F[n]/2}^2
○:Σ[k=1,n](F[k]*F[k+1])^3={F[n+2]*F[n+1]*F[n]/2}^2

863 :132人目の素数さん:2015/04/26(日) 04:43:52.03 ID:Csln/AA5.net
10進法で各桁に同じ数字が2度以上現れない自然数を「プレミア数」と
呼ぶことにする. 例えば, 2015はプレミア数である.
A,B(A<B)はともに4桁のプレミア数であり, Aより大きくかつBより小さいプレミア数は存在しないという. このとき, B-Aの値が最大となるようなA,Bを全て求めよ.

864 :132人目の素数さん:2015/04/26(日) 14:55:33.35 ID:boKeo1S0.net
(A,B)=(1098, 1203) (8796, 8901)

>>842はまだ締切すぎてないよね

865 :132人目の素数さん:2015/04/26(日) 15:21:20.30 ID:T3D4IB1d.net
>>864
何か問題が差し替わってる。関数方程式だったのに…

866 :132人目の素数さん:2015/04/26(日) 15:43:35.88 ID:boKeo1S0.net
>>865
ちなみにどんな感じの問題だったの?

867 :132人目の素数さん:2015/04/26(日) 15:53:57.45 ID:T3D4IB1d.net
>>866
週明けに捨てるゴミ袋を漁って、計算用紙を発掘!

f : N → N
∀x、∀y ∈N、f(x + f(x)・f(y)) = (1+y)・f(y)

868 :132人目の素数さん:2015/04/26(日) 15:59:38.01 ID:zBcOm79P.net
Nって正の整数?

869 :132人目の素数さん:2015/04/26(日) 16:01:49.11 ID:T3D4IB1d.net
すまん、Nは自然数全体の集合

870 :132人目の素数さん:2015/04/29(水) 12:50:50.68 ID:+SnV9o4i.net
だーかーらー自然数に0が入ってるんか?

871 :132人目の素数さん:2015/04/29(水) 12:57:56.97 ID:KdQ2RsbE.net
入らないよん、ちゃん!

872 :132人目の素数さん:2015/04/29(水) 13:57:04.19 ID:oVt7pGEk.net
>>867
問題おかしい気がする

873 :132人目の素数さん:2015/04/29(水) 16:32:39.48 ID:XzFULP7H.net
n≧2に対し、
f(n)=Π[k,1,(n-1)/2]{3+2cos(2kπ/n)}
で定義される f(n)は
f(n+2) = f(n+1) + f(n)
を満たすことを示せ
(kの上限 (n-1)/2 が半整数の時は、小数部分を切り捨てる)

874 :132人目の素数さん:2015/04/29(水) 16:35:27.43 ID:Ec9SWfhx.net
nが整数とかいう後出しは当然なしだよな

875 :132人目の素数さん:2015/04/29(水) 16:57:16.76 ID:3AMk9r5K.net
申し訳ないけど、盲目的な厳密教徒は有害なので黙ってて

876 :132人目の素数さん:2015/04/29(水) 20:49:17.97 ID:JMSWHRVp.net
正の整数nについて√nを連分数で表したとき周期が3となるのはどういうときか。
ただしここでいう連分数とはa+1/(b+1/(c+...))の形のものでa,b,c,...は正の整数とする。

877 :132人目の素数さん:2015/05/02(土) 00:34:09.35 ID:hHOqtvQG.net
>>876の類題
素数pについて√pの連分数展開の周期が4となるときp+2は平方数となることを示せ。

878 :132人目の素数さん:2015/05/05(火) 09:22:39.17 ID:i2P9wMwe.net
結局>>841って誰も解けないの?

879 :132人目の素数さん:2015/05/06(水) 00:34:58.14 ID:y6+sjrNo.net
>>841
辺の長さが3,4,5の直角三角形の頂点上に
無限個の点がそれぞれ分布していれば、
題意の条件を満たすが一直線上にない。

880 :132人目の素数さん:2015/05/06(水) 00:39:12.17 ID:W/Ktf+F1.net
>>879
一つの頂点には一つの点しか乗れないのですがそれは

881 :132人目の素数さん:2015/05/06(水) 00:45:39.39 ID:+MaE56nX.net
後出しご苦労。

882 :132人目の素数さん:2015/05/06(水) 00:50:06.55 ID:Th7k4UK7.net
後出しも何も、「無限個の点からなる集合」という表現を見てどうして>>879の設定が出てくるんだ

883 :132人目の素数さん:2015/05/06(水) 00:56:53.10 ID:+MaE56nX.net
識別を明確にしてないから、だね。残念。

884 :132人目の素数さん:2015/05/06(水) 00:57:58.41 ID:Th7k4UK7.net
は?
いや、何言ってんの

885 :132人目の素数さん:2015/05/06(水) 00:59:45.46 ID:Th7k4UK7.net
>>879自身は冗談で言ってるとは思うんだが、この遊びに便乗した>>881はガチっぽいんだよな…

886 :132人目の素数さん:2015/05/06(水) 01:01:26.81 ID:+MaE56nX.net
「P,Qを相異なる2点とする」という表現はよく目にするのではないかい?

887 :132人目の素数さん:2015/05/06(水) 01:04:30.49 ID:+MaE56nX.net
>>885
ガチも何も、>>841の表現の迂闊さを突いたのが>>879
冗談でもなんでもない。

888 :132人目の素数さん:2015/05/06(水) 01:04:42.53 ID:Th7k4UK7.net
本当に「ガチ」だったか

あのな、同じものに異なる名前を付けることが可能だから「P,Qを相異なる2点とする」という表現が必要になるんだよ
でも「無限個の点からなる集合」という表現に名前なんて出てこないからね
これ、単に「無限集合」の言い換えだよ?分かってた?

889 :132人目の素数さん:2015/05/06(水) 01:05:39.29 ID:+MaE56nX.net
後出しご苦労。

890 :132人目の素数さん:2015/05/06(水) 01:06:31.66 ID:Th7k4UK7.net
>>889
何が後出しなのかはっきりと言ってみ?w

891 :132人目の素数さん:2015/05/06(水) 01:07:46.31 ID:+MaE56nX.net
惨めな抵抗、お疲れ!

892 :132人目の素数さん:2015/05/06(水) 01:09:29.74 ID:+MaE56nX.net
>>890
そうそう、それで>>841に対するお前の解釈での解はどこ?

893 :132人目の素数さん:2015/05/06(水) 01:22:27.68 ID:+MaE56nX.net
誤解のない表現の一例としては

座標面上の無限個の相異なる点からなる集合に属する任意の2点間の距離が整数であるという。
この集合に属する全ての点はある一直線上にあることを示せ。

かな。

あ、オレにはこの解は思いついてない。

これ真かどうかも知らないが
>>879が真っ当な回答の一つであることはよ〜く分るよ。
お前も、そうは思うだろ。

894 :132人目の素数さん:2015/05/06(水) 01:25:04.22 ID:w4qdyJpP.net
>>841はSの任意の相異なる3点が同一直線上にあることを示せばいいんだよね
うーん

895 :132人目の素数さん:2015/05/06(水) 13:01:25.87 ID:YAoHrVAA.net
>>893
全然思わん
>>841 の表現に問題は無い

896 :132人目の素数さん:2015/05/06(水) 13:15:30.42 ID:T1/5hPxK.net
対偶命題を考えれば、簡単に証明できるんじゃないのか?

897 :132人目の素数さん:2015/05/06(水) 15:54:06.31 ID:Zlq98Q5A.net
>>841
異なる2つの双曲線の共有点が高々4個であることを示せばいい

898 :132人目の素数さん:2015/05/06(水) 18:53:00.95 ID:7amcWDQv.net
結論通りじゃないなら、Sはとある格子上(したがってdiscrete)
まで考えたところで酒の時間だ

899 :132人目の素数さん:2015/05/07(木) 04:29:49.85 ID:zpDErsxr.net
今気付いたのだけど、 >>841 の問題って、無限の理解が曖昧だと
うっかり間違った対偶をとって偽だと言ってしまいそうで怖いな。

次の命題が偽であることはわりと簡単に証明できるのだけど。

「座標平面上のn個の点からなるある集合Sが、
Sに属する任意の2点間の距離が整数であり、
なおかつSに属するすべての点を通る直線は存在しないという条件を満たすとき、
nの値には上界が存在する」

900 :132人目の素数さん:2015/05/07(木) 04:42:18.54 ID:zpDErsxr.net
要するに、有限個の集合の個数に上限はないけど、
無限個の集合は存在しなさそうってことのようなので、
「*個以上であれば必ず同一直線上」というアプローチにはno chanceって話。

901 :132人目の素数さん:2015/05/07(木) 04:57:27.51 ID:zpDErsxr.net
任意の個数の「全ての点を通る直線は存在しないが、
全ての距離が整数」である集合の作り方の例:

nを3以上の自然数とする。
a(k)=2k,b(k)=k^2-1という2つの整数列を考え、
a(1)〜a(n-1)の最小公倍数をLとし、n個の点P_0〜P_{n-1}を
P_0(0,L)
P_k(b(k)L/a(k),0) (k=1,…,n-1)
とすれば、P_1〜P_{n-1}は全てx軸上にあるが、点P_0だけはx軸上になく、
なおかつ、任意の2点間の距離は整数。

(ピタゴラス数が無限に存在することを利用。最小公倍数を使っている時点で
この方法で無限個の集合は構築できない。)

902 :132人目の素数さん:2015/05/07(木) 05:48:28.06 ID:7SgTC73q.net
a^2+b^2=c^2、a<b<c、a,b,cは整数とすると、1≦c-b=a^2/(b+c)<a^2/(2b)
つまり、b < a^2/2
三点、(0,a),(0,0),(b,0)を取って、これがピタゴラス三角形になるためには、
b < a^2/2 という条件があるため、無限にはとれない。
直線外に点を求めると、(直線との距離に依存する)上限が現れる

903 :132人目の素数さん:2015/05/07(木) 06:27:59.45 ID:zpDErsxr.net
>>902
つまりそれは「全ての点の距離が整数である無限個の点の集合Sの要素のうち、
1つの直線上に無限個の点が存在して、なおかつその直線以外にも
点が存在する」ことがありえないということだと思いますが.
それだけでは、「無限個の点を含む直線は存在しないが、点自体は無限に存在する」
ような例が存在しないことの証明にはまだなっていないですよね。

904 :132人目の素数さん:2015/05/07(木) 13:56:38.90 ID:2Q25sLle.net
「無限個存在する」が、もし、
「十分大きな数Nを用意し、そのNに対応する点の配置法がある」なら、
N-1個を直線上に取り、『最後』の1個を直線外に取ることにより、完成させることができるだろう。
しかし、無限個に不相当な『最後』という言葉が使われているように、「無限個存在する」の意味は
この様なことでは無いはず。


「無限個存在する」を証明するためには、
「十分多くの数が存在しているところに、新たに、何度でも加えることができる」
という動的な意味が必要。

直線上にだけ点を配置する分には、いくらでも加えることが可能だけど、
一端、直線外に点を求めると、その点と直線の距離をaとすると、
その直線上の、-a^2/2〜a^2/2 という範囲内にしか、点を求めることはできない。
精々、〜a^2 回しか追加できない。つまり、有限回。
この意味で、「無限個存在する」は否定できているとおもうが。

905 :132人目の素数さん:2015/05/07(木) 16:04:30.36 ID:V7lETSuQ.net
点間の距離が整数、つまり離散的ってことが重要なのかね
有理数だと例があるんだろか

906 :132人目の素数さん:2015/05/07(木) 18:33:29.61 ID:zpDErsxr.net
>>905
有理数でいいなら
>>901 の座標を全部Lで割れば
それが例になると思います

907 :132人目の素数さん:2015/05/07(木) 19:38:46.71 ID:04I/B1WS.net
無限という語の意味について後付の解説を始めた時点で
元の問題の表現が曖昧だったと証明されたということが
理解できていないのだろうか

本当に分かっているなら素直に不備を認めて
出題意図に沿った表現に改めるだけの話だろう

908 :132人目の素数さん:2015/05/07(木) 20:21:59.06 ID:yTPwzRkT.net
お前が理解できないだけ

909 :132人目の素数さん:2015/05/07(木) 20:27:12.52 ID:t9xczrar.net
いつまでもつまらねー問題といてんじゃねーよ

910 :132人目の素数さん:2015/05/07(木) 20:46:21.93 ID:yTPwzRkT.net
http://www.geocities.jp/ikuro_kotaro/koramu/1686_m8.htm

911 :132人目の素数さん:2015/05/07(木) 20:53:35.55 ID:LtAnJ1ty.net
>>904
なんでaが整数だっていえるの?

912 :132人目の素数さん:2015/05/07(木) 22:46:18.59 ID:zpDErsxr.net
>>910
なるほど、エルデシュの名前を冠した定理なのか。
それにしても、佐藤郁郎氏のサイトは何でもよく集めてるなー

913 :132人目の素数さん:2015/05/07(木) 23:45:29.44 ID:0/7iUFdB.net
>>910
鮮やかな証明だ。感激した。

914 :132人目の素数さん:2015/05/09(土) 05:43:03.75 ID:2Ji3jztR.net
f : R→R
∀x, ∀y ∈R , f(f(x)+y) = 2x + f(f(f(y))-x)

915 :132人目の素数さん:2015/05/09(土) 13:34:50.36 ID:p3UoDKP0.net
>>914
fを求める問題?

916 :132人目の素数さん:2015/05/09(土) 14:24:47.79 ID:thUnZu1m.net
解の1つがf(x)=xなのはわかった

917 :132人目の素数さん:2015/05/09(土) 18:33:22.73 ID:2Ji3jztR.net
こういう関数方程式の問題のお決まりの解法ってなんだろうな

918 :132人目の素数さん:2015/05/10(日) 16:02:40.40 ID:CIiswLGB.net
>>914
2003 春合宿なら
f(f(x)+y) = 2x + f(f(y)-x) 
だけど、それとは別の問題?

919 :132人目の素数さん:2015/05/10(日) 16:31:57.91 ID:3rqDb3p4.net
>>918
別。

920 :132人目の素数さん:2015/05/10(日) 18:48:38.37 ID:CIiswLGB.net
>>914
与式においてy=-f(x)として
f(0)=2x+f(f(f(-f(x)))-x)
この左辺は定数で, xは実数全体を動くのでfは全射.
a,b∈R, f(a)=f(b)とする. 与式においてy=a, bとして
f(f(x)+a)=f(f(x)+b)
fは全射よりf(x)は任意の実数値をとりうるのでfは周期b-aをもつ.
また, 与式においてx=a, bとして
2a+f(f(f(y))-a)=2b+f(f(f(y))-b)
f(f(y))-a-{f(f(y))-b)}=b-aとfの周期性よりf(f(f(y))-a)=f(f(f(y))-b)なので
a=b よってfは単射.
与式においてx=y=0として
f(f(0))=f(f(f(0))) fは単射なのでf(0)=0
∀c∈R, 与式においてx=-c, y=0として
f(f(-c))=-2c+f(c)・・・@
また, 与式においてx=0, y=-cとして
f(-c)=f(f(f(-c)))
fは単射なので-c=f(f(-c))・・・A
@, Aより-2c+f(c)=-c
よってf(c)=c
逆にf(x)=xは与式を満たす. □

921 :132人目の素数さん:2015/05/10(日) 19:17:28.97 ID:WPPJSoaI.net
へー(´・ω・`)

922 :132人目の素数さん:2015/05/10(日) 19:38:46.25 ID:CIiswLGB.net
以下の条件(i), (ii)をともに満たす集合Sが存在するような正の整数nをすべて決定せよ:
 条件(i):Sは相異なるn個の正の整数からなる
   (ii):Sの任意の空でない部分集合Aに対して, Aの要素の総和は累乗数でない

923 :132人目の素数さん:2015/05/11(月) 12:05:04.54 ID:JOB5mkPY.net
Wikipedia「累乗数」の
>累乗数を小さいほうからa_1=1, a_2=4, ...と並べるとき、a_{i+1}-a_iはiと共に無限大に発散すると予想されている
という予想が正しいならば、
数学的帰納法により任意の自然数nについて条件を満たす集合Sが存在することが示せるので
答えは「全ての自然数」
(その予想を使わずに示せるかどうかは知らん)

n個の要素からなる条件を満たすSに対し、その要素の総和をMとする
a_{i+1}-a_i≧M+2となるようなa_iを選び、
Sにa_i +1を追加した集合S'を作ると、S'も条件を満たす

924 :132人目の素数さん:2015/05/12(火) 01:00:00.40 ID:5Kxo+n16.net
>>923
もっとずっと初等的に解ける

925 :132人目の素数さん:2015/05/12(火) 01:21:14.84 ID:1aK7nWo1.net
そのページに書いてある「ゴールドバッハの定理」を使えば同じ証明が通用するじゃん

926 :132人目の素数さん:2015/05/12(火) 03:28:34.59 ID:s0mVYDPm.net
x, y, z は自然数で x≦y≦z のとき、x^y + y^z = z^x の解

927 :132人目の素数さん:2015/05/12(火) 05:22:20.55 ID:5Kxo+n16.net
f(x)=(logx)/xがx≧3で単調減少であること(微分で容易に証明可)と
y^z<x^y+y^z=z^x≦z^y よりy≦2
y=1のときは(x,y,z)=(1,1,2)
y=2のときはf(2)=f(4)よりz≦4 順に調べて解なし

928 :132人目の素数さん:2015/05/12(火) 05:34:40.23 ID:5Kxo+n16.net
>>925
もっと具体的に教えて

929 :132人目の素数さん:2015/05/12(火) 06:11:02.69 ID:I13rWrE/.net
>>922
任意のnに対して
n(n+1)/2 < p をみたす十分大きい素数pをとり
S={p,2p,...,np} とする

930 :132人目の素数さん:2015/05/12(火) 11:59:59.78 ID:3BlB8DxD.net
a(1)=p(1)。
a(n)=a(n−1)p(n−1)p(n)。

931 :132人目の素数さん:2015/05/12(火) 17:01:29.43 ID:5Kxo+n16.net
>>929 >>930
正解

932 :132人目の素数さん:2015/05/12(火) 17:11:21.06 ID:5Kxo+n16.net
正の整数の組(a, b, c)であって,
a/b+b/c+c/a と a/c+c/b+b/a がともに整数であるようなものをすべて求めよ.

933 :132人目の素数さん:2015/05/13(水) 01:19:18.79 ID:8q0tyZQK.net
>>932
a=b=c(aは任意の正の整数)
しかないようですな

a,b,cのGCDをgとして、a=gA,b=gB、c=gCとおくと
A/B+B/C+C/A=m, A/C+C/B+B/A=n(m,nは整数)
これを変形して
BC^2=A(mBC-CA-B^2)
CB^2=A(nBC-AB-C^2)
これから、Aの素因数の1つpが、Aの素因数分解の中でp^iの形で現れるとすると
・pはB,Cのいずれかの素因数であるが、両方ではない。
・pがBの素因数であり、Bの素因数分解の中でp^jの形で現れるとき、j≧i
・pがCの素因数であり、Cの素因数分解の中でp^jの形で現れるとき、j≧i
となり、同様の議論を繰り返すと結局
A=yz, B=zx, C=xy(ただし、xとy,yとz,zとxはそれぞれ互いに素)となる。

これを元の式に代入して整理すると
y/x+z/y+x/z=m,z/x+y/z+x/y=nとなり
また同様の変形をすると、pがxの素因数ならばpはyまたはzの素因数となるが、これは
互いに素であることと矛盾するので、xには素因数は存在しないことになり、
結局x=y=z=1

よって、A=B=C=1,a=b=c=g

934 :132人目の素数さん:2015/05/13(水) 03:37:29.99 ID:si3QnkKj.net
>>932
(x-a/b)(x-b/c)(x-c/a)は整数係数モニック多項式なのでa/b,b/c,c/aは整数すなわちa=b=c

935 :132人目の素数さん:2015/05/13(水) 10:49:10.41 ID:86nDKUeq.net
(n+1)!/(2^n-1)が3で割り切れない整数となるような正の整数nを全て求めよ

936 :132人目の素数さん:2015/05/13(水) 11:42:55.67 ID:M1de1A4J.net
>>934
しゅごい

937 :132人目の素数さん:2015/05/13(水) 12:38:42.03 ID:ZwfUsv0M.net
>>935
それが人に物を頼む態度か!ああ!?

938 :132人目の素数さん:2015/05/13(水) 14:47:35.70 ID:86nDKUeq.net
>>937
誠に申し訳ございませんが、(n+1)!/(2^n-1)が3で割り切れない整数となるような正の整数nを全て求めて頂けないでしょうか。 無礼な依頼であることは承知ですが、ご検討頂ければ幸いです。何卒宜しくお願い申し上げます。

939 :132人目の素数さん:2015/05/14(木) 14:09:30.58 ID:7tSBb3jJ.net
むずかしいからダメ

940 :132人目の素数さん:2015/05/14(木) 17:01:35.76 ID:ACZwxwZd.net
nが3でちょうどm回割り切れるとして条件からm(またはn)の不等式を作り、それが十分大きいnで成り立たないことを示す

941 :132人目の素数さん:2015/05/14(木) 21:08:53.78 ID:bIHfcpAN.net
n=2,4

942 :132人目の素数さん:2015/05/14(木) 21:20:28.14 ID:qYPony3p.net
n=1 も追加

943 :132人目の素数さん:2015/05/15(金) 08:08:07.92 ID:06V0dwp2.net
ヒント
2以上の整数kに対して、kを素因数分解したときの3の指数をord kと書くことにする。nが条件を満たすとき
1.ord(2^n-1)=ord n +1 を示す
2.ord(n+1)! の考察によりn≧9においてn≦3ord n +1 を示す
3.n≧9において(n+1)!<2^(2^((n+1)/2)) を示す

944 :132人目の素数さん:2015/05/15(金) 08:45:47.10 ID:06V0dwp2.net
>>943
無論n≧2で

945 :132人目の素数さん:2015/05/15(金) 10:39:03.71 ID:RnqkudAI.net
次の2条件をみたす f : R^+ → R^+ を求めよ。R^+は正の実数の集合
(1) ∀x, ∀y ≧0 に対して、f(x)f(y) = f(xy) + f(y/x)
(2) 1<x<y ⇒ f(x)<f(y)

946 :132人目の素数さん:2015/05/15(金) 11:05:43.38 ID:pEpb7hJG.net
nを素因数分解した場合の3の指数をmとしたとき、4^n-1は3^(m+1)で割り切れる

947 :132人目の素数さん:2015/05/15(金) 11:08:50.42 ID:RnqkudAI.net
f : R^+ → R^+
∀x, ∀y >0 に対して、f(f(x)+f(y)) = x+y
をみたすfを求めよ

948 :132人目の素数さん:2015/05/15(金) 11:09:42.78 ID:RnqkudAI.net
>>945 訂正
(1) ∀x, ∀y >0 に対して、f(x)f(y) = f(xy) + f(y/x)

949 :132人目の素数さん:2015/05/15(金) 11:11:33.25 ID:hq3tcZyV.net
R^+ の恒等写像だけはわかった。

950 :132人目の素数さん:2015/05/15(金) 11:11:37.48 ID:RnqkudAI.net
f : R → R
∀x, ∀y ∈R に対して、f(x^2) + f(xy) = f(x)f(x+y)
をみたすfを求めよ

951 :関数方程式:2015/05/15(金) 11:18:16.99 ID:RnqkudAI.net
ゴチャゴチャしてきたので、まとめる。 R^+は正の実数の集合とする。

>>945
次の2条件をみたす f : R^+ → R^+ を求めよ。
(1) ∀x, ∀y > 0 に対して、f(x)f(y) = f(xy) + f(y/x)
(2) 1<x<y ⇒ f(x)<f(y)

>>947
f : R^+ → R^+
∀x, ∀y >0 に対して、f(f(x)+f(y)) = x+y

>>950
f : R → R
∀x, ∀y ∈R に対して、f(x^2) + f(xy) = f(x)f(x+y)

>>951
f : R → R
∀x, ∀y ∈R に対して、f(f(x)-y) + y・f(2x) = f(x^2 +y)

952 :132人目の素数さん:2015/05/15(金) 16:47:53.10 ID:06V0dwp2.net
>>947
∀a, b∈R^+ (a<b), ∃s,t,u∈R^+ s.t. b-a=s+t+u
与式にx=f(a)+f(s), y=f(t)+f(u)を代入して
f(b)=f(a)+f(s)+f(t)+f(u)>f(a)
よってfは狭義単調増加, とくに単射.
∀r∈R^+, 与式にx=y=f(r)+f(r)を代入して f(4r)=4f(r) …(ア)
また, f(f(r)+f(4r))=f(f(2r)+f(3r))=5r fは単射なので f(r)+f(4r)=f(2r)+f(3r) …(イ)
同様にf(r)+f(3r)=f(2r)+f(2r) …(ウ)
(ア),(イ),(ウ)よりf(2r)=2f(r) …(エ)
∀n∈N(n≧2), f(r)+f(nr)=f(2r)+f((n-1)r) これと(エ)より
f(nr)=f(r)+f((n-1)r) よって帰納的にf(nr)=nf(r)
これより∀p,q∈Z^+, pf(1)=f(p)=f(p/q・q)=qf(p/q) すなわちf(p/q)=(p/q)f(1) …(オ)
ここで, f(1)=tとおく.与式にx=y=1を代入して(エ)を用いることでf(t)=1
fは狭義単調増加なのでt=1 よって(オ)より∀r∈Q^+, f(r)=r
∃x∈R^+, x<f(x) と仮定すると∃r∈Q^+, x<r<f(x)
fは狭義単調増加なのでf(x)<f(r)=r これは矛盾.
∃x∈R^+, x>f(x) と仮定しても同様に矛盾.
よって∀x∈R^+, f(x)=x 逆にこれは与式を満たす.

953 :132人目の素数さん:2015/05/15(金) 19:23:50.63 ID:06V0dwp2.net
>>950
(i)f(1)=0のとき
与式にx=1を代入してf(y)=0 よって∀x∈R, f(x)=0 これは与式を満たす.
(ii)f(0)=2のとき
与式にx=0を代入してf(y)=2 よって∀x∈R, f(x)=2 これは与式を満たす.
(iii)f(1)≠0かつf(0)≠2のとき
与式にx=y=0を代入してf(0)(f(0)-2)=0 f(0)≠2よりf(0)=0
与式にx=1, y=0を代入してf(1)(f(1)-1)=0 f(1)≠0よりf(1)=1
∀n∈Z, 与式にx=1, y=nを代入して1+f(n)=f(n+1)
よって帰納的に∀n∈Z, f(n)=n
∀p∈Z, q∈N, 与式にx=q, y=p/q-qを代入してf(p/q)=p/q
よって∀r∈Q, f(r)=r

t∈R^+とする.
与式にx=√t, y=0を代入してf(t)=f(√t)^2≧0…(ア)
与式にx=√t, y=-√tを代入してf(-t)=-f(t)…(イ)
ここでf(t)=0と仮定すると, 与式にx=t, y=1/tを代入して
f(t^2)+f(1)=0 これと(ア)よりf(1)=0となって矛盾.よってf(t)>0…(ウ)

a,b∈R(0≦a<b)とする.
与式にx=√b, y=-a/√bを代入して(イ)より
f(b)-f(a)=f(√b)f((b-a)/√b)
√b>0, (b-a)/√b>0なので(ア), (ウ)よりf(√b)f((b-a)/√b)>0
よってf(a)<f(b) ゆえにfはx≧0で狭義単調増加.
>>952 と同様にして∀x∈R^+, f(x)=x これと(イ)より
∀x∈R, f(x)=x これは与式を満たす.
以上より, 求めるfは f(x)=0, f(x)=2, f(x)=x

954 :132人目の素数さん:2015/05/15(金) 21:09:45.92 ID:06V0dwp2.net
>>951
t∈Rとする.
与式にx=t, y=-t^2を代入して f(f(t)+t^2)-t^2f(2t)=f(0)
与式にx=t, y=f(t)を代入して f(0)+f(t)f(2t)=f(f(t)+t^2)
これらよりf(2t)(f(t)-t^2)=0 よって∀t∈R, f(2t)=0 またはf(t)=t^2…(☆)
∀t∈R, f(2t)=0 のとき
f(x)=0 これは与式を満たす.
∀t∈R, f(t)=t^2 のとき
f(x)=x^2 これは与式を満たす.
∃a,b∈R (a,b≠0), f(2a)=0 かつ f(b)=b^2 のとき
与式にx=a, y=f(a)-bを代入して
b^2=f(a^2-b+f(a))
b≠0よりa^2-b+f(a)=bまたは−b
a^2-b+f(a)=bのとき, a^2+f(a)=2bより, bとしてありうる値は高々1つなので
∀x∈R(x≠0, x≠b), f(x)=-x^2+2b これは(☆)に矛盾.
a^2-b+f(a)=−bのとき,a^2+f(a)=0
(☆)よりf(a)≧0なのでa=0となり矛盾.
以上より求めるfは f(x)=0, f(x)=x^2 

955 :132人目の素数さん:2015/05/15(金) 22:53:50.39 ID:06V0dwp2.net
>>945
(x,y)=(1,1),(2,2),(√2,2),(√2,2√2)をそれぞれ代入してあれこれいじるとf(2)>2が示される
あとは2004 春合宿 と本質的に同じ
解答は黄色い本に書いてある

956 :132人目の素数さん:2015/05/15(金) 22:58:32.97 ID:xZK5Qx9q.net
f(2)>f(1)=2 はあれこれするまでもないが

957 :132人目の素数さん:2015/05/15(金) 23:34:07.43 ID:RnqkudAI.net
>>955
黄色い本って何ですか?

958 :132人目の素数さん:2015/05/16(土) 08:43:26.07 ID:pViSrwFE.net
>>956
(2)の条件の1<xに意地悪く等号がついてない(春合宿と違う所)からあれこれしなきゃならない

959 :132人目の素数さん:2015/05/16(土) 08:45:16.24 ID:pViSrwFE.net
>>957
大きめの本屋の数学コーナーに行ったら一際目立つ真っ黄色な本があるからそれ

960 :132人目の素数さん:2015/05/16(土) 08:54:59.19 ID:Vp+m9OwM.net
>>959
数学書で黄色い本と言ったら、サイエンス社の演習書しか思いつかないなぁ

961 :132人目の素数さん:2015/05/16(土) 09:04:50.91 ID:pViSrwFE.net
http://www.asakura.co.jp/G_12.php?isbn=ISBN978-4-254-11132-3

962 :132人目の素数さん:2015/05/16(土) 09:06:34.65 ID:QCuUaG62.net
>>958
確かにそうだな、勝手にx=1での連続性を移入してたわwww

963 :132人目の素数さん:2015/05/16(土) 09:18:36.31 ID:Vp+m9OwM.net
>>961
おー、こんな本があったのか。さんくそ。

964 :132人目の素数さん:2015/05/16(土) 09:22:05.86 ID:Vp+m9OwM.net
>>572
リンクを色々みたが、AM-GMを用いた証明が分かりません。

965 :132人目の素数さん:2015/05/16(土) 13:11:58.30 ID:Y5sGel31.net
>>745
(3) f(x)=(x^2+x+1)^n (n:自然数)

複素数 a を f(x)=0 の解とすると、f(a)=0 より f(a^2)=0
よって a, a^2, a^4, a^8,・・・ はすべて f(x)=0 の解となるが
f(x)=0 の解は有限個なので |a|=1 または a=0
a=0が解なら、f(4)=f(1)f(2)=f(0)f(1)f(2)=0 より
a=4 も解となるが、|a|=1 に矛盾
また、f((x+1)^2)=f(x)f(x+1)より、f(a)=0なら f((a+1)^2)=0
よって(a+1)^2も解となるので、|a+1|=1
以上よりf(x)=0の複素数解は |a|=|a+1|=1をみたすので
a=ω,ω^2 (1の3乗根)の2つしかない
以上よりf(x)は k(x^2+x+1)^n (kは定数) とかける
逆にk=1とすれば、与えられた条件式をみたす

966 :132人目の素数さん:2015/05/16(土) 13:27:14.45 ID:Y5sGel31.net
>>745
(2) f(x)=x^n
f(x) = a(n)・x^n + a(n-1)・x^(n-1) + ・・・ + a(1)・x + a(0)
とおいて係数比較すると、a(n)=1, a(n-1)= ・・・ =a(0)=0 となる 

(4) f(x)=(x-1)(x-2)(x-3)(x-4)(x-5)(x-6)(x-7)
x=8を代入して、f(7)=0  x=0を代入して、f(0)=0
x≠0,8のとき f(x-1)=0 ⇔ f(x)=0  
f(x)=0の解は有限個しかないので x=0,1,2,3,4,5,6,7 のみ

967 :132人目の素数さん:2015/05/16(土) 13:31:46.45 ID:Y5sGel31.net
>>966 訂正
(4) f(x)=x(x-1)(x-2)(x-3)(x-4)(x-5)(x-6)(x-7)

968 :132人目の素数さん:2015/05/16(土) 13:39:15.31 ID:u6Mc3ZNf.net
おもすれー!

969 :132人目の素数さん:2015/05/16(土) 15:13:04.35 ID:Vp+m9OwM.net
また一人数学沼に引きずり込まれようだ。今回の餌食は関数方程式の沼

                       _
                   /)/ノ,>    す  嵌
                   | lン.)っ   で   っ
                   |/ .〈/    に.  て
                      l   )    泥.  い
                   |iii||||||    中  る
                     |iii||||||        :
                    |ii||||||||      首.  :
                ┌|ii|||||||||      ま   :
       、〜"イイ__   /iiii|||||||||||     で   :
      ゝ:::::::::::レヘ .|__「iii|||||||||||||      :   :
;;;;;;;;;;;;;;;;;;;;;イ::::::::</(ン ,_,<||||||||||||l'';;;;;;;,,,,,   :
;;:;:::;;;;;:::,,,;;;ム;/);;;モヽ u (ニ'ヘ||||||||l;;;::;;;;;;;..''':;;;,
::::;;;;;;:;;;;;..,,,,;;;;/フ;;;;;/);;;;:'''''::::;;;;;;;:::::'';;:;;;;;;,,::::;;;;;,,,,
;;;;;:::;;;;;;;::::::'''',,尸;;;;;';;''::::::;;;;;;;;'';;;::::;;;;;;;;::;;;:;;;;::::::''';;;;;;;
::;;;;;;;;:::;;;;;;;;;'';;;つ;;::::;;;''';;;;;;,,,'':::::::;;'';;;;;;::::;;::;;;''::::;;;;;;;

970 :132人目の素数さん:2015/05/16(土) 15:18:15.87 ID:pViSrwFE.net
(n+1)^k=n!+1 を満たす正の整数n,kを全て求めよ

971 :132人目の素数さん:2015/05/16(土) 18:46:22.31 ID:MyWjm7uV.net
nCk=m^l を満たす自然数の組(n,k,m,l)を全て求めよ
(ここでnCkは二項係数)

972 :132人目の素数さん:2015/05/17(日) 22:10:16.82 ID:rkDPdzLp.net
>>970
n>4のときn!+1は奇数→n+1は奇数→nは偶数
2<n/2<nよりn!は2・n/2・n=n^2の倍数
両辺をmod n^2でみて(二項定理より)n|k
よってn≦k
n^n<(左辺)=(右辺)<n^n+1より不適
n≦4のとき(n,k)=(1,1),(2,1),(4,2)

973 :132人目の素数さん:2015/05/17(日) 23:31:58.77 ID:inO97hsW.net
俺にも性奴隷をまわしてくれ
ナマポも頼むわ

974 :132人目の素数さん:2015/05/18(月) 16:28:56.68 ID:hoxK9OGH.net
四角形ABCDはAB=CD、∠ABC=132°、∠CAD=24°、∠ADC=102°をみたす。このとき、∠BCAを求めよ。
ただし、初等幾何で解くとほうびがもらえるとする。

975 :132人目の素数さん:2015/05/18(月) 17:27:06.05 ID:BC+ZL+kX.net
>>974
正弦定理を使って解いて、18°
初等幾何による解法に期待。

976 :132人目の素数さん:2015/05/18(月) 17:35:24.75 ID:sEoGZx57.net
また正五角形が出てくるのか?

977 :132人目の素数さん:2015/05/19(火) 02:44:18.75 ID:2VXN+23x.net
>>974
凸六角形PQRSTUであって、△PQUが正三角形、五角形QRSTUが正五角形であるものを考えると、四角形PQST∽四角形ABCDが簡単な角度の計算でわかる。
よって、∠BCA=∠QSP=18°

978 :132人目の素数さん:2015/05/19(火) 03:31:24.81 ID:pliD+kpe.net
>>977
やられた。

>>974
汚い補助線引きまくりの方法を概略のみ。

最初は△DACのみに着目する。

△DACの外心をEとし、AEを一辺とする正三角形FAEをAEから見てDと同じ側に作ると、
Eを中心とする円周上にA,F,D,Cがこの順にあり、
∠AEF=60°、∠FED=∠DEC=48°という状況ができるので
円周角と中心角の関係からいろんな角度がわかる。

次に∠GEA=∠EAG=6°の二等辺三角形GEAをAEから見てDと逆側に作ると、
∠AGE+∠ECA=180°よりA,G,E,Cは共円。
∠FGC=∠CFG=72°よりCF=CG

Aを通りFGと平行な直線と直線DFとの交点をHとすると、
∠AGF=∠GFH=84°より、四角形AGFHはAH//GF,AG=HFの等脚台形となり、
△FHC≡△GAC

あとは、CH=AC、CD=AB、∠CHD=132°=∠ABCより、△CDH≡△ABCとなり、
∠BCA=∠DHC=∠GAC=18°

979 :132人目の素数さん:2015/05/19(火) 16:22:22.52 ID:2VXN+23x.net
2015個の相異なる正の整数であって, それらの和が平方数, 積が立方数であるようなものが存在することを示せ.

980 :132人目の素数さん:2015/05/19(火) 18:41:49.38 ID:suW/IKzl.net
>>979
2,8,54,64,128,...,2^2017

981 :132人目の素数さん:2015/05/19(火) 18:51:09.37 ID:XKcvxMhG.net
>>979
1^3, 2^3, ……, 2015^3

982 :132人目の素数さん:2015/05/20(水) 02:23:09.56 ID:1Uhc0RBr.net
2015個の相異なる正の整数であって, それらの和が立方数, 積が平方数であるようなものが存在することを示せ. 

983 :132人目の素数さん:2015/05/20(水) 14:11:59.84 ID:ztZEQGaJ.net
>>982
たとえば
11,13,15,…,2019,2021←1006個
18,22,26,30,…,4038,4042←1007個
ここまでの全ての和をSとし、十分大きい数Nをとって
あと2個を
(27N^3-S)/3と、2(27N^3-S)/3
とすればよい。

984 :132人目の素数さん:2015/05/20(水) 14:23:25.27 ID:ztZEQGaJ.net
まあ、十分に大きい数Nを使えば
a(i) = i(i=1,…,2012)
a(2013) = 3*4*5*6*…*2012
S=a(1)+a(2)+…+a(2012)
a(2014)=(27N^3-S)/3
a(2015)=2(27N^3-S)/3
とかでもいいわな

985 :132人目の素数さん:2015/05/20(水) 14:54:06.48 ID:ztZEQGaJ.net
1,3,5,7,
11,13,15,17,19,21,…,2013,
2,6,10,14,18,22,…,4026,
44795,89590

986 :132人目の素数さん:2015/05/20(水) 15:09:52.03 ID:1Uhc0RBr.net
S=1^2+2^2+・・・+2015^2 とおいて
S^2, (2S)^2, ... , (2015S)^2

987 :132人目の素数さん:2015/05/20(水) 15:17:54.14 ID:1Uhc0RBr.net
任意の正の整数nに対して,
Πcos(kπ/(2n+1))=1/(2^n)
が成り立つことを示せ.
ただし積はk=1,2,...,nについてとる.

988 :132人目の素数さん:2015/05/20(水) 16:05:01.63 ID:BZe1cI9J.net
>>982
16, 3・4^2, 3・4^3, ..., 3・4^2014, 3・4^2015
和:4^2016=(4^672)^3
積:(3^2014)・(4^2031121)=((3^1007)・(2^2031121))^2

989 :132人目の素数さん:2015/05/20(水) 17:58:58.23 ID:BZe1cI9J.net
>>987
cos(kπ/(2n+1)) = −cos((2n+1−k)π/(2n+1)) k=1,2,...,n
より、Π[k=1〜n]cos(kπ/(2n+1)) = (−1)^n・Π[k=n+1〜2n]cos(kπ/(2n+1))
Π[k=1〜n]cos(kπ/(2n+1)) = P とおくと、
P^2 = (−1)^n・Π[k=1〜2n]cos(kπ/(2n+1)) となる
以下、積と和は k=1,2,...,2n について取る
P^2 = (−1)^n・2^(−2n)・Π(exp(ikπ/(2n+1)) + exp(−ikπ/(2n+1)))
= (−1)^n・2^(−2n)・expΣ(−ikπ/(2n+1))・Π(exp(2ikπ/(2n+1)) + 1)
= (−1)^n・2^(−2n)・exp(−inπ)・Π(exp(i2kπ/(2n+1)) + 1)
= 2^(−2n)・Π(exp(i2kπ/(2n+1)) + 1) ・・・(※)
ここで、f(x) = x^(2n+1) − 1 とおくと、f(x) = (x−1)・Π(x − exp(ikπ/(2n+1)))
f(−x) = −(x+1)・Π(x + exp(ikπ/(2n+1)))
f(x)f(−x) = −(x^2−1)・Π(x^2 − exp(2ikπ/(2n+1)))
x=i を代入すると、
f(i)f(−i) = (i^(2n+1)−1)(−i^(2n+1)−1) = −(−1−1) = 2
= 2・Π(−1 − exp(2ikπ/(2n+1))) = 2・Π(exp(i2kπ/(2n+1)) + 1)
よって、Π(exp(i2kπ/(2n+1)) + 1) = 1
(※)より、P^2 = 2^(−2n)  P > 0 より P = 1/(2^n)

990 :132人目の素数さん:2015/05/20(水) 20:12:26.84 ID:cSxGMp8Z.net
倍角公式使えば5行くらいで示せるよ

991 :132人目の素数さん:2015/05/20(水) 20:40:09.85 ID:DEcziQZO.net
嘘を証明してしまうとは

992 :132人目の素数さん:2015/05/20(水) 21:04:31.23 ID:34ydGlLu.net
>>982
1,3,12,16,32,...,2^2015

>>979かつ>>982
64,128,288,1024,2592,4096,8192,...,2^2021

993 :132人目の素数さん:2015/05/20(水) 23:32:15.56 ID:/8iBr8Zz.net
>>991
問題が間違っているってこと?

994 :132人目の素数さん:2015/05/21(木) 01:06:21.42 ID:rY9i59H1.net
【解答】
sin(kπ/(2n+1))=sin((2n+1-k)π/(2n+1))をkが奇数のときに適用して、
Πsin(kπ/(2n+1))
=Πsin(2kπ/(2n+1))
=2^nΠsin(kπ/(2n+1))Πcos(kπ/(2n+1)) (∵倍角公式)
Πsin(kπ/(2n+1))≠0よりΠcos(kπ/(2n+1))=1/(2^n) 

995 :132人目の素数さん:2015/05/21(木) 02:47:17.04 ID:4qvmnIFl.net
http://manjitoushikeiba.blog.fc2.com/blog-entry-69.html?sp
これの2着率って、この条件じゃ出せない筈だよな?

996 :132人目の素数さん:2015/05/21(木) 13:48:56.13 ID:rY9i59H1.net
立方体をある平面で切断したとき、切断面は正五角形になりえないことを示せ。

997 :132人目の素数さん:2015/05/21(木) 13:58:55.02 ID:tDYp4Jaj.net
994さんの質問の答えわかりません
なぜ僕は示せないんでしょう? 示せる人は賢いんですか?

998 :132人目の素数さん:2015/05/21(木) 14:17:42.91 ID:ASW65NaJ.net
座標で書けば一発ちゃう?

999 :132人目の素数さん:2015/05/21(木) 19:20:37.22 ID:Zw/UOBUU.net
>>996
実在するどんな立方体を切断したとしても、原子レベルでは正五角形にはなりえない
はい論破

1000 :132人目の素数さん:2015/05/21(木) 19:34:25.61 ID:4jbcQn4x.net
まるで立方体は実在するかのような主張
笑いのセンスだけでなく注意力までないとは…

1001 :132人目の素数さん:2015/05/21(木) 21:01:23.62 ID:c6nn8WA7.net
>>1000
こいつ最高にアホ!

1002 :132人目の素数さん:2015/05/21(木) 21:22:10.84 ID:yENOyrzH.net
あほ

1003 :2ch.net投稿限界:Over 1000 Thread
2ch.netからのレス数が1000に到達しました。

総レス数 1003
251 KB
掲示板に戻る 全部 前100 次100 最新50
read.cgi ver 2014.07.20.01.SC 2014/07/20 D ★